Инфоурок Математика Другие методич. материалыМЕТОДИЧЕСКИЕ УКАЗАНИЯ по выполнению практических работ по математике

МЕТОДИЧЕСКИЕ УКАЗАНИЯ по выполнению практических работ по математике

Скачать материал

Министерство образования Республики Башкортостан

Баймакский филиал

Государственное автономное профессиональное образовательное учреждение

Уфимский топливно-энергетический колледж

 

 

 

 

ОДОБРЕНО

методической цикловой комиссией

Председатель МЦК

________Михайлова Е.М.

«___» ________ 2022г.

УТВЕРЖДАЮ

Зам. заведующегопо УПР

_______ Гайнуллина Н.Ю. «___» ___________  2022г.

 

 

 

 

 

 

                                                                                                                                                                                                                                                                                                                                                                                                                                                                                                                                                                                                                                                                                                                                                                                                                                                                                                                                                                                                                                                                                                                                                                                                                                                                                                                                                                                                                                                                                                                                                                                                                                                                                                                                                                                                                                                                                                                                                                                                                                                                                                                                                                                                                                                                                                                                                                                                                                                                                                                                                                                                                                                                                                                                                                    

 

 

 

 

 

 

методические указания

по выполнению практических работ

по математике

 

Специальность: 13.02.11 Техническая эксплуатация и обслуживание электрического и электромеханического оборудования

 

 

 

 

 

 

 

 

 

 

 

 

 

 

 

 

 

2022 г.

 

Введение

 

Математика – наука о количественных отношениях и пространственных формах действительного мира. Советский академик А.Н.Колмогоров выделяет четыре периода развития математики: зарождение математики, элементарная математика, математика переменных величин, современная математика.

Понимание самостоятельного положения математики как особой науки стало возможным после накопления достаточно большого фактического материала и возникло впервые в Древней Греции в IV – V веках до нашей эры. Это было началом периода элементарной математики.

В течение этого периода математические исследования имеют дело лишь с достаточно ограниченным запасом основных понятий, возникших в связи с самыми простыми задачами хозяйственной жизни. Вместе с тем уже происходит качественное совершенствование математики как науки. Из арифметики постепенно вырастает теория чисел. Создается алгебра как буквенное исчисление. А созданная древними греками система изложения элементарной геометрии – геометрии Евклида – на два тысячелетия вперед сделалась образцом дедуктивного построения математической теории.

В XVII веке запросы естествознания и техники привели к созданию методов, позволяющих математически изучать движение, процессы изменения величин, преобразование геометрических фигур. С употреблением переменных величин в аналитической геометрии и создании дифференциального и интегрального исчисления начинается период математики переменных величин.

На первый план выдвигается понятие функции. Изучение функции приводит к основным понятиям математического анализа: пределу, производной, дифференциалу, интегралу. Создание аналитической геометрии позволило существенно расширить предмет изучения геометрии благодаря найденному универсальному способу перевода вопросов геометрии на язык алгебры и анализа – методу координат Р.Декарта. С другой стороны, открылась возможность геометрической интерпретации алгебраических и аналитических фактов.

XIX – XX века открывают период современной математики. Современная математика играет важную роль в естественнонаучных, инженерно – технических, гуманитарных исследованиях. Без современной математики с ее развитым логическим и вычислительным аппаратом был бы невозможен прогресс в различных областях человеческой деятельности. Современная математика изучает математические модели. Это могут быть как непосредственно математические модели реальных явлений, так и объекты для изучения этих моделей. Одна и та же модель может описывать свойства далеких друг от друга по своему конкретному содержанию реальных явлений. Для математики важна не природа рассматриваемых объектов, а существующие между ними отношения.

Математика является не только мощным средством решения прикладных задач и универсальным языком науки, но также и элементом общей культуры. Поэтому математическое образование является важнейшей составляющей в системе фундаментальной подготовки современного специалиста.

В данных методических указаниях вы найдете изложение теоретического материала, справочный материал, примеры решения задач, задания для самостоятельных занятий, для подготовки к контрольным работам, зачету, экзамену.

Методические указания не являются учебником, поэтому не все изучаемые понятия рассмотрены одинаково подробно. По этой причине в некоторых случаях необходимо приложить для освоения материала больше усилий, чем в других. В данном пособии рассматриваются элементы математики, относящиеся к периоду математики переменных величин и современному периоду, имеющие большое значение в современной фундаментальной и прикладной математике.

Работая над каждой темой, лучше всего сначала изучить теоретический материал, повторить ранее изученные формулы, теоремы, разобраться в приведенных примерах. Если все понятно, то можно переходить к выполнению практических заданий.

Академик И.П. Павлов говорил: «Последовательность, последовательность и последовательность. С самого начала своей работы приучите себя к строгой последовательности в накоплении знаний. Никогда не беритесь за последующее, не изучив предыдущего».

Учебные и воспитательные цели практических занятий

В рамках традиционного подхода:

1) актуализировать знания студентов из курса математики по теме занятия;

2) создать условия для развития творческой активности, самостоятельности и критичности мышления, умения работать в коллективе.

В рамках компетентностного подхода:

1) содействовать развитию у студентов общенаучных компетенций (аналитико-синтетической, прогностической, проектировочной);

2) создать условия для развития коммуникативной, адаптивной и информационной компетенций.

Данные указания предназначены для использования в средних профессиональных учебных заведениях, в учебных планах которых предусмотрена дисциплина «Математика», соответствующая действующим программам. Представленные в указаниях основные математические структуры имеют настолько большую общеобразовательную и математическую значимость, что являются обязательными для рассмотрения студентами всех специальностей.

 

Требования к оформлению практических работ

После изучения соответствующей темы студенты выполняют практическую работу. Содержание практических работ полностью соответствует рабочей программе по математике.

К выполнению практической работы можно приступать только после изучения соответствующей темы и получения навыков решения задач. Предусмотренные задания носят репродуктивный, частично-поисковый и поисковый характер. Все задачи и расчеты обязательно должны быть доведены до окончательного числового результата. Вариант практической работы определяется по последней цифре порядкового номера списочного состава в журнале учебных занятий.

Все практические работы, сдаваемые студентами на проверку, должны быть выполнены в обычной тетради в клетку (96 листов).

При выполнении практической работы студентам рекомендуется:

-  использовать учебные пособия, справочники;

-  проводить несложные дедуктивные рассуждения;

-  обосновывать шаги решения задач;

-  формулировать определения математических понятий;

-  пользоваться математической терминологией и символикой;

-  письменно оформлять решения задач;

-  пользоваться калькулятором;

-  самостоятельно изучать учебный материал.

Все представленные варианты практических работ даны одинаковой степени трудности.

Практическая работа выполняется в сроки, установленные в соответствии с календарно-тематическим планом. За каждую практическую работу студент должен получить положительную оценку.

Итоговой формой изучения дисциплины является экзамен. Студенты, не выполнившие все практические работы, не аттестуются и к экзамену не допускаются.

 

Помните, что «царского пути» в математике нет и дорогу осилит только упорно идущий! Но, с другой стороны, не так страшна математика как ее малюют. Искренне желаю успехов!


Практическое занятие № 1

Действительные числа. Приближенные вычисления

 

Цель работы:

студент должен:

знать:

-       формулы для вычисления границ абсолютной и относительной погрешности суммы, разности, произведения и частного приближенных значений чисел;

уметь:

-       вычислять сумму, разность, произведение и частное приближенных значений чисел.

 

Сведения из теории:

Сложение приближенных значений чисел

Граница абсолютной погрешности суммы приближенных значений чисел равна сумме границ абсолютных погрешностей этих чисел:

Δ(a+b)=Δab,

где a и bприближенные значения чисел; Δa и Δb – границы абсолютных погрешностей соответствующих приближений.

Граница относительной погрешности сумы вычисляется по формуле:

εa+b=.

 

Пример 1.

Найти сумму S приближенных значений чисел 6,8±0,05; 4,3±0,05 и 3,575±0,0005.

Решение:

вычислим сумму заданных чисел и сумму их погрешностей:

S=6,8+4,3+3,575=14,675;

ΔS=0,05+0,05+0,0005=0,1005.

Граница абсолютной погрешности заключена в пределах 0,05<0,1005<0,5. В приближенном значении суммы верными являются лишь две цифры (в разрядах десятков и единиц). Полученный результат округлим до единиц S=14,675≈15.

 

Вычитание приближенных значений чисел

Граница абсолютной погрешности разности двух приближенных значений чисел равна сумме границ их абсолютных погрешностей:

Δ(a-b)=Δab.

Граница относительной погрешности разности вычисляется по формуле:

εa-b=.

 

Пример 2.

Вычислить разность двух приближенных значений чисел a=5,863±0,0005 и b=2,746±0,0005. Найти Δ(a-b) и εa-b.

Решение:

вычисляем границу абсолютной погрешности разности a-b:

Δ(a-b)=0,0005+0,0005=0,001.

В приближенном значении разности цифра в разряде тысячных не может быть верной, так как Δ(a-b)>0,0005. Итак, a-b=3,117≈3,12. Абсолютная погрешность разности 0,001. В приближенном числе 3,12 все цифры верные. Находим относительную погрешность разности:

εa-b==0,00032≈0,03%.

 

Умножение приближенных значений чисел

Формулы для оценки границ абсолютной погрешности произведения (частного) сложны, поэтому на практике сначала находят относительную погрешность произведения (частного), а затем границу абсолютной погрешности произведения (частного).

Формулы для границ абсолютной и относительной погрешности некоторых функций приведены в таблице 1.

 

Таблица 1. Формулы для границ абсолютной и относительной погрешностей.

№ п/п

Функция

Граница абсолютной погрешности

Граница относительной погрешности

1

y=ab

Δy=|bΔa+|aΔb

2

y=abc

Δy=|bcΔa+|acΔb+|abΔc

3

y=an

Δy=n an-1·Δa

4

y=a2

Δy=2a·Δa

5

y=a3

Δy=3a2·Δa

6

y=

Δy=

7

y=

Δy=

8

y=

Δy=

 

Пример 3.

Найти верные цифры произведения приближенных значений чисел a=0,3862 и b=0,8.

Решение:

имеем 0,3862·0,8=0,30896. Границы абсолютной погрешности сомножителей равны 0,00005 и 0,05. По формуле  находим относительную погрешность произведения:

.

Находим границу абсолютной погрешности произведения:

Δ(ab)=0,30896·0,063=0,0195;

0,0050,0195 0,05.

Полученный результат означает, что в произведении одна верная цифра (в разряде десятых): 0,30896≈0,3.

 

Пример 4.

Вычислить объем цилиндра V= πR2H, если R=45,8 см, H=78,6 cм.

Решение:

по формуле объема цилиндра, имеем

V= π·45,82·78,6=517000 (см3).

Используя формулу  и полагая π≈3,14, находим относительную погрешность:

.

Находим границу абсолютной погрешности: 

ΔV=V·εV=517 000·0,0044 = 2270 (см3).

Верными цифрами являются 5 и 1.

 

Деление приближенных значений чисел

 

Пример 5.

Найти границу абсолютной погрешности частного приближенных значений чисел a=8,36±0,005 и b=3,72±0,004.

Решение:

имеем 8,36:3,72=2,25.

По формуле  находим относительную погрешность частного:

.

Находим границу абсолютной погрешности частного:

Δ(a/b)=2,25·0,002=0,0045.

Полученный результат означает, что в частном все три цифры верные.

 

Пример 6.

Вычислить X=, если известно, что a=7,2±0,05, b=3,46±0,03, с=5,09±0,04.

Решение:

находим ;

;

ΔX=X·εX=0,844·0,015=0,0127; X=0,844±0,0127 или X0,84±0,01.

 

Задания для самостоятельного решения:

Вычислите сумму, разность, произведение и частное приближенных значений чисел:

1 вариант

 с четырьмя значащими цифрами.

2 вариант

0,456±0,0005 и 3,35±0,005.

3 вариант

 с четырьмя значащими цифрами.

4 вариант

8,72 и 2,6532, границы абсолютной погрешности которых соответственно равны 0,005 и 0,00005.

5 вариант

6,54±0,005; 16,022±0,0005 и 1,9646±0,00005.

6 вариант

, взяв приближенные значения корней с точностью до 0,001.

7 вариант

 с четырьмя значащими цифрами.

8 вариант

a=19,8±0,05 и b=48,4±0,03.

9 вариант

a=68,4±0,02 и b=72,8±0,4.

 

Контрольные вопросы:

1.     Перечислите действия над приближенными значениями чисел.

2.        Перечислите формулы для вычисления границ абсолютной и относительной погрешностей некоторых функций.

Практическое занятие № 2

Действия над комплексными числами

 

Цель работы:

студент должен:

знать:

-       алгебраическую форму комплексного числа;

-       тригонометрическую форму комплексного числа;

уметь:

-       выполнять действия над комплексными числами, представленными в различных формах.

 

Сведения из теории:

Алгебраическая форма комплексного числа

Обозначим  и назовём мнимой единицей, (). Тогда число вида  где - любые действительные числа, назовём комплексным числом.

Здесь а - называют действительной частью комплексного числа,  - называют мнимой частью, b - коэффициентом мнимой части комплексного числа.

 

Действия над комплексными числами, представленными в алгебраической форме

Пусть даны два числа  и .

Для этих чисел понятия равенство и действия сложения, умножения определены следующим образом:

1)       Два комплексных числа называются равными, если равны их действительная и мнимая части, т. е. а12, b1=b2.

2)       Суммой двух комплексных чисел z1 и z2 называется комплексное число .

3)       Произведением двух комплексных чисел z1 и z2 называется комплексное число .

4)       Модулем комплексного числа называется длина вектора соответствующего этому комплексному числу на плоскости и вычисляется по формуле: .

5)       Аргументом комплексного числа называется угол, образованный вектором с положительным направлением действительной оси и вычисляется по формуле: . Т. о. для каждого комплексного числа можно указать бесконечное множество аргументов.

Для нахождения аргумента необходимо:

1.   Определить в какой координатной четверти находится комплексное число.

2.   Найти в этой четверти угол решив уравнение:

.

 

Пример 7.

Решите квадратное уравнение: .

Решение:

вычислим корни квадратного уравнения через дискриминант:

 

Получена пара взаимно - сопряжённых комплексных чисел  где

Заметим, что всякое алгебраическое уравнение степени n имеет ровно n корней, среди которых могут быть как действительные (различные или равные), так и комплексные (обязательно попарно взаимно – сопряжённые) корни.

 

Тригонометрическая форма комплексного числа

Запись комплексного числа в виде  называется тригонометрической формой комплексного числа.

Действия над комплексными числами, представленными в тригонометрической форме

Над комплексными числами в тригонометрической форме выполняются действия умножения, деления, возведения в степень и извлечение корня n-ой степени.

Пусть даны два числа  и , тогда:

1) Произведением комплексных чисел называется комплексное число, которое вычисляется по формуле: .

2) Частным комплексных чисел называется комплексное число, которое вычисляется по формуле: .

3) Для возведения в степень: .

 

Пример 8.

Упростите: .

Решение:

упростим дробь (понизим степень числителя и знаменателя), используя ():

.

Подставим полученные выражения в исходную дробь и преобразуем её:

.

 

Пример 9.

Вычислите: .

Решение:

для первого комплексного числа используем формулу возведения в степень, а затем воспользуемся формулой произведения комплексных чисел:

 

Для извлечения корня n-й степени из комплексного числа используется формула:

,

где - арифметический корень, .

 

Пример 10.

Решите уравнение: х2-2х+10=0.

Решение:

для решения воспользуемся обычными формулами вычисления корней квадратных уравнений:

Получили пару комплексных взаимно сопряженных корней.

 

Задания для самостоятельного решения:

1 вариант

№1. Выполните действия, вычислите аргумент и модуль комплексного числа:

1) ;

2) ;

3) .

№2. Решите уравнение:

х2-6х+13=0.

2 вариант

№1. Выполните действия, вычислите аргумент и модуль комплексного числа:

1) ;

2) ;

3) .

№2. Решите уравнение:

х2+3х+4=0.

4 вариант

№1. Выполните действия, вычислите аргумент и модуль комплексного числа:

1) ;

2) ;

3) .

№2. Решите уравнение:

х2-4х+16=0.

3 вариант

№1. Выполните действия, вычислите аргумент и модуль комплексного числа:

1) ;

2) ;

3) .

№2. Решите уравнение:

9х2+12х+29=0.

5 вариант

№1. Выполните действия, вычислите аргумент и модуль комплексного числа:

1) ;

2) ;

3) .

№2. Решите уравнение:

2,5х2+х+1=0.

6 вариант

№1. Выполните действия, вычислите аргумент и модуль комплексного числа:

1) ;

2) ;

3) .

№2. Решите уравнение:

х2-2х+4=0.

7 вариант

№1. Выполните действия, вычислите аргумент и модуль комплексного числа:

1) ;

2) ;

3) .

№2. Решите уравнение:

х2-4х+13=0.

8 вариант

№1. Выполните действия, вычислите аргумент и модуль комплексного числа:

1) ;

2) ;

3) .

№2. Решите уравнение:

4х2-20х+26=0.

9 вариант

№1. Выполните действия, вычислите аргумент и модуль комплексного числа:

1) ;    2) ;   3) .

№2. Решите уравнение:

х2-2х+26=0.

 

Контрольные вопросы:

1.   Дайте определение алгебраической форме комплексного числа.

2.   Перечислите действия над комплексными числами, представленными в алгебраической форме.

3.   Дайте определение тригонометрической форме комплексного числа.

4.   Перечислите действия над комплексными числами, представленными в тригонометрической форме.

 

Практическое занятие № 3

Степени с действительными показателями, их свойства

 

Цель работы:

студент должен:

знать:

-       основные показательные тождества;

-       свойства степеней с действительными показателями;

уметь:

-       вычислять степени с действительными показателями.

 

Сведения из теории:

Повторим определения понятия степени с натуральным, нулевым, целым отрицательным и рациональным показателями:

an= a a a∙…∙a; a-n=1/(an); a0=1, a≠0; am/n=,

 

                                          n раз

mЄZ, nЄN, n≥2.

Повторим свойства степеней с рациональным показателем:

при любых х и y справедливы равенства:

axay=ax+y;

ax/ay=ax-y;

(ax)y=axy;

(ab)x=axbx;

(a/b)x=ax/bx.

 

Степень с действительным показателем

Свойства степеней с действительным показателем:

1. ax/y=a(xk)/(yk), a>0, y, kЄN, xЄZ.

2. ax>0, a>0, xЄR (любая степень положительного числа положительна).

3. ax>1 при a>1, x>0.

4. ax<1 при a>1, x<0.

5. 1x=1 (любая степень единицы равна единице).

6. ax<1 при 0<a<1, x>0.

7. ax>1 при 0<a<1, x<0.

8. Если a>1, a≠1, то для любого положительного числа b существует единственное действительное число х такое, что ах=b при b>0.

9. Любая положительная степень нуля равна нулю.

 

Кроме перечисленных свойств важно отметить три свойства, на которых основано решение простейших показательных уравнении и неравенств:

10. Если ax=ay, то x=y при a>0, x, y≠1.

11. Если ax<ay, то x<y при a>0.

12. Если ax<ay, то x>y при 0<a<1.

 

Правила действия над степенями с действительным показателем выражаются формулами (тождествами):

13. aαaβ=aα+β.

14. aα ׃aβ=aα-β.

15. (aα)β=aαβ.

16. (ab)α=aαbα при a>0, b>0.

17. |ab|α=|a|α |b|α при ab>0.

18. (a/b)α=aα /bα при a>0, b>0.

19. |a/b|α=|a|α /|b|α при ab>0.

 

Формулы, обратные формулам 1-7, так же верны.

 

Пример 11.

Вычислите: .

Решение:

упростим заданное выражение, использую свойства степеней:

.

 

Пример 12.

Вычислите: .

Решение:

упростим заданное выражение, использую свойства степеней:

 

Задания для самостоятельного решения:

1 вариант

№1. Вычислите:

1) ;

2) .

№2. Упростите:

.

2 вариант

№1. Вычислите:

1) ;

2) .

№2. Упростите:

.

3 вариант

№1. Вычислите:

1) ;

2) .

№2. Упростите:

.

4 вариант

№1. Вычислите:

1) ;

2) .

№2. Упростите:

.

5 вариант

№1. Вычислите:

1) ;

2) .

№2. Упростите:

.

6 вариант

№1. Вычислите:

1) ;

2) .

№2. Упростите:

.

7 вариант

№1. Вычислите:

1) ;

2) .

№2. Упростите:

.

8 вариант

№1. Вычислите:

1) ;

2) .

№2. Упростите:

.

9 вариант

№1. Вычислите:

1) ;

2) .

№2. Упростите:

.

 

Контрольные вопросы:

1.     Перечислите основные показательные тождества.

2.     Перечислите свойства степеней с действительными показателями.

 

Практическое занятие № 4

Действия со степенями

 

Цель работы:

студент должен:

знать:

-       основные показательные тождества;

-       свойства степеней с действительными показателями;

уметь:

-       вычислять степени с действительными показателями.

 

Сведения из теории:

Свойства степеней с действительным показателем:

1. ax/y=a(xk)/(yk), a>0, y, kЄN, xЄZ.

2. ax>0, a>0, xЄR (любая степень положительного числа положительна).

3. ax>1 при a>1, x>0.

4. ax<1 при a>1, x<0.

5. 1x=1 (любая степень единицы равна единице).

6. ax<1 при 0<a<1, x>0.

7. ax>1 при 0<a<1, x<0.

8. Если a>1, a≠1, то для любого положительного числа b существует единственное действительное число х такое, что ах=b при b>0.

9. Любая положительная степень нуля равна нулю.

 

Так же при упрощении выражений, содержащих степени пользуются формулами: a0=1, a≠0; am/n=, mЄZ, nЄN, n≥2.

 

Пример 13.

Решить уравнение: .

Решение:

т.к. степень уравнения 5 – нечетное число, то уравнение имеет один корень: .

 

Пример 14.

Упростите: .

Решение:

используя свойства степеней, имеем:

.

 

Пример 15.

Вычислите: .

Решение:

используя свойства степеней, имеем:

 

Задания для самостоятельного решения:

1 вариант

1) Вычислите:

.

2) Решить уравнение:

.

3) Упростите:

.

2 вариант

1) Вычислите:

.

2) Решить уравнение:

.

3) Упростите:

.

3 вариант

1) Вычислите:

.

2) Решить уравнение:

.

3) Упростите:

.

4 вариант

1) Вычислите:

.

2) Решить уравнение:

.

3) Упростите:

.

5 вариант

1) Вычислите:

.

2) Решить уравнение:

.

3) Упростите:

.

6 вариант

1) Вычислите:

.

2) Решить уравнение:

.

3) Упростите:

.

7 вариант

1) Вычислите:

.

2) Решить уравнение:

.

3) Упростите:

.

8 вариант

1) Вычислите:

.

2) Решить уравнение:

.

3) Упростите:

.

9 вариант

1) Вычислите:

.

2) Решить уравнение:

.

3) Упростите:

.

 

Контрольные вопросы:

1.     Перечислите основные показательные тождества.

2.     Перечислите свойства степеней с действительными показателями.

 

Практическое занятие № 5

Десятичные и натуральные логарифмы

 

Цель работы:

студент должен:

знать:

-       определение логарифма;

-       свойства логарифмов;

уметь:

-       вычислять логарифмы по любому основанию.

 

Сведения из теории:

Логарифмом числа b по основанию а называется показатель степени (х), в которую нужно возвести основание а, чтобы получить число b, т.е. logab=x ax=b.

При работе с логарифмами применяются следующие их свойств, вытекающие из свойств показательной функции:

1. аlogab=b (где b>0, a>0 и a≠0) называют основным логарифмическим тождеством.

При любом a>0 (a≠0) и любых положительных х и у выполняются равенства:

2. loga1=0.

3. logaа=1.

4. Логарифм произведения равен сумме логарифмов: logaxу=logax+logaу.

5. Логарифм частного равен разности логарифмов: loga(x)=logax-logaу.

6. Логарифм степени равен произведению показателя степени на логарифм основания этой степени: logaxk=klogax.

 

Основные свойства логарифмов широко применяются в ходе преобразования выражений, содержащих логарифмы. Среди них формула перехода к новому основанию: logax=logbx/logba. Эта формула верна, если обе ее части имеют смысл, т.е. при x>0, a>0 и a≠0, b>0 и b≠1).

По правилу логарифмирования степени и основному логарифмическому тождеству получаем:

logbx=logb(аlogaх), откуда logbx=logax·logba. Эту формулу так же можно использовать для упрощения выражений.

С помощью формулы перехода можно найти значение логарифма с произвольным основанием а, имея таблицы логарифмов, составленные для какого-нибудь одного основания b. Наиболее употребительны таблицы десятичных и натуральных логарифмов (десятичными называют логарифмы по основанию 10 и обозначают lg, а натуральными логарифмами называют логарифмы по основанию е~2,72 и обозначают ln).

 

Пример16.

Вычислите log0,37.

Решение:

воспользуемся формулой перехода к новому основанию и перейдем к основанию 10:

logax=logbx/logba

log0,37=log107/log100,3=lg7/lg0,3.

Пользуясь калькулятором или специальными таблицами, например, таблицей В.М. Брадиса, находим значение lg7=0,8451.

Используя 5 и 3 свойства логарифмов, вычисляем

lg0,3=lg(3/10)=lg3-lg10=0,4771-1=-0,5229.

Итак, log0,37=0,8451/(-0,5229)=-1,6162.

 

Пример 17.

Вычислите: (lg72-lg9)/(lg28-lg7).

Решение:

используя 5 и 6 свойства логарифмов, вычисляем

lg72-lg9=lg(72/9)=lg8=lg23=3lg2;

lg28-lg7=lg(28/7)=lg4=lg22=2lg2.

Итак,

(lg72-lg9)/(lg28-lg7)=(3lg2)/(2lg2)=3/2=1,5.

Задания для самостоятельного решения:

1 вариант

1) Вычислите log0,25.

2) Дано: . Вычислите: .

2 вариант

1) Вычислите log3 0,1.

2) Вычислите: .

3 вариант

1) Вычислите log0,51.

2) Дано: . Вычислите: .

4 вариант

1) Вычислите log0,74.

2) Вычислите:

5 вариант

1) Вычислите log0,29.

2) Вычислите: .

6 вариант

1) Вычислите log0,310.

2) Вычислите:

7 вариант

1) Вычислите log0,370.

2) Вычислить:

8 вариант

1) Вычислите log0,320.

2) Вычислите: .

9 вариант

1) Вычислите log0,330.

2) Вычислите:

 

Контрольные вопросы:

1.     Дайте определение логарифма числа.

2.     Перечислите свойства логарифмов.

 

Практическое занятие № 6

Преобразование логарифмических выражений

 

Цель работы:

студент должен:

знать:

-       определение логарифма числа;

-       формулы основного логарифмического тождества, логарифма произведения, частного, степени, перехода от одной системы логарифмов к другой;

уметь:

-       вычислять значения несложных логарифмических выражений.

 

Сведения из теории:

Логарифмом числа b по основанию а называется показатель степени (х), в которую нужно возвести основание а, чтобы получить число b, т.е. logab=x ax=b.

При работе с логарифмами применяются следующие их свойств, вытекающие из свойств показательной функции:

1. аlogab=b (где b>0, a>0 и a≠0) называют основным логарифмическим тождеством.

При любом a>0 (a≠0) и любых положительных х и у выполняются равенства:

2. loga1=0.

3. logaа=1.

4. Логарифм произведения равен сумме логарифмов: logaxу=logax+logaу.

5. Логарифм частного равен разности логарифмов: loga(x)=logax-logaу.

6. Логарифм степени равен произведению показателя степени на логарифм основания этой степени: logaxk=klogax.

 

Основные свойства логарифмов широко применяются в ходе преобразования выражений, содержащих логарифмы. Среди них формула перехода к новому основанию: logax=logbx/logba. Эта формула верна, если обе ее части имеют смысл, т.е. при x>0, a>0 и a≠0, b>0 и b≠1).

По правилу логарифмирования степени и основному логарифмическому тождеству получаем:

logbx=logb(аlogaх), откуда logbx=logax·logba. Эту формулу так же можно использовать для упрощения выражений.

 

Пример 18.

Вычислите, используя определение логарифма числа .

Решение:

вычислим отдельно каждый логарифм:

Вернемся в пример:

 

Пример 19.

Вычислите, используя основное логарифмическое тождество: .

Решение:

используя свойство степени, разложим данное выражение на множители:

Используя 6 свойство логарифма степени, имеем:

Используя основное логарифмическое тождество, имеем:

 

Задания для самостоятельного решения:

Вычислите:

1 вариант

1) log160,5;

2) ;

3) .

2 вариант

1) log64(1/16);

2) ;

3) .

3 вариант

1) log487;

2) ;

3) .

4 вариант

1) log0.20,08;

2) ;

3) .

5 вариант

1) lg0,01;

2) ;

3) .

6 вариант

1) log50,04;

2) ;

3) .

7 вариант

1) ;

2) ;

3) .

8 вариант

1) ;

2) ;

3) .

9 вариант

1) ;

2) ;

3) .

 

Контрольные вопросы:

1.     Дайте определение логарифма числа.

2.     Перечислите свойства логарифмов.

 

Практическое занятие № 6

Преобразование показательных выражений

 

Цель работы:

студент должен:

знать:

-       свойства степеней;

-       способы решения показательных уравнений;

уметь:

-       решать уравнения, содержащие переменную в показателе степени.

 

Сведения из теории:

Уравнение, содержащее переменную в показателе, называется показательным.

При решении показательных уравнений вида af(x)=ak(x) (где a>0, a≠0) используется следующее свойство: (af(x)=ak(x))→(f(x)=k(x)).

Преобразование показательного уравнения к виду af(x)=ak(x) выполняется многими способами. Рассмотрим некоторые способы.

 

Пример 20.

Решите уравнение: .

Решение:

по определению нулевого показателя степени: 1=20, получим:

.

По свойству (af(x)=ak(x))→(f(x)=k(x)), получаем обычное квадратное уравнение, корни которого вычисляем через дискриминант:

,

х1=3, х2=4.

Ответ: 3, 4.

 

Пример 21.

Решите уравнение: .

Решение:

приведем обе части уравнения к основанию 2:

,

По свойству (af(x)=ak(x))→(f(x)=k(x)), получаем 6х=7 и х=7/6.

Ответ: 7/6.

 

Пример 22.

Решите уравнение: .

Решение:

разделив обе части уравнения на одно и то же число 5х-2, получим:

Ответ: 2.

 

Пример 23.

Решите уравнение: .

Решение:

вынесем общий множитель 2х за скобку, получим:

Ответ: 4.

 

Задания для самостоятельного решения:

Решите уравнения:

1 вариант

1) ;

2) .

2 вариант

1) 23х=5х;

3 вариант

1) 3х=7х/2;

2) .

4 вариант

1) 5х-3=23;

2) .

5 вариант

1) ;

2) .

6 вариант

1) 3х-5=81;

2) 0,01=10-х.

7 вариант

1) ;

2) .

8 вариант

1) ;

2)

9 вариант

1) ;

2) 1000=100х.

 

Контрольные вопросы:

1.   Что называется показательным уравнением?

2.   Запишите свойство, которое используют при решении показательных уравнений.

 

Практическое занятие № 7

Преобразование логарифмических выражений

 

Цель работы:

студент должен:

знать:

-       определение логарифма;

-       свойства логарифмов;

уметь:

-  решать уравнения, содержащие переменную под знаком логарифма.

 

Сведения из теории:

Уравнение, содержащее переменную под знаком логарифма или в основании логарифма, называется логарифмическим.

Рассмотрим простейшее логарифмическое уравнение: logax=b.

Логарифмическая функция возрастает (или убывает) на промежутке (0; +∞) и принимает на этом промежутке все действительные значения.

Теорема о корне: пусть функция f возрастает (убывает) на промежутке I, число а – любое из значений, принимаемых f на этом промежутке. Тогда уравнение f(x)=a имеет единственный корень в промежутке I.

По вышесказанной теореме следует, что для любого b данное уравнение имеет, и притом только одно, решение.

Из определения логарифма числа следует, что таким числом является ab.

 

Пример 24.

Решите уравнение: log2(х2+4х+3)=3.

Решение:

данному уравнению удовлетворяют те значения х, для которых выполнено равенство: х2+4х+3=23.

Получаем обычное квадратное уравнение х2+4х+3=8 или х2+4х-5=0, корни которого вычисляем с помощью дискриминанта: х1=1; х2=-5.

 

Пример 25.

Решите уравнение: log5(2х+3)=log5(х+1).

Решение:

данное уравнение определено для тех значений х, при которых выполнены неравенства: 2х+3>0 и х+1>0 (это следует из определения логарифма).

Для этих х данное уравнение равносильно уравнению: 2х+3=х+1, из которого находим х=-2.

Выполняя проверку, убеждаемся, что х=-2 не удовлетворяет неравенству х+1>0. Следовательно, данное уравнение корней не имеет.

 

Пример 26.

Решите уравнение: log25х-log5х2-3=0.

Решение:

данное уравнение, воспользовавшись свойством степени логарифма, можно переписать в виде: (log5х)2-2log5х-3=0.

Сделаем замену переменной: t=log5х, тогда наше уравнение перепишется в виде: t2-2t-3=0, корни которого вычислим через дискриминант: t1=3, t2=-1.

Вернемся к исходной переменной: log5х=3 или log5х=-1.

Используя определение логарифма получаем корни исходного уравнения: х1=53=125, х2=5-1=1/5=0,2.

 

Задания для самостоятельного решения:

Решите уравнение:

1 вариант

1) ;

2 вариант

1) ;

3 вариант

1) ;

2) .

4 вариант

1) ;

2) .

5 вариант

1) ;

2).

6 вариант

1) log2(2x-1)=4;

7 вариант

1) log3(x-12)=2;

8 вариант

1) logx16-logx2=1/2;

9 вариант

1) log3(x+8)=-2;

 

Контрольные вопросы:

1.     Что называется логарифмическим уравнением?

2.      Перечислите способы решения уравнений, содержащих переменную под знаком логарифма или в основании логарифма.

 

Практическое занятие № 8

Преобразование выражений

 

Цель работы:

студент должен:

знать:

-       правила преобразования рациональных, иррациональных, степенных выражений;

уметь:

-       выполнять преобразования рациональных, иррациональных, степенных выражений.

 

Сведения из теории:

Преобразование алгебраических выражений, используя приведение дробей к общему знаменателю, формулы сокращенного умножения.

Формулы сокращенного умножения:

;

;

;

;

где а, b, с – любые действительные числа;

,

где а¹0, х1 и х2 – корни уравнения .

 

Основное свойство дроби и действия над дробями

, где b¹0, с¹0;

;

;

;

.

 

Пример 27.

Упростите:

Решение:

решаем по действиям: 1) деление; 2) сложение; 3) вычитание.

1) Используя формулы сокращенного умножения разности квадратов: , суммы кубов , получим:

;

2) Для сложения приведем дроби к общему знаменателю :

3) Выполним вычитание дробей с одинаковыми знаменателями:

 

Преобразование выражений, содержащих радикалы

Чтобы освободиться от иррациональности в знаменателе необходимо и числитель и знаменатель дроби помножить на одно и то же число, сопряженное к знаменателю.

 

Пример 28.

Освободитесь от иррациональности в знаменателе: .

Решение:

чтобы освободиться от иррациональности в знаменателе необходимо и числитель и знаменатель дроби помножить на одно и то же число, сопряженное к знаменателю: , тогда получим:

 

Решение иррациональных уравнений

Наиболее часто используемым при решении иррациональных уравнений способов является возведение обеих частей уравнения в квадрат.

 

Пример 29.

Решите уравнение: .

Решение:

возведем обе части уравнения в квадрат, при этом в уравнении появятся посторонние корни, поэтому проверка при решении иррациональных уравнений обязательна:

Получилось обычное квадратное уравнение, корни которого вычисляем через дискриминант: х1=12, х2=7.

Выполним проверку, для этого подставим в наше исходное уравнение получившиеся корни:

х1=12:

х2=7:

Ответ: 12.

 

Задания для самостоятельного решения:

1 вариант

№1. Упростите:

№2. Освободитесь от иррациональности в знаменателе:

.

№3. Решите иррациональное уравнение: .

2 вариант

№1. Упростите:

№2. Освободитесь от иррациональности в знаменателе: .

№3. Решите иррациональное уравнение: .

3 вариант

№1. Упростите:

№2. Освободитесь от иррациональности в знаменателе:

.

№3. Решите иррациональное уравнение: .

4 вариант

№1. Упростите:

№2. Освободитесь от иррациональности в знаменателе: .

№3. Решите иррациональное уравнение: .

5 вариант

№1. Упростите:

№2. Освободитесь от иррациональности в знаменателе:

.

№3. Решите иррациональное уравнение: .

6 вариант

№1. Упростите:

№2. Освободитесь от иррациональности в знаменателе: .

№3. Решите иррациональное уравнение: .

7 вариант

№1. Упростите:

№2. Освободитесь от иррациональности в знаменателе:

.

№3. Решите иррациональное уравнение: .

8 вариант

№1. Упростите:

№2. Освободитесь от иррациональности в знаменателе: .

№3. Решите иррациональное уравнение: .

9 вариант

№1. Упростите:

№2. Освободитесь от иррациональности в знаменателе: .

№3. Решите иррациональное уравнение: .

 

Контрольные вопросы:

1.      Какие формулы можно использовать при преобразовании алгебраических выражений?

2.     Как можно освободиться от иррациональности в знаменателе?

3.     Сформулируйте правила решения иррациональных уравнений.

 

Практическое занятие № 9

Параллельность прямой и плоскости

 

Цель работы:

студент должен:

знать:

- признаки параллельности прямой и плоскости;

- признаки параллельности плоскостей;

- признаки параллельности прямых в пространстве;

уметь:

-       строить параллельные прямые, плоскости в пространстве.

 

Сведения из теории:

Признаки параллельности прямой и плоскости

1) Если прямая, лежащая вне плоскости, параллельна какой-либо прямой, лежащей в этой плоскости, то она параллельна этой плоскости.

2) Если прямая и плоскость перпендикулярны одной и той же прямой, то они параллельны.

 

Признаки параллельности плоскостей

1) Если две пересекающиеся прямые одной плоскости cоответственно параллельны двум пересекающимся прямым другой плоскости, то эти плоскости параллельны.

2) Если две плоскости перпендикулярны одной и той же прямой, то они параллельны.

 

Признаки параллельности прямых в пространстве

1) Если две прямые перпендикулярны одной и той же плоскости, то они параллельны.

2) Если в одной из пересекающихся плоскостей лежит прямая, параллельная другой плоскости, то она параллельна линии пересечения плоскостей.

 

Параллельные прямые

Возьмём, например, две такие прямые АВ и , из которых одна пересекает некоторую плоскость Р, а другая лежит на ней, но не проходит через точку (С) пересечения первой прямой и плоскости Р.

http://oldskola1.narod.ru/Kiselev70/004.gif

Рисунок 33. Непересекающиеся прямые

 

Через такие две прямые нельзя провести плоскость, потому что в противном случае через прямую и точку С проходили бы две различные плоскости: одна Р, пересекающая прямую АВ, и другая, содержащая её, а это невозможно.

Две прямые, не лежащие в одной плоскости, конечно, не пересекаются, сколько бы их ни продолжали; однако их не называют параллельными.

Две прямые, не лежащие в одной плоскости, называются скрещивающимися.

 

Прямая и плоскость параллельные между собой

Плоскость и прямая, не лежащая в этой плоскости, называются параллельными, если они не пересекаются, сколько бы их ни продолжали.

 

Если прямая (АВ) параллельна какой-нибудь прямой (СD), расположенной в плоскости (Р), то она параллельна самой плоскости.

Если плоскость (R) проходит через прямую (АВ), параллельную другой плоскости (Р), и пересекает эту плоскость, то линия пересечения (СD) параллельна первой прямой (АВ).

http://oldskola1.narod.ru/Kiselev70/005.gif

Рисунок 34. Прямая и плоскость параллельные между собой

 

Если прямая (АВ) параллельна каждой из двух пересекающихся плоскостей (Р и Q), то она параллельна линии их пересечения (СD).

http://oldskola1.narod.ru/Kiselev70/006.gif

Рисунок 35. Параллельность прямой линии пересечения плоскостей

 

Если две прямые (АВ и СD) параллельны третьей прямой (ЕF), то они параллельны между собой.

http://oldskola1.narod.ru/Kiselev70/007.gif

Рисунок 36. Параллельность трех прямых

 

Параллельные плоскости

Две плоскости называются параллельными, если они не пересекаются, сколько бы их ни продолжали.

Если две пересекающиеся прямые (АВ и АС) одной плоскости (Р) соответственно параллельны двум прямым (А1В1 и А1С1) другой плоскости(Q), то эти плоскости параллельны. Прямые АВ и АС параллельны плоскости Q.

http://oldskola1.narod.ru/Kiselev70/008.gif

Рисунок 37. Параллельные плоскости

 

Задания для самостоятельного решения:

Решите следующие задачи (выполнить чертеж, дать подробные пояснения):

1) Сторона АС треугольника АВС параллельна плоскости a, а стороны АВ и ВС пересекаются с этой плоскостью в точках М и N. Докажите, что треугольники АВС и МВN подобны.

2) Сколько существует плоскостей, проходящих через данные прямую и точку в пространстве?

3) В пространстве даны прямая a и точка M. Сколько существует прямых, проходящих через M и параллельных прямой a?

4) Даны плоскость и точка M вне плоскости. Сколько существует прямых, проходящих через M и параллельных плоскости?

5) В пространстве даны две параллельные прямые a и b. Сколько существует плоскостей, проходящих через прямую a и параллельных прямой b?

6) Даны две скрещивающиеся прямые a и b. Сколько существует пар параллельных плоскостей, одна из которых проходит через a, а другая – через b?

7) В пространстве даны две пересекающиеся прямые a, b и не лежащая на них точка M. Сколько существует плоскостей, проходящих через M и параллельных прямым a и b?

 

Контрольные вопросы:

1.        Сформулируйте признаки параллельности прямой и плоскости.

2.        Сформулируйте признаки параллельности плоскостей.

3.        Сформулируйте признаки параллельности прямых в пространстве.

 

Практическое занятие №10

Геометрические преобразования пространства: параллельный перенос, симметрия относительно плоскости

 

Цель работы:

студент должен:

знать:

- определение параллельного переноса и его свойства;

- формулы для параллельного переноса;

уметь:

-       выполнять геометрические преобразования пространства: параллельный перенос, симметрия относительно плоскости.

 

Сведения из теории:

Параллельный перенос и его свойства

Наглядно параллельный перенос определяется как преобразование, при котором точки смещаются в одном и том же направлении на одно и то же расстояние. Такое определение не является математически строгим, потому что в нем употребляется выражение «в одном и том же направлении», которое само нуждается в точном определении. В связи с этим параллельному переносу мы дадим другое, отвечающее тому же наглядному представлению, но уже строгое определение.

Введем на плоскости декартовы координаты х, у. Преобразование фигуры F, при котором произвольная ее точка (х; у) переходит в точку (х+а; у+b), где а и b одни и те же для всех точек (х; у), называется параллельным переносом. Параллельный перенос задается формулами x'=x+а, у=у+b.

Эти формулы выражают координаты х', у' точки, в которую переходит точка (х; у) при параллельном переносе.

Параллельный перенос и его свойства

Рисунок 38. Параллельный перенос

 

Параллельный перенос есть движение

Действительно, две произвольные точки А(х1; у1) к В(х2; у2) переходят при параллельном переносе в точки А'(х1+а; у1+b), В'(х2+а; y2+b).

Поэтому

АВ2=(х2-х1)2+(у2-у1)2,

A'В'2=(х2-х1)2+(у2-у1)2.

Отсюда АВ=А'В'. Т. о., параллельный перенос сохраняет расстояния, а значит, является движением, что и требовалось доказать.

Название «параллельный перенос» оправдывается тем, что при параллельном переносе точки смещаются по параллельным (или совпадающим) прямым на одно и то же расстояние.

Параллельный перенос и его свойства

Рисунок 39. Параллельный перенос

 

Симметрия относительно плоскости

Симметрия относительно плоскости – это такое свойство геометрической фигуры, когда любой точке, расположенной по одну сторону плоскости, всегда будет соответствовать точка, расположенная по другую сторону плоскости, а отрезки, соединяющие эти точки, будут перпендикулярны плоскости симметрии и делятся ею пополам.

Следует отметить, что две симметричные фигуры или две симметричные части одной фигуры при всем их сходстве, равенстве объемов и площадей поверхностей, в общем случае, неравны, т.е. их нельзя совместить друг с другом. Это разные фигуры, их нельзя заменить друг другом, например, правая перчатка, ботинок и т.д. не годятся для левой руки, ноги. Предметы могут иметь одну, две, три и т.д. плоскостей симметрии.

Например, прямая пирамида (рис. 40а, 41а), основанием которой является равнобедренный треугольник, симметрична относительно одной плоскости Р. Призма с таким же основанием (рис. 40б, 41б) имеет две плоскости симметрии. У правильной шестиугольной призмы (рис. 40в, 41в) их семь. Тела вращения: шар, тор, цилиндр, конус и т.д. имеют бесконечное количество плоскостей симметрии.

Рисунок 40. Плоскости симметрии

 

Рисунок 41. Изображение плоскостей симметрии

 

На чертежах плоскости симметрии изображаются тонкими штрихпунктирными линиями, являющимися как бы следами этих плоскостей. Если такой след совпадает с другой линией чертежа, например, с контурной, то она проводится в виде тонких штрихов, выводимых за контур изображения на 5 – 8 мм. На чертеже наносятся следы только тех плоскостей симметрии, которые перпендикулярны плоскости проекций данного изображения.

При наличии нескольких подобно расположенных плоскостей симметрии, как у призмы (рис. 40в), на чертеже изображается только одна взаимно перпендикулярная пара следов, по возможности тех, которые параллельны плоскостям проекций.

Для геометрических тел с плоскостями симметрии, параллельными их основаниям, например для призм, следы плоскостей симметрии на чертежах показывать не принято.

 

Задания для самостоятельного решения:

Решите задачи:

1) Докажите, что при движении параллельные прямые отображаются на параллельные прямые.

2) Докажите, что при движении: а) параллелограмм отображается на параллелограмм; б) трапеция отображается на трапецию; в) ромб отображается на ромб; г) прямоугольник отображается на прямоугольник, а квадрат – на квадрат.

3) На сторонах AB и CD параллелограмма ABCD построены квадраты (плоскости квадратов перпендикулярны плоскости параллелограмма). Используя параллельный перенос, докажите, что отрезок, соединяющий центры этих квадратов, равен и параллелен стороне AD.

 

Контрольные вопросы:

1.        Дайте определение параллельного переноса и перечислите его свойства.

2.        Запишите формулы для параллельного переноса.

 

Практическое занятие № 11

Параллельное проектирование

 

Цель работы:

студент должен:

знать:

- свойства параллельного проектирования;

уметь:

-       строить фигуры с помощью параллельного проектирования.

 

Сведения из теории:

Параллельное проектирование

Пусть даны плоскость α и прямая l, пересекающая плоскость а. Возьмем произвольную точку пространства A1 и проведем через эту точку прямую l1, параллельную l. Прямая l1 пересечет плоскость α в некоторой точке A. Полученная таким образом точка A называется проекцией точки A на плоскость α при проектировании параллельно прямой l. Обычно кратко говорят, что точка A есть параллельная проекция точки A1.

http://www.fizmatik.ru/images/Geometry/5_1.JPG

Рисунок 42. Параллельное проектирование

 

Параллельной проекцией пространственной фигуры Φ1 называется множество Φ параллельных проекций всех точек данной фигуры.

Свойства параллельного проектирования

1) Проекция прямой есть прямая.

2) Проекции параллельных прямых параллельны.

3) Отношение проекций двух параллельных отрезков равно отношению проектируемых отрезков.

 

Ортогональное проектирование

Частным случаем параллельного проектирования является ортогональное проектирование

Пусть даны плоскость α и прямая l, перпендикулярная α. Возьмем произвольную точку пространства A1 и проведем через нее прямую l1 параллельную l (и, следовательно, перпендикулярную плоскости α). Прямая l1 пересечет плоскость α в некоторой точке A.

http://www.fizmatik.ru/images/Geometry/5_2.JPG

Рисунок 43. Ортогональное проектирование

 

Полученная точка A называется ортогональной проекцией точки A1 на плоскость α.

 

Ортогональной проекцией фигуры Φ1 на плоскость α называется множество Φ ортогональных проекций всех точек данной фигуры Φ1. Как частный случай параллельного проектирования, ортогональное проектирование обладает всеми свойствами параллельного проектирования.

Свойство ортогональной проекции плоского многоугольника

Площадь s ортогональной проекции плоского многоугольника на плоскость α равна площади S проектируемого многоугольника, умноженной на косинус угла γ между плоскостью многоугольника и плоскостью α:

s=S·cos(γ).

 

Пример 112.

Через сторону основания правильной треугольной призмы проведена плоскость под углом γ=300 к плоскости ее основания. Найти площадь образующегося сечения, если сторона основания равна 6 см.

Решение:

т.к. призма правильная, то ее боковые ребра перпендикулярны плоскости основания. Следовательно, плоскость основания есть проекция плоскости сечения.

Т.к. в основании правильный треугольник, то его площадь равна:

.

Используя свойство ортогональной проекции, имеем:

.

Зная, что сторона основания равна 6 см и угол γ=300, вычислим площадь:

.

 

Задания для самостоятельного решения:

Решите задачи:

1) Каковы проекции двух прямых на плоскость, если: а) прямые пересекаются; б) прямые скрещиваются; в) прямые параллельны.

2) На модели куба ABCDA1B1C1D1 укажите проекции на плоскость грани АА1В1В отрезков C1D1, AD, C1D и DB1, треугольников C1CD и ACD, квадрата BВ1С1C.

3) Гипотенуза прямоугольного треугольника равна 10 см, а острый угол 600. Найдите площадь проекции этого треугольника на плоскость, составляющую с плоскостью треугольника угол 300.

4) Стороны треугольника равны 3,9 см, 4,1 см и 2,8 см. Найдите площадь его проекции на плоскость, составляющую с плоскостью треугольника угол 600.

 

Контрольные вопросы:

1.        Что называется параллельной проекцией?

2.        Перечислите свойства параллельного проектирования.

3.        Что называется ортогональной проекцией фигуры?

 

Практическое занятие №12

Изображения пространственных фигур

 

Цель работы:

студент должен:

знать:

- свойства параллельного проектирования;

уметь:

-       изображать пространственные фигуры на плоскости с помощью параллельного проектирования.

 

Сведения из теории:

Изображение пространственных фигур на плоскости

Для изображения пространственных фигур на плоскости обычно пользуются параллельным проектированием.

Изображения фигуры на плоскости

Рисунок 44. Изображение пространственных фигур на плоскости

 

Прямолинейные отрезки фигуры изображаются на плоскости чертежа отрезками.

Изображения фигуры на плоскости

Рисунок 45. Изображение отрезка на плоскости

 

Действительно, все прямые, проектирующие точки отрезка АС, лежат в одной плоскости, пересекающей плоскость а по прямой А1С1. Произвольная точка В отрезка АС изображается точкой В1 отрезка А1С1.

Отношение отрезков одной прямой или параллельных прямых сохраняется при параллельном проектировании:

.

 

Пример 113.

Дана параллельная проекция треугольника. Как построить проекции медиан этого треугольника?

Решение:

при параллельном проектировании сохраняется отношение отрезков прямой. Поэтому середина стороны треугольника проектируется в середину проекции этой стороны. Следовательно, проекции медиан треугольника будут медианами его проекции.

 

Задания для самостоятельного решения:

Решите задачи:

1) Построить изображение правильного треугольника ABC , изображение высоты BH и биссектрисы АK.

2) Трапеция ABCD – параллельная проекция равнобедренной трапеции. Построить ось симметрии и высоту данной трапеции.

3) Начертите параллельную проекцию ромба АBCD, имеющего угол A=600. Постройте изображение высоты этого ромба, проведенной из вершины острого угла.

 

Контрольные вопросы:

1.        Что называется параллельной проекцией?

2.        Перечислите свойства параллельного проектирования.

3.        Что является параллельной проекцией отрезка, треугольника, прямоугольника, квадрата, окружности?

4.         Какие величины не изменяются при параллельном проецировании? (длина отрезка, градусная мера углов, отношения длин отрезков, отношение площадей двух фигур)?

5.        Может ли при параллельном проецировании параллелограмма получиться трапеция и наоборот?

 

Практическое занятие №13

Правила сложения векторов

 

Цель работы:

студент должен:

знать:

- правила сложения векторов;

уметь:

-       строить сумму векторов по правилу треугольника, параллелограмма;

-       вычислять координаты суммы векторов.

 

Сведения из теории:

Линейные операции над векторами

Суммой двух векторов  называется вектор, который идет из начала вектора  в конец вектора  при условии, что вектор  приложен к концу вектора  (правило треугольника).

http://a-geometry.narod.ru/theory/img_30/fig_30_01.gif

Рисунок 81. Правило треугольника

 

Наряду с правилом треугольника часто пользуются (равносильным ему) правилом параллелограмма: если векторы  и  приведены к общему началу и на них построен параллелограмм, то сумма  есть вектор, совпадающий с диагональю этого параллелограмма, идущей из общего начала  и . Отсюда сразу следует, что .

http://a-geometry.narod.ru/theory/img_30/fig_30_02.gif

Рисунок 82. Правило параллелограмма

 

Сложение многих векторов производится при помощи последовательного применения правила треугольника, построим сумму четырех векторов , , , .

http://a-geometry.narod.ru/theory/img_30/fig_30_03.gif

Рисунок 83. Правило многоугольника

 

Разность двух векторов  называется вектор, который в сумме с вектором  составляет вектор . Если два вектора  и  приведены к общему началу, то разность их есть вектор, идущий из конца  («вычитаемого») к концу  («уменьшаемого»).

Два вектора равной длины, лежащие на одной прямой и направленные в противоположные стороны, называются взаимно обратными: если один из них обозначен символом , то другой обозначается символом -. Легко видеть, что . Т. о., построение разности равносильно прибавлению к «уменьшаемому» вектора, обратного «вычитаемого».

Три вектора в пространстве можно складывать по правилу параллелепипеда: если на трех векторах , , , как на ребрах, построить параллелепипед, то его диагональ, выходящая из общего начала данных векторов, и будет их суммой =++:

 

 

 

 

 

 

Рисунок 84. Правило параллелепипеда

 

Задания для самостоятельного решения:

1) По данным векторам  и  построить каждый из следующих векторов: 1) , 2) , 3) , 4) .

 

При сложении векторов складываются их соответствующие координаты, при вычитании вычитаются соответствующие координаты, т.е. если даны координаты векторов  и , =(х1, у1, z1), =(х2, у2, z2) и =+; =-, то координаты векторов и  вычисляются по формулам:

=(х1+х2; у1+у2; z1+z2),

=(x1-x2; y1-y2; z1-z2).

 

Пример 142.

Вычислить координаты векторов =+; =-, если =(-3; 5; 1), =(4; -2; 8).

Решение:

по формулам

=(х1+х2; у1+у2; z1+z2),

=(x1-x2; y1-y2; z1-z2),

имеем

=(-3+4; 5+(-2); 1+8)=(1; 3; 9),

=(-3-4; 5-(-2); 1-8)=(-7; 7; -7).

 

Задания для самостоятельного решения:

Вычислить координаты векторов =; =, если =(4; -3; 10), =(-4; 12; -1), =(3; -7; -11).

 

Контрольные вопросы:

1.        Сформулируйте правило треугольника для сложения векторов.

2.        Сформулируйте правило параллелограмма для сложения векторов.

3.        Запишите формулы сложения (разности) векторов в координатах.

 

Практическое занятие №13

Умножение вектора на число

 

Цель работы:

студент должен:

знать:

- правило умножения векторов;

уметь:

-       строить произведение вектора на число;

-       вычислять координаты вектора .

 

Сведения из теории:

Произведение  вектора  на число  называется вектор, модуль которого равен произведению модуля вектора  на модуль числа ; он параллелен вектору  или лежит с ним на одной прямой и направлен так же, как вектор , если  – число положительное, и противоположно вектору , если  – число отрицательное.

Если =0, для любого вектора  произведение  равно нуль-вектору: 0 ×=.

Если =1, то 1×=.

Если = -1, то (-1)× =- – вектор, противоположный вектору .

 

Пример 143.

Даны векторы, совпадающие со сторонами треугольника АВС: , . Выразить через векторы  и  вектор , где О – точка пересечения медиан треугольника. Выполните рисунок.

Решение:

известно, что точка О пересечения медиан треугольника делит отрезок медианы в отношении 2:1, считая от вершины. Поэтому , где точка D – середина стороны СВ.

Но вектор ; .

В треугольнике САD вектор .

Искомый вектор .

 

Задания для самостоятельного решения:

1) По данным векторам  и  построить каждый из следующих векторов: , , , .

2) В треугольнике АВС вектор  и вектор . Построить каждый из следующих векторов: , , , .

3) Точка О является точкой пересечения медиан треугольника АВС. Доказать, что .

4) В правильном пятиугольнике ABCDE заданы векторы, совпадающие с его ребрами: , , , , . Построить векторы: , , .

5) В параллелепипеде  заданы векторы, совпадающие с его ребрами: , ,  (рис. 85)

http://a-geometry.narod.ru/problems/img_30/fig_01.gif

Рисунок 85.

 

6) Построить каждый из следующих векторов: , , , .

 

Контрольные вопросы:

1.        Сформулируйте правило умножения вектора на число.

 

Практическое занятие №14

Скалярное произведение векторов

 

Цель работы:

студент должен:

знать:

- формулы для вычисления скалярного произведения векторов;

уметь:

-       вычислять скалярное произведение векторов, угол между векторами.

 

Сведения из теории:

Скалярным произведением двух векторов называется число, равное произведению модулей этих векторов на косинус угла между ними.

Скалярное произведение векторов  и  обозначается символом  (порядок записи сомножителей безразличен, то есть ).

Если угол между векторами  и  обозначить через φ, то их скалярное произведение можно выразить формулой:

.

Скалярное произведение векторов  и  можно выразить также формулой:

или

.

Из формулы  следует, что , если φ – острый угол, , если φ – тупой угол;  в том и только в том случае, когда векторы  и  перпендикулярны.

Скалярное произведение  называется скалярным квадратом вектора и обозначается символом . Скалярный квадрат вектора равен квадрату его модуля:

.

Если векторы  и  заданы своими координатами:  и , то их скалярное произведение может быть вычислено по формуле:

.

Отсюда следует необходимое и достаточное условие перпендикулярности двух векторов:

.

Угол φ между векторами  и  задается формулой , или в координатах .

Проекция произвольного вектора S=(x, y, z) на какую-нибудь ось u определяется формулой:

,

где  – единичный вектор, направленный по оси u.

Если даны α, β, γ, которые оси u составляют соответствующие углы с координатными осями, то  и для вычисления вектора  может служить формула:

.

 

Пример 144.

Векторы  и  образуют угол , зная, что , , вычислить: , , , , , , .

Решение:

из формулы , выразим , тогда ;

т.к. , то , ;

по формуле сокращенного умножения квадрата суммы, имеем

;

аналогично

;

по формуле сокращенного умножения квадрата разности, имеем

;

раскроем скобки

 

Задачи для самостоятельного решения:

1) Векторы  и  взаимно перпендикулярны; вектор  образует с ними углы, равные ; зная, что , , , вычислить: , , .

2) Векторы ,  и  попарно образуют друг с другом углы, каждый из которых равен 600. Зная, что , , , определить модуль вектора .

3) Даны векторы  и . Вычислить: , , , , , .

4) Даны точки A(-1; 3; -7), B(2; -1; 5), C(0; 1; -5). Вычислить: , , .

 

Контрольные вопросы:

1.        Запишите формулы для вычисления скалярного произведения векторов.

2.        Запишите формулу для вычисления угла между векторами.

 

Практическое занятие №15

Векторное задание прямых и плоскостей в пространстве

 

Цель работы:

студент должен:

знать:

- определение направляющего вектора прямой;

- канонические уравнения прямой;

- параметрические уравнения прямой;

уметь:

-       составлять уравнение прямой по двум точкам, по направляющему вектору.

 

Сведения из теории:

Направляющий вектор прямой. Канонические уравнения прямой. Параметрические уравнения прямой

Каждый не равный нулю вектор, лежащий на данной прямой или параллельный ей, называется направляющим вектором этой прямой.

Направляющий вектор произвольной прямой в дальнейшем обозначается буквой , его координаты – буквами l, m, n: .

Если известна одна точка М(x0, y0, z0) прямой и направляющий вектор , то прямая может быть определена уравнением вида:

.

В таком виде уравнение прямой называется каноническим.

Каноническое уравнение прямой, проходящей через данные точки М1(x1, y1, z1) и М2(x2, y2, z2) имеет вид:

.

Обозначим буквой t каждое из равных отношений в канонических уравнениях:

,

отсюда

 – параметрическое уравнение прямой, проходящей через точку М(x0, y0, z0) в направлении вектора .

 

Пример 145.

Составить канонические уравнения прямой, проходящей через данные точки: (1; -2; 1), (3; 1; -1).

Решение:

воспользуемся формулой , тогда получим

 

Задания для самостоятельного решения:

1) Составить канонические уравнения прямой, проходящей через точку М1(2; 0; -3) параллельно: вектору , прямой , оси Ох, оси Оу, оси Oz.

2) Составить канонические уравнения прямой, проходящей через данные точки: а) (3; -1; 0), (1; 0; -3); б) (0; -2; 3), (3; -2; 1); в) (1; 2; -4), (-1; 2; -4).

3) Составить параметрические уравнения прямой, проходящей через точку М1(1; -1; -3) параллельно: вектору ; прямой , прямой

4) Через точки М1(-6; 6; -5), М2(12; -6; 1) проведена прямая. Определить точки пересечения этой прямой с координатными плоскостями.

5) Даны вершины треугольника А(1; -2; -4), В(3; 1; -3), С(5; 1; -7). Составить параметрические уравнения его медианы, проведенной из вершины В на противоположную сторону.

 

Контрольные вопросы:

1.        Запишите в общем виде каноническое уравнение прямой.

2.        Запишите в общем виде параметрическое уравнение прямой.

 

Практическое занятие №16

Проекция вектора на ось

 

Цель работы:

студент должен:

знать:

- что называется числовой характеристикой проекции вектора на ось;

уметь:

-       вычислять числовую проекцию вектора на ось.

 

Сведения из теории:

Числовая проекция вектора на ось – это число, которое равно произведению длины данного вектора на косинус угла между этим вектором и вектором, определяющим направление оси.

Числовую проекцию вектора  на ось L обозначают как , а числовую проекцию вектора  на ось, определяемую вектором  – .

В этих обозначениях определение числовой проекции вектора  на прямую, направленную как вектор , примет вид .

Эта формула применяется, когда известны длина вектора  и угол между векторами  и .

 

Пример 146.

Вычислите числовую проекцию вектора  на прямую, направленную как вектор , если длина вектора  равна 8, а угол между векторами  и  равен 600.

Решение:

по формуле , имеем

Известно, что . Тогда формула , позволяющая найти числовую проекцию вектора на прямую, направленную как вектор , примет вид .

Т. о. числовая проекция вектора на ось, направление которой совпадает с направлением вектора , – это отношение скалярного произведения векторов и к длине вектора .

Полученную формулу вида  удобно применять для нахождения числовой проекции вектора на прямую, направление которой совпадает с направлением вектора , когда известны координаты векторов и .

 

Пример 147.

Известно, что вектор  задает направление оси L. Найдите числовую проекцию вектора  на ось L.

Решение:

запишем формулу  в координатной форме, тогда . Используем ее для нахождения требуемой числовой проекции вектора на ось L:

.

 

Пример 148.

Относительно прямоугольной системы координат Oxyz в трехмерном пространстве заданы два вектора  и . Найдите числовую проекцию вектора на ось L, направление которой совпадает с направлением вектора .

Решение:

по координатам векторов и вычислим скалярное произведение этих векторов: .

Длина вектора по его координатам вычисляется по следующей формуле . Тогда формула для определения числовой проекции вектора  на ось L в координатах имеет вид

Подставим в полученную формулу заданные координаты:

 

Для проекции выполняются следующие теоремы:

1. Проекции равных векторов на одну и ту же ось равны между собой:

=, тогда =.

2. Проекция суммы двух векторов на произвольну4ю ось равна сумме проекций этих векторов:

.

3. Проекция произведения скаляра на вектор на произвольную ось равна произведению этого скаляра на проекцию вектора:

.

 

Задания для самостоятельного решения:

1) Докажите, что для любых точек A, B, C, D справедливо равенство: .

2) Дано: , . Вычислите: ; , , .

3) Вектор  образует с осью Ох угол α и имеет длину . Определите координаты вектора  если:

а) α=900, ; б) α=1800, ; в) α=-900, ; г) α=450, .

 

Контрольные вопросы:

1.        Что называется числовой характеристикой проекции вектора на ось?

 

Практическое занятие №17

Расстояние между двумя точками. Вычисление координат середины отрезка

 

Цель работы:

студент должен:

знать:

- формулы для вычисления расстояния между двумя точками;

- формулы для вычисления координат середины отрезка;

уметь:

-       вычислять расстояние между двумя точками, координаты середины отрезка.

 

Сведения из теории:

Длиной отрезка АВ называется расстояние между точками А и В при заданном масштабе (отрезке единичной длины). Длину отрезка АВ будем обозначать как .

Расстояние между двумя точками A1(x1; y1) и A2(x2; y2) в прямоугольной системе координат выражается формулой:

Точка С называется серединой отрезка АВ, если она лежит на отрезке АВ и находится на одинаковом расстоянии от его концов, т. е.

 

Координаты середины отрезка на плоскости

Введем прямоугольную декартову систему координат Оxy на плоскости. Пусть нам даны две точки А(хА; уА) и В(хВ; уВ) и известно, что точка С – середина отрезка АВ. Найдем координаты хС и уС точки С.

Рассмотрим случай, когда точки А и В не совпадают и не лежат одновременно на одной из координатных осей или на прямой, перпендикулярной одной из координатных осей.

изображение

Рисунок 78. Координаты середины отрезка

 

По построению:

.

Т. о., середина отрезка АВ на плоскости с концами в точках А(хА; уА) и В(хВ; уВ) имеет координаты .

 

Пример 134.

На плоскости заданы координаты двух точек А(-7; 3), В(2; 4). Найдите координаты середины отрезка АВ.

Решение:

пусть точка С – середина отрезка АВ. Ее координаты равны полусуммам соответствующих координат точек А и В:

Т. о., середина отрезка АВ имеет координаты .

 

Часто с нахождением координат середины отрезка связаны задачи, в которых фигурирует термин «медиана».

 

Пример 135.

Найдите длину медианы АМ в треугольнике АВС, если известны координаты его вершин А(-1; 0), В(3; 2), С(9; -8).

Решение:

т. к. АМ – медиана, то точка М является серединой стороны ВС. Найдем координаты середины этого отрезка по известным координатам его концов:

Т. о., М(6; -3).

Осталось воспользоваться формулой для вычисления расстояния между точками А и М:

 

Существуют различные задачи, в которых известны координаты середины отрезка и одного из его концов, а требуется найти координаты другого конца. Рассмотрим решение одной из них.

 

Пример 136.

В прямоугольной системе координат трехмерного пространства дан параллелепипед ABCDA1B1C1D1. Известно, что C1(1; 1; 0), а М(4; 2; -4) – середина диагонали BD1. Найдите координаты точки А.

Решение:

диагонали параллелограмма пересекаются в одной точке, и эта точка является серединой каждой из этих диагоналей. Таким образом, мы можем утверждать, что точка М является серединой отрезка AC1 . Из формул для нахождения координат середины отрезка имеем:

Итак, точка А имеет координаты (7; 3; -8).

 

Задания для самостоятельного решения:

1) Вычислите периметр треугольника АВС, если А(4; 0), В(12; -2), С(5; -9).

2) Вычислите длину медианы АМ треугольника АВС, вершины которого имеют координаты А(0; 1), В(1; -4), С(5; 2).

3) Докажите, что треугольник АВС – равнобедренный и вычислите его площадь, если вершины которого имеют координаты А(-4; 1), В(-2; 4), С(0; 1).

4) Докажите, что четырехугольник АBCD является параллелограммом, и вычислите его диагонали, если А(1; 1), B(6; 1), C(7; 4), D(2; 4).

5) Докажите, что четырехугольник АBCD является прямоугольником, и вычислите его площадь, если А(-3; -1), B(1; -1), C(1; -3), D(-3; -3).

 

Контрольные вопросы:

1.   Запишите формулу для вычисления координат середины отрезка.

2.   Запишите формулу для вычисления расстояния между двумя точками.

 

Практическое занятие №18

Решение задач координатным методом

 

Цель работы:

студент должен:

знать:

- формулы для вычисления расстояния между двумя точками;

- формулы для вычисления координат середины отрезка;

уметь:

-       использовать формулы расстояния между двумя точками и формулу для вычисления координат середины отрезка при решении задач координатным методом.

 

Сведения из теории:

Вычисление координат точки, равноудаленной от заданных точек рассмотрим на примере 137

 

Пример 137.

Найти координаты точки О1, которая равноудалена от трех точек А(7; -1) и В(-2; 2) и С(-1; -5).

Решение:

из формулировки условия задачи следует, что О1А=О1В=О1С.

Пусть искомая точка О1 имеет координаты (а; b). По формуле:

,

найдем:

,

,

.

Составим систему из двух уравнений:

.

После возведения в квадрат левой и правой частей уравнений запишем:

.

Упростив, запишем:

.

Решив систему, получим: а=2; b=-1.

Точка О1(2; -1) равноудалена от трех заданных в условии точек, которые не лежат на одной прямой. Эта точка – есть центр окружности, проходящей через три заданные точки.

 

Вычисление абсциссы (ординаты) точки, которая лежит на оси абсцисс (ординат) и находится на заданном расстоянии от данной точки, рассмотрим на примере 138

 

Пример 138.

Расстояние от точки В(-5; 6) до точки А, лежащей на оси Ох равно 10. Найти координаты точки А.

Решение:

из формулировки условия задачи следует, что ордината точки А равна нулю и АВ = 10.

Обозначив абсциссу точки А через а, запишем А(а; 0).

По формуле

,

находим:

.

Получаем уравнение

.

Упростив его, имеем

а2+10а–39=0.

Корни этого уравнения а1=-13; а2=3.

Получаем две точки А1(-13; 0) и А2(3; 0) – рис. 79.

Расстояние между двумя точками на плоскости

Рисунок 79.

 

Вычисление абсциссы (ординаты) точки, которая лежит на оси абсцисс (ординат) и находится на одинаковом расстоянии от двух заданных точек, рассмотрим на примере 139

 

Пример 139.

Найти на оси Оу точку, которая находится на одинаковом расстоянии от точек А(6; 12) и В(-8; 10).

Решение:

пусть координаты нужной по условию задачи точки, лежащей на оси Оу, будут О1(0; b) (у точки, лежащей на оси Оу, абсцисса равна нулю). Из условия следует, что О1А=О1В.

По формуле

,

находим:

,

.

Имеем уравнение

.

Выполняя элементарные преобразования при решении иррациональных уравнений, получим b=4.

Необходимая по условию задачи точка О1(0; 4) – рис. 80.

Расстояние между двумя точками на плоскости

Рисунок 80.

 

Деление отрезка в данном отношении

Координаты x, y, z точки М, которая делит отрезок М1М2, ограниченный точками М1(x1, y1, z1) и М2(x2, y2, z2), в отношении λ, определяется по формулам:

.

 

Пример 140.

Даны концы отрезка АВ: А(-2; 5) и В(4; 17). На этом отрезке расположена точка С, расстояние от которой до точки А в два раза больше расстояния от точки В. Вычислить координаты точки С.

Решение:

по условию задачи АС=2ВС, тогда λ=2.

По формулам:

,

вычислим координаты точки С:

Т.о., С(2; 13).

 

Пример 141.

Доказать, что треугольник АВС: А(-3; -3), В(-1; 3), С(11; -1) – прямоугольный.

Решение:

вычислим длины сторон треугольника по формуле:

,

,

,

.

Т.к. АВ2=40, ВС2=160, АС2=200, то АВ2+ВС2=АС2.

Т.о., сумма квадратов длин двух сторон треугольника равна квадрату длины третьей стороны. Из этого следует, что треугольник АВС прямоугольный и сторона АС является его гипотенузой.

 

Задания для самостоятельного решения:

1) Доказать, что треугольник с вершинами  A(3; -1; 2),  B(0; -2; 2),  C(-3; 2; 1) равнобедренный.

2) На оси абсцисс найти точку, расстояние от которой до точки А(-3; 4; 8) равно 12.

3) На оси ординат найти точку, равноудаленную от точек А(1; -3; 7) и В(5; 7; -5).

4) Даны вершины A(2; -1; 4), B(3; 2; -6), C(-5; 0; 2) треугольника. Вычислить длину его медианы, проведенной из вершины А.

5) Даны две вершины A(2; -3; -5), B(-1; 3; 2) параллелограмма ABCD и точка пересечения его диагоналей E(4; -1; 7). Определить две другие вершины этого параллелограмма.

6) Вычислить координаты концов отрезка, который разделен точками C(2; 0; 2) и D(5; -2; 0) на три равные части.

 

Контрольные вопросы:

1.        Запишите формулы для вычисления расстояния между двумя точками.

2.        Запишите формулы для вычисления координат середины отрезка.

3.        Запишите формулы деления отрезка в данном отношении.

 

Практическое занятие №19

Радианная мера угла. Вращательное движение

 

Цель работы:

студент должен:

знать:

-       определения радиана, синуса, косинуса, тангенса и котангенса числового аргумента;

-       значения тригонометрических функций некоторых аргументов;

-       знаки значений тригонометрических функций по координатным четвертям;

уметь:

-       переводить значения углов из радианной меры угла в градусную меру и наоборот;

-       вычислять простейшие тригонометрические выражения.

 

Сведения из теории:

Радианная мера

Угол в 1 радиан – это такой центральный угол, длина дуги которого равна радиусу окружности.

Радианная и градусная меры связаны между собой зависимостью1800=π радиан; угол в n0= радиан.

Значения тригонометрических функций могут быть найдены так, как это делалось в курсе геометрии, из прямоугольного треугольника с гипотенузой равной 1 и по очереди задаваемых углов: 300, 450, 600.

 

Знаки тригонометрических функций по координатным четвертям:

Номер координатной четверти

I

II

III

IV

sinα

+

+

cosα

+

+

tgα

+

+

ctgα

+

+

 

Единственная четная функция – косинус

cos(-α)=cosα.

Все остальные основные тригонометрические функции нечетные:

sin(-α)= -sinα;

tg(-α)= -tgα;

сtg(-α)= -сtgα.

 

Таблица 2. Значения основных тригонометрических функций

Радианная мера угла

0

π

Градусная мера угла

00

300

450

600

900

1200

1350

1500

1800

sinα

0

1

0

cosα

1

0

-1

tgα

0

1

-1

0

ctgα

1

0

-1

 

Радианная мера угла

Градусная мера угла

2100

2250

2400

2700

3000

3150

3300

3600

sinα

-1

0

cosα

0

-1

tgα

1

-1

0

ctgα

1

0

-1

 

Пример 30.

Вычислите: sin4050.

Решение:

полный круг – 3600 можно «отбросить»:

sin4050=sin(4050-3600)=sin450=.

 

Пример 31.

Выразите в радианной мере значение угла 360.

Решение:

чтобы «перевести» градусную меру угла в радианную, необходимо заданное значение умножить на , т.о. получим

360=

 

Пример 32.

Выразите в градусной мере значение угла .

Решение:

чтобы «перевести» радианную меру угла в градусную, необходимо заданное значение умножить на , т. о. получим

 

Задания для самостоятельного решения:

1 вариант

№1. Выразите в радианной (градусной) мере значение угла: 600;  .

№2. Вычислите:

2 вариант

№1. Выразите в радианной (градусной) мере значение угла: 1800; .

№2. Вычислите:

3 вариант

№1. Выразите в радианной (градусной) мере значение угла: 2700; .

№2. Вычислите:

4 вариант

№1. Выразите в радианной (градусной) мере значение угла: 1200; .

№2. Вычислите:

5 вариант

№1. Выразите в радианной (градусной) мере значение угла: 3100; .

№2. Вычислите:

6 вариант

№1. Выразите в радианной (градусной) мере значение угла: 3600; .

№2. Вычислите:

7 вариант

№1. Выразите в радианной (градусной) мере значение угла: 15000; .

№2. Вычислите:

8 вариант

№1. Выразите в радианной (градусной) мере значение угла: 2160; .

№2. Вычислите:

9 вариант

№1. Выразите в радианной (градусной) мере значение угла: 900; .

№2. Вычислите:

 

Контрольные вопросы:

1.     Что называется углом в 1 радиан?

2.     В каких единицах измеряются углы?

3.     Перечислите значения некоторых тригонометрических функций.

 

Практическое занятие № 20

Основные тригонометрические тождества. Формулы приведения

 

Цель работы:

студент должен:

знать:

-       основные тригонометрические тождества;

-       формулы приведения;

уметь:

-       выполнять преобразования тригонометрических выражений, используя основные тригонометрические тождества, формулы приведения.

 

Сведения из теории:

Основные формулы тригонометрии

Из определений синуса, косинуса, тангенса и котангенса следуют основные тригонометрические тождества:

sin2α+cos2α=1;

tgα∙ctgα=1;

 

Основой для остальных формул являются формулы сложения:

cos(α-β)=cosαcosβ+sinαsinβ;

cos(α+β)=cosαcosβ-sinαsinβ;

sin(α-β)=sinαcosβ-cosαsinβ;

sin(α+β)=sinαcosβ+cosαsinβ;

 

Из формул сложения, полагая , где n Є Z, получаем формулы приведения преобразования выражений вида:

sin, cos, tg, ctg, n Є Z.

Для запоминания этих формул удобно пользоваться мнемоническим правилом:

1. Перед приведенной функцией ставится тот знак, который имеет исходная функция в соответствующей координатной четверти:

2. Функция меняется на «кофункцию», если n нечетно; функция не меняется, если n четно. (Кофункциями синуса, косинуса, тангенса и котангенса называются соответственно косинус, синус, котангенс, тангенс).

 

Пример 33.

Могут ли синус и косинус одного и того же числа быть равными соответственно: 0,4 и 0,7.

Решение:

используя основное тригонометрическое тождество sin2α+cos2α=1, имеем:

0,42+0,72=0,16+0,49=0,65.

Т.к. 0,65≠1 значения синуса и косинуса одного и того же числа не могут быть равными соответственно: 0,4 и 0,7.

 

Пример 34.

Найдите значения других трех основных тригонометрических функций, если: sinα=-0,8 и π<α<1,5π.

Решение:

используя основное тригонометрическое тождество sin2α+cos2α=1, имеем:

cos2α=1-sin2α, тогда cos2α=1-(-0,8)2=1-0,64=0,36.

Т. к. π<α<1,5π (III координатная четверть), то cosα=-0,6.

По формуле  вычисляем

По формуле tgαctgα=1 вычисляем ctgα=1׃=.

 

Задания для самостоятельного решения:

 

1 вариант

1) Могут ли синус и косинус одного и того же числа быть равными соответственно: 0,5 и 0,5.

2) Найдите значения других трех основных тригонометрических функций, если:

cosα= и .

2 вариант

1) Могут ли синус и косинус одного и того же числа быть равными соответственно: 0,2 и -0,8.

2) Найдите значения других трех основных тригонометрических функций, если:

sinα= и .

3 вариант

1) Могут ли синус и косинус одного и того же числа быть равными соответственно: 0,6 и -0,8.

2) Найдите значения других трех основных тригонометрических функций, если:

cosα= и .

4 вариант

1) Могут ли синус и косинус одного и того же числа быть равными соответственно:  и .

2) Найдите значения других трех основных тригонометрических функций, если:

sinα=0,5 и .

5 вариант

1) Могут ли синус и косинус одного и того же числа быть равными соответственно:  и .

2) Найдите значения других трех основных тригонометрических функций, если:

cosα=0,4 и .

6 вариант

1) Могут ли синус и косинус одного и того же числа быть равными соответственно:  и .

2) Найдите значения других трех основных тригонометрических функций, если:

sinα= и .

7 вариант

1) Могут ли тангенс и котангенс одного и того же числа быть равными соответственно:  и .

2) Найдите значения других трех основных тригонометрических функций, если:

cosα= и .

8 вариант

1) Могут ли тангенс и котангенс одного и того же числа быть равными соответственно: 2,4 и .

2) Найдите значения других трех основных тригонометрических функций, если:

sinα=0,7 и .

9 вариант

1) Могут ли тангенс и котангенс одного и того же числа быть равными соответственно:  и .

2) Найдите значения других трех основных тригонометрических функций, если:

cosα=0,9 и .

 

Контрольные вопросы:

1.     Перечислите основные тригонометрические тождества.

2.     Сформулируйте мнемоническое правило.

 

Практическое занятие № 21

Синус, косинус двойного угла

 

Цель работы:

студент должен:

знать:

-       формулы двойного угла тригонометрических функций;

-       формулы половинного аргумента тригонометрических функций;

уметь:

-       выполнять преобразования тригонометрических выражений, используя формулы двойного угла.

 

Сведения из теории:

Формулы двойного угла тригонометрических функций:

Подставляя в формулы cos2t=1-2sin2t и cos2t=2cos2t-1 значение , получаем формулы половинного аргумента:

Разделив  на  получаем формулу

 

Пример 35.

Выразите функции данного угла через функции вдвое меньшего угла, sin420.

Решение:

используя формулу , имеем

sin420=sin(2∙210)=2sin210cos210.

 

Пример 36.

Вычислите 2sin150cos150.

Решение:

используя формулу , имеем

2sin150cos150=sin(2∙150)=sin300=0,5.

 

Пример 37.

Вычислите sin(π/12).

Решение:

по формуле , имеем

 

Задания для самостоятельного решения:

1 вариант

1) Выразите функции данного угла через функции вдвое меньшего угла: sin540.

2) Вычислите: .

2 вариант

1) Выразите функции данного угла через функции вдвое меньшего угла: tg.

2) Вычислите: .

3 вариант

1) Выразите функции данного угла через функции вдвое меньшего угла: cos160.

2) Вычислите: .

4 вариант

1) Выразите функции данного угла через функции вдвое меньшего угла: ctg.

2) Вычислите cosα, если  и .

5 вариант

1) Выразите функции данного угла через функции вдвое меньшего угла: sin.

2) Вычислите , если  и .

6 вариант

1) Выразите функции данного угла через функции вдвое меньшего угла: tg680.

2) Вычислите , если  и .

7 вариант

1) Выразите функции данного угла через функции вдвое меньшего угла: cos.

2) Вычислите , если  и .

8 вариант

1) Выразите функции данного угла через функции вдвое меньшего угла: ctg 1020.

2) Вычислите , если  и .

9 вариант

1) Выразите функции данного угла через функции вдвое меньшего угла: tg 1620.

2) Вычислите , если  и .

 

Контрольные вопросы:

1.     Запишите формулы двойного угла тригонометрических функций.

2.     Запишите формулы половинного аргумента тригонометрических функций.

 

Практическое занятие № 22

Преобразование тригонометрических выражений с использованием тригонометрических тождеств

 

Цель работы:

студент должен:

знать:

-       формулы преобразования суммы тригонометрических функций в произведение;

-       формулы преобразования произведения тригонометрических функций в сумму;

уметь:

-       выполнять преобразования тригонометрических выражений, используя тригонометрические тождества.

 

Сведения из теории:

Формулы преобразования суммы тригонометрических функций в произведение:

Для преобразования произведения тригонометрических функций в сумму применяются формулы:

 

Пример 38.

Преобразуйте в алгебраическую сумму sin5xsin3x.

Решение:

по формуле  имеем

 

Пример 39.

Вычислите: sin400+sin200.

Решение:

по формуле  имеем

 

Задания для самостоятельного решения:

1 вариант

1) Упростите:

.

2) Вычислите:

sin750+sin150.

3) Вычислите:

sin52030’·cos7030’.

2 вариант

1) Упростите:

.

2) Вычислите:

sin750+sin1050.

3) Вычислите:

sin37030’·sin7030’.

3 вариант

1) Упростите:

.

2) Вычислите:

соs750+соs150.

3) Вычислите:

8 cos7α·cos3α.

4 вариант

1) Упростите:

.

2) Вычислите:

3) Вычислите:

cos750·cos1050.

5 вариант

1) Упростите:

.

2) Вычислите:

3) Вычислите:

2 sin(x+α)·cos(x-α).

6 вариант

1) Упростите:

.

2) Вычислите:

3) Вычислите:

12 sin(-9αsin4α.

7 вариант

1) Упростите:
.

2) Вычислите:

tg22030’-tg67030’.

3) Вычислите:

4 sin16α·sin4α.

8 вариант

1) Упростите:
.

2) Вычислите:

tg13030’+tg76030’.

3) Вычислите:

4 cos(α+βcos(α-β).

9 вариант

1) Упростите:

.

2) Вычислите:

tg300+tg600.

3) Вычислите:

4 cos150sin200sin400.

 

Контрольные вопросы:

1.     Перечислите основные тригонометрические тождества.

2.     Перечислите формулы двойного угла тригонометрических функций.

3.     Какие есть формулы для преобразования суммы тригонометрических функций?

 

Практическое занятие № 23

Решение тригонометрических уравнений

 

Цель работы:

студент должен:

знать:

-       формулы для решения тригонометрических уравнений в общем виде и частные случаи решения;

уметь:

-       решать простейшие тригонометрические уравнения.

 

Сведения из теории:

Решение простейших тригонометрических уравнений

 

Уравнение cos t=a

Очевидно, что если |а|>1, то уравнение cos t=a не имеет решений, т.к. |cos t|≤1 для любого t.

Пусть |а|≤1. Надо найти все такие числа t, что cos t=a. На отрезке [0; π] существует только одно решение уравнения cos t=a – это число arсcos a.

Косинус – четная функция, и, значит на отрезке [-π; 0] уравнение также имеет единственное решение – это число –arсcos a.

Итак, уравнение cos t=a на отрезке [-π; π] длиной 2π имеет два решения tarсcos a (совпадающие при а=1).

Вследствие периодичности функции косинус все остальные решения отличаются от найденных на 2πn, (nЄZ), т.е. формула корней уравнения cos t=a имеет вид:

tarсcos a+2πn, (n Є Z).

 

Пример 40.

Решите уравнение: cos t=1/2.

Решение:

по формуле tarсcos (1/2)+2πn, (n Є Z).

Поскольку arсcos (1/2)=π/3 приходим к ответу t=± π/3+2πn, (n Є Z).

 

Пример 41.

Решите уравнение: cos t=-0,2756.

Решение:

по формуле tarсcos (-0,2756)+2πn, (n Є Z).

Значение arсcos (-0,2756) находим с помощью калькулятора или по таблице В.М. Брадиса, оно примерно равно 1,85.

Итак, приходим к ответу t=±1,85+2πn, (n Є Z).

 

Пример 42.

Решите уравнение: cos (2х-π/4)=1/2.

Решение:

по формуле

2х-π/4=±arсcos (1/2)+2πn, (n Є Z).

Поскольку arсcos (1/2)=π/3 получаем

2х-π/4=± π/3+2πn, (n Є Z)

2х=π/4± π/3+2πn, (n Є Z).

Разделив обе части уравнения на 2 получим ответ: х=π/8±π/6+πn, (nЄZ).

 

Уравнение sin t=a

Очевидно, что если |а|>1, то уравнение sin t=a не имеет решений, т.к. |sin t|≤1 для любого t.

При |а|≤1 на отрезке [-π/2; π/2] уравнение sin t=a имеет одно решение t1=arcsin a. На отрезке [π/2; 3π/2] функция синус убывает и принимает все значения от -1 до 1. По теореме о корне уравнение и на этом отрезке имеет одно решение.

Это решение есть число t2=π-arcsin a, т.к. sin t2=sin (π-t1)=sin t1=а.

Кроме того, поскольку -π/2≤ t1≤π/2,

имеем -π/2≤-t1≤π/2

и π-π/2≤π-t1≤π+π/2,

т.е. π/2≤ t2≤3π/2,  t2Є[π/2; 3π/2].

Итак, уравнение sin t=a на отрезке [π/2; 3π/2] имеет два решения t1=arcsin a и t2=π-arcsin a ( совпадающие при а=1). Учитывая, что период синуса равен 2π, получаем формулу для решения уравнения sin t=a:

t=(-1)karcsin ak, kЄZ.

 

Пример 43.

Решите уравнение: sin t=.

Решение:

по формуле t=(-1)karсsin ()+πk, (k Є Z).

Поскольку arсsin ()=π/4 приходим к ответу t=(-1)k π/4+πk, (k Є Z).

 

Пример 44.

Решите уравнение: sin t=0,3714.

Решение:

по формуле t=(-1)karсsin (0,3714)+πk, (k Є Z).

Значение arсsin (0,3714) находим с помощью калькулятора или по таблице В.М. Брадиса, оно примерно равно 0,3805.

Итак, приходим t= (-1)k0,3805+πk, (k Є Z).

 

Пример 45.

Решите уравнение: sin =.

Решение:

функция синус нечетная, поэтому sin=-sin=-.

Тогда по формуле: =(-1)karсsink, (k Є Z).

Т.к. arсsin=, имеем

=(-1)k()+πk, (k Є Z)

или

, (k Є Z).

Умножив обе части уравнения на 2, получим ответ:

, (k Є Z).

 

Уравнение tg x=a

При любом а на интервале (-π/2; π/2) существует одно число t , что tgt=a, – это arctg a. Поэтому уравнение tg x=a имеет на интервале (-π/2; π/2) длиной π единственный корень.

Функция тангенс имеет период π. Следовательно, остальные корни уравнения tg t=a отличаются от найденного на πn, (n Є Z), т.е.

t=arctg a+πn, (n Є Z).

 

Пример 46.

Решите уравнение: tg t=.

Решение:

по формуле t=arсtg()+πn, (n Є Z).

Поскольку arсtg()= приходим к ответу t=n, (n Є Z).

 

Пример 47.

Решите уравнение: tg t=5,177.

Решение:

по формуле t=arсtg(5,177)+πn, (n Є Z).

Значение arсtg(5,177) находим с помощью калькулятора или по таблице В.М. Брадиса, оно примерно равно 1,38.

Итак, приходим t=1,38+πn, (n Є Z).

 

Сводная таблица решения простейших тригонометрических уравнений

Уравнение

Решение

sinx=a

cosx=a

tgx=a

ctgx=a

 

Частные случаи решения простейших тригонометрических уравнений

Уравнение

Частные случаи

а=-1

а=0

а=1

sinx=a

cosx=a

tgx=a

ctgx=a

 

Задания для самостоятельного решения:

Решите уравнения:

1 вариант

1) ;

2) ;

3) .

2 вариант

1) ;

2) ;

3) .

3 вариант

1) ;

2) ;

3) .

4 вариант

1) ;

2) ;

3) .

5 вариант

1) ;

2) ;

3) .

6 вариант

1) ;

2) ;

3) .

7 вариант

1) ;

2) ;

3) .

8 вариант

1) ;

2) ;

3) .

9 вариант

1) ;

2) ;

3) .

 

Контрольные вопросы:

1.        Перечислите формулы для решения простейших тригонометрических уравнений в общем виде.

2.        Перечислите формулы частных случаев решения простейших

 

Практическое занятие № 24

Решение тригонометрических неравенств

 

Цель работы:

студент должен:

знать:

-       формулы для решения простейших тригонометрических неравенств;

уметь:

-       решать простейшие тригонометрические неравенства.

 

Сведения из теории:

Простейшими тригонометрическими неравенствами называются неравенства

sin x<m, sin x>m, cos x<m, cos x>m, tg x<m, tg x>m, ctg x<m, ctg x>m, где m – данное число.

Решить простейшее тригонометрическое неравенство – значит найти множество всех значений аргумента, которые обращают данное неравенство в верное числовое неравенство.

 

Пример 48.

Решить неравенство: 1) sin x>; 2) cos x>.

Решение:

1) решение иллюстрируется рисунком 1 слева: здесь точке М1 соответствует угол , М2 – угол , М3 – угол , М4 – угол .

Неравенство выполняется для <x< и <х<. Общим решением служит неравенство:

<х<, kЄZ.

2) Данное неравенство иллюстрируется рисунком 1 справа: здесь точке М1 соответствует угол , М2 – угол . Общим решением неравенства является

<х<, kЄZ.

 

Рисунок 1. Решение тригонометрического неравенства.

 

Задания для самостоятельного решения:

Решить неравенство:

1 вариант

1) ;

2) ;

3) .

2 вариант

1) ;

2) ;

3) .

3 вариант

1) ;

2) ;

3) .

4 вариант

1) ;

2) ;

3) .

5 вариант

1) ;

2) ;

3) .

6 вариант

1) ;

2) ;

3) .

7 вариант

1) ;

2) ;

3) .

8 вариант

1) ;

2) ;

3) .

9 вариант

1) ;

2) ;

3) .

 

Контрольные вопросы:

1.       Что называется простейшими тригонометрическими неравенствами?

2.     Проиллюстрируйте решение неравенства sin x>m на окружности.

 

Практическое занятие № 25

Построение графиков элементарных функций

 

Цель работы:

студент должен:

знать:

-       элементарные функции, что является их графиками;

уметь:

-       строить графики элементарных функций.

 

Сведения из теории:

Числовая функция

Числовой функцией с областью определения D называется соответствие, при котором каждому числу х из множества D сопоставляется по некоторому правилу число у, зависящее от х.

Функции обычно обозначают латинскими буквами. Рассмотрим произвольную функцию f. Независимую переменную х называют аргументом функции. Число у, соответствующее числу х, называют значением функции f в точке х и обозначают f(х). Область определения функции f обозначают D(f). Множество, состоящее из всех чисел f(х), таких, что х принадлежит области определения функции f, называют областью значений функции и обозначают Е(f).

Графиком функции f называют множество всех точек (ху) координатной плоскости, где у=f(х), а х «пробегает» всю область определения функции f.

 

График линейной функции

Линейная функция задается уравнением у=ах+b. Графиком линейной функций является прямая. Чтобы построить прямую достаточно две точки.

 

Пример 49.

Построить график функции у=2х+1.

Решение:

найдем две точки. В качестве одной из точек выгодно выбрать нуль. Если х=0, то у=2·0+1=1.

Берем еще какую-нибудь точку, например, 1. Если х=1, то у=2·1+1=3.

При оформлении заданий координаты точек обычно сводятся в таблицу:

х

0

1

у

1

3

Две точки найдены, выполним чертеж:

График линейной функции

Рисунок 2. График функции у=2х+1

 

При оформлении чертежа всегда подписываем графики.

Частные случаи линейной функции

Частные случаи линейной функции

Рисунок 3. Частные случаи графика линейной функции

 

1) Линейная функция вида у=ах (а≠0) называется прямой пропорциональностью. Например, . График прямой пропорциональности всегда проходит через начало координат. Таким образом, построение прямой упрощается – достаточно найти всего одну точку.

2) Уравнение вида у=b задает прямую, параллельную оси Ох, в частности, сама ось Ох задается уравнением у=0.

3) Уравнение вида х=b задает прямую, параллельную оси Оу, в частности, сама ось Оу задается уравнением х=0.

 

График квадратичной, кубической функции

Парабола. График квадратичной функции у=ax2+bx+c (а≠0) представляет собой параболу. Рассмотрим канонический случай: у=x2. Область определения – любое действительное число. Функция у=x2 является чётной. Если функция является чётной, то ее график симметричен относительно оси Оу.

График квадратичной функции (парабола)

Рисунок 4. График функции у=x2

 

Пример 50.

Построить график функции у=-х2+2х.

Решение:

сначала находим вершину параболы: , . Рассчитываем соответствующее значение «игрек»: у=-12+2·1=-1+2=1. Таким образом, вершина находится в точке (1; 1).

Теперь находим другие точки, при этом пользуемся симметричностью параболы.

х

-2

-1

0

1

2

3

4

у

-8

-3

0

1

0

-3

-8

Выполним чертеж:

Как быстро построить параболу?

Рисунок 5. График функции у=-х2+2х

 

Для квадратичной функции у=ax2+bx+c (а≠0) справедливо следующее: Если a>0, то ветви параболы направлены вверх. Если a<0, то ветви параболы направлены вниз.

 

Кубическая парабола

Кубическая парабола задается функцией у=х3. Область определения, область значений – любое действительное число. Функция является нечётной. График строим по точкам:

х

-2

-1

0

1

2

у

-8

-1

0

1

8

 

Кубическая парабола

Рисунок 6. График функции у=х3

 

График функции .

Область определения: D(y): [0; +∞). Область значений: E(y): [0; +∞). То есть, график функции полностью находится в первой координатной четверти. При построении подбираем такие значения «икс», чтобы корень извлекался нацело:

х

0

1

4

9

у

0

1

2

3

Строим график:

График корня из икс

Рисунок 7. График функции

Гипербола

Общий вид . Область определения: D(y): (-∞; 0) и (0; +∞). Область значений: E(y): (-∞; 0) и (0; +∞). Функция является нечётной, гипербола симметрична относительно начала координат.

Выполним чертеж:

График гиперболы

Рисунок 8. График функции

 

График функции вида  (а≠0) представляют собой две ветви гиперболы.

Если а>0, то гипербола расположена в первой и третьей координатных четвертях. Если а<0, то гипербола расположена во второй и четвертой координатных четвертях.

 

Пример 51.

Построить правую ветвь гиперболы .

Решение:

значения х выгодно подбираем так, чтобы делилось нацело:

х

1

2

3

6

у

6

3

2

1

Выполним чертеж:

Правая ветвь гиперболы

Рисунок 9. График функции

 

Задания для самостоятельного решения:

Построить графики функций:

1 вариант

1) у=x2+2x+3;

2) ;

3) .

2 вариант

1) у=x2-4x;

2) ;

3) .

3 вариант

1) у=-x2+2x-1;

2) ;

3) .

4 вариант

1) у=-x2+x;

2) ;

3) .

5 вариант

1) у=-2x2+3x;

2) ;

3) .

6 вариант

1) у=x2+x+3;

2) ;

3) .

7 вариант

1) у=x2-6x;

2) ;

3) .

8 вариант

1) у=-x2+8x+1;

2) ;

3) .

9 вариант

1) у=-2x2+x-3;

2) ;

3) .

 

Контрольные вопросы:

1.     Что называется функцией?

2.     Что является графиком линейной, квадратичной функций?

 

Практическое занятие № 26

Промежутки возрастания, убывания, наибольшее, наименьшее значения функции. Точки экстремума

 

Цель работы:

студент должен:

знать:

-       определения возрастающей (убывающей) функции;

-       определения точки максимума (минимума) функции;

уметь:

-       находить промежутки монотонности функции;

-       вычислять точки экстремума функции.

 

Сведения из теории:

Возрастание и убывание функций

Функция f возрастает на множестве Р, если для любых х1 и х2 из множества Р, таких, что х2>х1, выполнено неравенство f(x2)>f(x1).

Функция f убывает на множестве Р, если для любых х1 и х2 из множества Р, таких, что х2>х1, выполнено неравенство f(x2)<f(x1).

Иными словами, функция f называется возрастающей на множестве Р, если большему значению аргумента из этого множества соответствует большее значение функции. Функция f называется убывающей на множестве Р, если большему значению аргумента из этого множества соответствует меньшее значение функции.

 

Пример 52.

Докажите, что функция f(x)=1/x является убывающей.

Решение:

область определения функции: (-∞; 0) и (0; +∞). Рассмотрим поведение функции на каждом интервале:

(-∞; 0): х1=-8, х2=-4, т.е. х2>х1, тогда f(-8)=-0,125, f(-4)=-0,25, т.е f(x2)<f(x1), а значит функция f(x)=1/x является убывающей на интервале (-∞; 0).

(0; +∞): х1=4, х2=8, т.е. х2>х1, тогда f(4)=0, 25, f(8)=0,125, т.е. т.е f(x2)<f(x1), а значит функция f(x)=1/x является убывающей на интервале (0; +∞).

Однако эта функция не является убывающей на объединении этих промежутков. Например, 1>-1, но f(1)<f(-1).

 

При исследовании функций на возрастание и убывание принято указывать промежутки возрастания и убывания максимальной длины, включая концы (если, конечно, они входят в эти промежутки). Так, можно было сказать, что функция f(x)=1/x является убывающей на отрезке [2; 500]. Это верно, но такой ответ неполон.

 

При исследовании поведения функции вблизи некоторой точки удобно пользоваться понятием окрестности.

Окрестностью точки а называется любой интервал, содержащий эту точку. Например, интервал (2; 6) – одна из окрестностей точки 3, интервал (-3,3; -2,7) – окрестность точки -3.

 

Экстремумы

Точка х0 называется точкой минимума функции f, если для всех х из некоторой окрестности х0 выполнено неравенство f(x)≥f(x0).

Точка х0 называется точкой максимума функции f, если для всех х из некоторой окрестности х0 выполнено неравенство f(x)≤f(x0).

 

По определениям значение функции f в точке максимума х0 является наибольшим среди значений функции из некоторой окрестности этой точки, поэтому график функции в окрестности х0, как правило, имеет вид гладкого «холма» или заостренного «пика». В окрестности точки минимума графики, как правило, изображаются в виде «впадины», тоже или гладкой или заостренной.

Для точек максимума и минимума функции принято общее название – их называют точками экстремума.

Значение функции в этих точках называют соответственно максимумами и минимумами функции (общее название – экстремум функции). Точки максимума обозначают xmax, а точки минимума xmin. Значения функции в этих точках обозначаются соответственно ymax, ymin.

 

Пример 53.

Начертите эскиз графика функции f, если известно, что f возрастает на промежутке (-∞; 2] и убывает на промежутке [2; +∞). Какой будет точка х=2?

Решение:

схематично график можно изобразить в виде:

Рисунок 10. Эскиз графика

 

График имеет вид гладкого «холма», а значит точка х=2 – точка максимума.

 

Задания для самостоятельного решения:

Начертите эскиз графика функции f, определите вид точек, если:

1 вариант

f возрастает на промежутке (-∞; 2] и убывает на промежутке [2; +∞).

2 вариант

f возрастает на промежутках (-∞; -2] и [0; 3], убывает на промежутке [2; 0].

3 вариант

f возрастает на промежутке [1; 4] и убывает на промежутках (-∞; 1] и [4; +∞).

4 вариант

f возрастает на промежутках (-∞; -5] и [1; 5], убывает на промежутках [-5; 1] [5; +∞).

5 вариант

f возрастает на промежутке (-∞; 5] и убывает на промежутке [5; +∞).

6 вариант

f возрастает на промежутке (-∞; 0] и убывает на промежутке [0; +∞).

7 вариант

f возрастает на промежутке [-1; 2] и убывает на промежутках (-∞; -1] и [2; +∞).

8 вариант

f возрастает на промежутках (-∞; -4] и [2; 4], убывает на промежутках [-4; 2] [4; +∞).

9 вариант

f возрастает на промежутках (-∞; -3] и [2; 5], убывает на промежутках [-3; 2] [5; +∞).

 

Контрольные вопросы:

1.        Какая функция называется возрастающей (убывающей) на некотором промежутке?

2.     Дайте определение точке максимума (минимума) функции.

 

Практическое занятие № 26

Арифметические операции над функциями

 

Цель работы:

студент должен:

знать:

-       графики элементарных функций;

уметь:

-       строить график функции как композицию двух функции.

 

Сведения из теории:

Построение графика суммы (произведения) двух функций производится сложением (умножением) ординат точек графиков с одинаковыми абсциссами.

 

Пример 54.

Пусть даны графики функций y=x и y=sin x. Построить y=x+sin x и y=xsin x, являющихся соответственно суммой и произведением заданных графиков.

Решение:

графики функций y=x+sin x и y=xsin x:

 

Рисунок 11. Графики функций y=x, y=sin x, y=x+sin x, y=xsin x

 

Пусть известен график y=f(x) и нужно построить график функции y=|f(x)|. По определению,

Значит, часть графика, лежащую в верхней координатной полуплоскости, изменять не надо, а часть графика, лежащую в нижней координатной полуплоскости, нужно отобразить симметрично относительно оси Oх.

 

Пусть известен график y=f(x) и нужно построить график функции y=f(|x|). Заметим, что при x≥0 f(|x|)=f(x), а функция y=f(|x|) четная. Поэтому, чтобы построить график функции y=f(|x|), нужно часть графика функции y=f(x), лежащую в левой координатной полуплоскости, отбросить, а часть графика, лежащую в правой координатной полуплоскости, отобразить симметрично относительно оси Oу.

 

Задания для самостоятельного решения:

Построить графики функций:

1 вариант

1) y=x+соs x;

2) у=|x2+2x+3|.

2 вариант

1) y=xсоs x;

2) у=|x2-4x|.

3 вариант

1) y=x+tg x;

2) у=|x2-6x|.

4 вариант

1) y=x tg x;

2) .

5 вариант

1) y=x+ctg x;

2) .

6 вариант

1) y=x ctg x;

2) .

7 вариант

1) y=-x+sin x;

2) .

8 вариант

1) y=-x sin x;

2) .

9 вариант

1) y=-x+cos x;

2) .

 

Контрольные вопросы:

1.     Как построить сумму (произведение) двух функций?

2.     Как построить модуль функции, модуль аргумента?

 

Практическое занятие № 27

Построение графиков

 

Цель работы:

студент должен:

знать:

-       свойства функций;

-       схему исследования функции;

уметь:

-       строить графики функций.

 

Сведения из теории:

Четные и нечетные функции

Рассмотрим функции области определения которых симметричны относительно начала координат, т.е. для любого х из области определения функции число (-х) также принадлежит области определения. Среди таких функций выделяют четные и нечетные.

Функция f называется четной, если для любого х из области определения f(-x)=f(x).

Функция f называется нечетной, если для любого х из области определения f(-x)=-f(x).

 

Пример 55.

Определите какая из функций является четной (нечетной): у=х4, у=х3.

Решение:

функция у=х4 четная, т.к. х4=(-х)4, т.е. у(-x)=у(x), а функция у=х3 является нечетной, т.к. х3=(-х)3=- х3, т.е. у(-x)=-у(x).

 

Пример 56.

Докажите, что функция  четная.

Решение:

вычислим f(-x):

,

т.е.  – четная по определению.

 

Свойства графиков:

1. График четной функции симметричен относительно оси ординат.

2. График нечетной функции симметричен относительно начала координат.

Из этих двух правил вытекает следующее: при построении графика четной или нечетной функции достаточно построить его часть для неотрицательных х, а затем отразить полученный график относительно оси ординат (в случае четной функции) или начала координат (в случае нечетной).

 

Ранее мы строили графики функций «по точкам». Во многих случаях этот метод дает хорошие результаты, если, конечно, отметить достаточно большое число точек. Однако при этом приходится составлять большие таблицы значений функции, а главное, можно не заметить существенных особенностей функции и в итоге ошибиться при построении графика.

Для того чтобы избежать ошибок, надо научиться выявлять характерные особенности функции, т.е. предварительно провести ее исследование.

 

Схема исследования функций:

1.   Найти область определения и область значений данной функции.

2.   Выяснить, обладает ли функция особенностями, облегчающими исследование, т.е. является ли функция четной (нечетной), периодической.

3.   Вычислить координаты графика функции с осями координат.

4.   Найти промежутки знакопостоянства функции.

5.   Выяснить, на каких промежутках функция убывает, а на каких возрастает.

6.   Найти точки экстремума, вид экстремума (минимум или максимум) и вычислить значения функции в этих точках.

7.   Исследовать поведение функции в окрестности характерных точек, не входящих в область определения (например, точка х=0 для функции f(x)=1/x), и при больших (по модулю) значениях, аргумента.

 

Необходимо заметить, что этот план имеет примерный характер.

 

Задания для самостоятельного решения:

Построить график функции f, если известны ее свойства:

1.

Свойство функции

1 вариант

2 вариант

3 вариант

4 вариант

 

Область определения

Область значений

[-6; 6]

 

[-2; 5]

[-5; 4]

 

[0; 6]

[-4; 4]

 

[-3; 6]

[-5; 3]

 

[0; 5]

2

Точки пересечения графика:

а) с осью Ох

(-4; 0),

(-2; 0)

(0; 0)

(-4; 0),

(-1; 0),

(2,5; 0)

(3; 0)

б) с осью Оу

(0; 2,5)

(0; 0)

(0; -2)

(0; 4,5)

3

Промежутки знакопостоянства:

а) f(x)>0

[-6; -4),

(-2; 6]

[-5; 0),

(0; 4]

(-4; -1),

(2,5; 4]

[-5; 3)

б) f(x)<0

(-4; -2)

-

(-1; 2,5)

-

4

Промежутки:

а) возрастания

[-3; 1],

[4; 6]

[-5; -2],

[0; 4]

[-4; -2],

[1; 4]

[-3; 1]

б) убывания

[-6; -3],

[1; 4]

[-2; 0]

[-2; 1]

[-5; -3],

[1; 3]

5.1

Точки максимума, максимум функции

xmax=1

ymax=3

xmax=-2

ymax=2

xmax=-2

ymax=2

xmax=1

ymax=5

5.2

Точки минимума, минимум функции

xmin1=-3,

ymin1=-2;

xmin2=4

ymin2=1

xmin=0

ymin=0

xmin=1

ymin=-3

xmin=-3

ymin=2

6

Дополнительные точки графика

(-6; 3),

(6; 5)

(-5; 0,5),

(4; 6)

(4; 6)

(-5; 3)

 

Контрольные вопросы:

1.   Перечислите свойства функций.

2.   Перечислите основные этапы исследования функции.

 

Практическое занятие № 28

Степенная функция, ее график и свойства

 

Цель работы:

студент должен:

знать:

-       свойства степенной функции с различными показателями степени;

уметь:

-       строить график степенной функции с различными показателями степени.

 

Сведения из теории:

Степенная функция с натуральным показателем

Функция у=хn, где n – натуральное число, называется степенной функцией с натуральным показателем. При n=1 получаем функцию у=х.

 

Прямая пропорциональность

Прямой пропорциональностью называется функция, заданная формулой у=kxn, где число k называется коэффициентом пропорциональности.

Перечислим свойства функции у=kx:

1. Область определения функции – множество всех действительных чисел.

2. y=kx – нечетная функция, т.к. f(-х)=k(-х)=-kx=-k(х)=-f(х).

3. При k>0 функция возрастает, а при k<0 убывает на всей числовой прямой.

Рисунок 12. График функции у=kx

 

При n=2 получаем функцию y=х2. Перечислим свойства функции y=х2:

1. Область определения функции – вся числовая прямая.

2. y=х2 – четная функция, т.к. f(-х)=(-x)2 =x2=f(х).

3. На промежутке [0; +∞) функция возрастает. На промежутке (-; 0] функция убывает.

4. Графиком функции y=х2 является парабола.

Рисунок 13. График функции y=х2

 

При n = 3 получаем функцию у=х3, ее свойства:

1. Область определения функции – вся числовая прямая.

2. у=х3 – нечетная функция, т.к. f(-х)=(-x)3 =-х3=-f(x).

3. Функция у=х3 возрастает на всей числовой прямой.

4. График функции у=х3 называется кубической параболой.

Рисунок 14. График функции y=х3

 

Пусть n – произвольное четное натуральное число, большее двух: n=4, 6, 8,... .

В этом случае функция у=хn обладает теми же свойствами, что и функция у=х2. График такой функции напоминает параболу у=х2, только ветви графика при |n|>1 круче идут вверх, чем больше n, а при |n|<1 «теснее прижимаются» к оси х, чем больше n.

Пусть n – произвольное нечетное число, большее трех: n=5, 7, 9, ... .

В этом случае функция у=хn обладает теми же свойствами, что и функция у=х3. График такой функции напоминает кубическую параболу (только ветви графика тем круче идут вверх, вниз, чем больше n). Отметим также, что на промежутке (0; 1) график степенной функции у=хn тем медленнее отдаляется от оси Ох с ростом х, чем больше n.

 

Степенная функция с целым отрицательным показателем.

Рассмотрим функцию у=х-n, где n – натуральное число. При n=2 получаем у=х-2 или у=. Свойства этой функции:

1. Функция определена при всех х0.

2. у= – четная функция.

3. у= – убывает на (0; +∞) и возрастает на (-∞; 0).

Теми же свойствами обладают любые функции вида y=х-n при четном n, большем двух.

Функции вида , ,  обладают теми же свойствами, как и функция .

 

Степенная функция с положительным дробным показателем

Рассмотрим функцию у=хr, где r – положительная несократимая дробь. Перечислим некоторые свойства этой функции:

1. Область определения – луч [0; +∞).

2. Функция ни четная, ни нечетная.

3. Функция у = хr возрастает на [0; +∞).

Рисунок 15. Графики степенных функций

 

На рисунке слева изображен график функции . Он заключен между графиками функций у=х2 и у=х3, заданных на промежутке [0; +∞).

Подобный вид имеет график любой функции вида у=хr, где .

На том же рисунке посередине изображен график функции . Подобный вид имеет график любой степенной функции у=хr, где .

 

Степенная функция с отрицательным дробным показателем

Рассмотрим функцию у=х-r, где r – положительная несократимая дробь. Перечислим свойства этой функции:

1. Область определения – промежуток (0; +∞).

2. Функция ни четная, ни нечетная.

3. Функция у=х-r убывает на (0; +∞).

 

Пример 57.

Построить график функции .

Решение:

построим таблицу значений данной функции:

х

1

4

9

у

3

2

1

Нанесем полученные точки на координатную плоскость и соединим их плавной кривой:

Рисунок 16. График функции

 

Подобный вид имеет график любой функции у=х-r, где r – отрицательная дробь.

 

Задания для самостоятельного решения:

Постройте график функции и опишите ее свойства:

1 вариант

.

2 вариант

.

3 вариант

.

4 вариант

.

5 вариант

.

6 вариант

.

7 вариант

.

8 вариант

.

9 вариант

.

 

Контрольные вопросы:

1.     Что называется степенной функцией?

2.     Перечислите виды степенных функций.

3.      Перечислите свойства функции для различных показателей степени.

 

Практическое занятие № 29

Логарифмическая функция, ее график и свойства

 

Цель работы:

студент должен:

знать:

-       основные свойства логарифмов;

уметь:

-       строить график логарифмической функции с разными основаниями.

 

Сведения из теории:

Пусть а – положительное число, а≠1.

Функцию, заданную формулой y=logax называют логарифмической функцией с основанием а.

Перечислим основные свойства логарифмической функции:

1. Область определения – множество всех положительных чисел R+, т.е. D(loga)=(0; +∞).

2. Область значений – множество всех действительных чисел R, т.е. Е(loga)=(-∞; +∞).

3. Логарифмическая функция на всей области определения возрастает при a>1 или убывает при 0<а<1.

 

Для построения графика заметим, что значение 0 логарифмическая функция принимает в точке 1; loga1=0 при любом a>1, т.к. а0=1.

Вследствие возрастания функции при a>1 получаем, что при х>1 логарифмическая функция принимает положительные значения, а при 0<х<1 – отрицательные.

Если 0<а<1 ,то логарифмическая функция убывает на R+, поэтому функция принимает положительные значения при 0<х<1, а при х>1 – отрицательные.

Опираясь на все вышесказанное строим графики логарифмической функции y=logax при a>1 и при 0<а<1.

http://wiki.ippk.ru/images/1/19/Clip_image225.jpg     http://wiki.ippk.ru/images/5/52/Clip_image226.jpg

Рисунок 17. График логарифмической функции

 

Справедливо следующее утверждение: графики показательной и логарифмической функций, имеющих одинаковое основание, симметричны относительно прямой у=х.

 

Пример 58.

Решить графически уравнение lоg2х=-х+1.

Решение:

построим графики функций у=lоg2х и у=-х+1 в одной координатной плоскости:

Рисунок 18. Графики функций у=lоg2х и у=-х+1

 

Графики этих функций пересекаются в точке с абсциссой х=1.

Проверка показывает, что х=1 – корень данного уравнения.

 

Задания для самостоятельного решения:

Решите графически уравнение:

1 вариант

.

2 вариант

.

3 вариант

.

4 вариант

.

5 вариант

.

6 вариант

.

7 вариант

.

8 вариант

.

9 вариант

.

 

Контрольные вопросы:

1.     Что называется логарифмической функцией?

2.     Перечислите свойства логарифмической функции.

 

Практическое занятие № 30

Косинус, его график и свойства

 

Цель работы:

студент должен:

знать:

-       определение тригонометрических функций;

-       свойства тригонометрических функций;

уметь:

-       строить косинусоиду.

 

Сведения из теории:

Функции синус и косинус

Числовые функции, заданные формулами y=sin x и y=cos x, называют соответственно синусом и косинусом (и обозначают sin и cos).

Область определения этих функций – множество всех действительных чисел. Областью значений функций синус и косинус является отрезок [-1; 1]. Т.е. D(sin)=D(cos)=R; E(sin)=E(cos)=[-1; 1].

 

Свойства функций синус и косинус:

для любого х справедливы равенства:

1) sin(-x)=-sin x, cos (-x)=cos x;

2) sin(x+2πn)=sin x, cos(x+2πn)=cos x, где n – произвольное целое число.

 

Синусоида

Построим график функции синус на отрезке [0; 2π]. Для этого отметим на оси ординат точки (0; -1) и (0; 1), а на оси абсцисс точку с абсциссой 2π (длина отрезка [0; 2π] шесть клеток ~ 6,28). Далее пользуясь вычисленными значениями синуса построим график функции на отрезке [0; 2π]. Вне этого отрезка заметим, что sin(x+2πn)=sin x и с помощью параллельных переносов вдоль оси Ох влево и вправо достроим график функции на отрезках [-4π; -2π], [-2π; 0], [2π; 4π]. График синуса называется синусоидой.

График синуса (синусоида)

Рисунок 19. График функции y=sin x

 

Для построения графика косинуса необходимо воспользоваться формулой cos x=sin(x+π/2). Это означает, что график косинуса получается из графика синуса с помощью параллельного переноса на расстояние π/2 влево вдоль оси Ох. Поэтому график функции y=cos x также является синусоидой.

График косинуса

Рисунок 20. График функции y=cos x

 

Сведем известные свойства функций в таблицу (всюду полагая, что n – произвольное целое число).

 

 

Функция

y=sin x

y=cos x

1.1 Область определения

R

R

1.2 Область значений

[-1; 1]

[-1; 1]

2.1 Четность (нечетность)

Нечетная

Четная

2.2 Наименьший положительный период

3.1 Координаты точек пересечения графика с осью Ох

(πn; 0)

(π/2+πn; 0)

3.2 Координаты точек пересечения графика с осью Оу

(0; 0)

(0; 1)

4.1 Промежутки, на которых функция принимает положительные значения

(2πn; π+2πn)

(-π/2+2πn; π/2+2πn)

4.2 Промежутки, на которых функция принимает отрицательные значения

(-π+2πn; 2πn)

(π/2+2πn; 3π/2+2πn)

5.1 Промежутки возрастания

[-π/2+2πn; π/2+2πn]

[-π+2πn; 2πn]

5.2 Промежутки убывания

[π/2+2πn; 3π/2+2πn]

[2πn; π+2πn]

6.1 Точки минимума

-π/2+2πn

π+2πn

6.2 Минимумы функции

-1

-1

6.3 Точки максимума

π/2+2πn

2πn

6.4 Максимумы функции

1

1

 

Задания для самостоятельного решения:

№1. Построить схематически косинусоиду на интервале [-3π; 3π] и выполнить следующие упражнения:

1) Проиллюстрировать по графику, что:

а) функция cos x не может принимать значений, превосходящих по абсолютной величине единицу, т. е. -1<cos x<1;

б) каждому действительному значению х соответствует только одно значение cos х (свойство однозначности косинуса);

в) при замене произвольного значения аргумента х противоположным ему значением – х значение функции не изменяется, т. е. cos(-х)=cos х (свойство четности косинуса). Как можно использовать свойство четности косинуса при построении его графика;

г) при изменении произвольного значения аргумента на число, кратное числу 2π, значение функции cos x не изменяется, т. е. cos(х+2πk)=cos x (свойство периодичности косинуса). Как можно использовать периодичность косинуса при построении его графика;

д) при изменении произвольного знамения аргумента на число π значение функции у заменяется противоположным ему значением -у, т. е. cos(x±π)=-cos x;

е) уравнение cos х=0,5 имеет бесчисленное множество решений. Назвать несколько частных решений этого уравнения.

2) Указать интервалы, в которых функция у=cos х принимает:

а) положительные значения;

б) отрицательные значения.

Какие четверти единичной окружности соответствуют этим интервалам.

3) Выделить на оси абсцисс и на единичной окружности интервалы, в которых функция у=cos x:

а) возрастает;

б) убывает.

Проиллюстрировать на графике, что в любом интервале монотонности косинус последовательно принимает все свои возможные значения, каждому из которых соответствует только одно значение аргумента в рассматриваемом интервале.

 

№2. По графику функции у=cos x ответить на следующие вопросы:

1) Как изменяется cos x, если аргумент х:

а) увеличивается от -2π до π;

б) уменьшается от 2,5π до 1,5π?

2) Чему равен косинус числа: а) π; б) 2π; в) -0,5π; г) -2π?

3) Что меньше: a) cos 0,7 или cos 1; б) cos(π/2+1) или cos(π/2-1)?

4) При каких значениях х функция cos x равна: а) 0; б) 1; в) -1?

5) Проиллюстрировать на графике, что не существует значений аргумента х, при которых функция cos x была равна 2.

 

Контрольные вопросы:

1.     Какие функции называют синусом и косинусом?

2.     Что является графиком функций синус и косинус?

3.     Перечислите свойства функций синус и косинус.

 

Практическое занятие № 30

Котангенс, его график и свойства

 

Цель работы:

студент должен:

знать:

-       определение тригонометрических функций;

-       свойства тригонометрических функций;

уметь:

-       строить графики тригонометрических функций.

 

Сведения из теории:

Числовые функции, заданные формулами y=tg x и y=ctg x, называют соответственно тангенсом и котангенсом (и обозначают tg и ctg).

Областью определения функции тангенс является множество всех чисел х, для которых cos x≠0, т.е. все числа х≠π/2+πn, где n - произвольное целое число. Областью определения функции котангенс является множество всех чисел х, для которых sin x≠0, т.е. все числа х≠πn, где n - произвольное целое число.

Область значений тангенса (котангенса) – вся числовая прямая.

 

Свойства функций тангенс и котангенс:

для любого х справедливы равенства:

1) tg (-x)=-tg x, ctg (-x)=-ctg x;

2) tg (x+πn)=tg x, ctg (x+πn)=ctg x, где n – произвольное целое число.

Построение графика тангенса на интервале (-π/2; π/2) аналогично построению синуса. Вследствие тождества tg (x+πn)=tg x график тангенса на всей области определения получается из графика на интервале (-π/2; π/2) параллельным переносом вдоль оси Ох влево и вправо на π, 2π и т.д. График функции тангенс называют тангенсоидой.

График тангенса

Рисунок 21. График функции y=tg x

 

Для построения графика y=ctg x воспользуемся тождеством ctg x=-tg (x+π/2). Из этого тождества следует, что для построения графика котангенса необходимо сдвинуть график тангенса на π/2 влево вдоль оси Ох и отразить полученную кривую относительно оси Ох.

График котангенса

Рисунок 22. График функции y=ctg x

 

Сведем известные свойства функций в таблицу (всюду полагая, что n – произвольное целое число).

 

Функция

y=tg x

y=ctg x

1.1 Область определения

(-π/2+πn; π/2+πn)

(πn; π+πn)

1.2 Область значений

R

R

2.1 Четность (нечетность)

Нечетная

Нечетная

2.2 Наименьший положительный период

π

π

3.1 Координаты точек пересечения графика с осью Ох

(πn; 0)

(π/2+πn; 0)

3.2 Координаты точек пересечения графика с осью Оу

(0; 0)

Нет

4.1 Промежутки, на которых функция принимает положительные значения

(πn; π/2+πn)

(πn; π/2+πn)

4.2 Промежутки, на которых функция принимает отрицательные значения

(-π/2+πn; πn)

(-π/2+πn; πn)

5.1 Промежутки возрастания

(-π/2+πn; π/2+πn)

Нет

5.2 Промежутки убывания

Нет

(πn; π+πn)

6.1 Точки минимума

Нет

Нет

6.2 Точки максимума

Нет

Нет

 

Задания для самостоятельного решения:

№1. Построить схематически тангенсоиду на интервале (-3π/2; 3π/2). При построении:

1) отметить на оси абсцисс точки, соответствующие числам:

-1,5π; -π; -0,5π; 0,5π; π; 1,5π (за единицу масштаба принять отрезок, равный 1 см);

2) через точки (-1,5π; 0); (-0,5π; 0); (0,5π; 0) и (1,5π; 0) провести (пунктиром) прямые, параллельные оси ординат;

3) отметить точки тангенсоиды с ординатами ±1;

4) вычертить (от руки) тангенсоиду.

 

№2. Пользуясь схематическим графиком функции у=tg x выполнить следующие упражнения:

1) Указать интервалы, в которых функция принимает: а) положительные значения; б) отрицательные значения.

2) Определить, при каких значениях х на интервале (-3π/2; 3π/2) функция у=tg x: а) убывает; б) возрастает; в) принимает значение, равное нулю; г) теряет смысл.

Выразить формулой множество таких значений х, при которых у=tg x теряет смысл.

3) Убедиться, что каждому допустимому значению аргумента х соответствует только одно значение функции.

4) Проиллюстрировать на графике, что функция у=tg x есть периодическая функция с периодом π, т. е. tg(xk)=tg x.

5) Показать, что каждому значению функции у соответствует бесчисленное множество определенных значений аргумента х.

6) Решить неравенства: a) tg x>-1; б) | tg x |<1.

 

№3. Построить на одном чертеже графики функций: у=х; у=sin х и у=tg х, если 0<х<π/2. Пользуясь чертежом, проиллюстрировать неравенство sin x<х<tg x.

 

Контрольные вопросы:

1.     Какие функции называют тангенсом и котангенсом?

2.     Что является графиком функций тангенс и котангенс?

3.     Перечислите свойства функций тангенс и котангенс.

 

 

Практическое занятие № 31

Параллельный перенос, симметрия относительно осей ординат

 

Цель работы:

студент должен:

знать:

-       графики элементарных функций;

-       формулы преобразования графиков;

уметь:

-       выполнять построение графиков функций с помощью параллельного переноса, симметрии относительно осей ординат.

 

Сведения из теории:

Параллельный перенос на вектор (0; b) вдоль оси ординат.

Обозначая через (; ) координаты точки, в которую переходит произвольная точка (х; y) плоскости при данном преобразовании, получим формулы:

.

Для построения графика функции f(х)+b, где b – постоянное число, надо «перенести» график функции f(х) на вектор (0; b) вдоль оси Оу вверх если b>0, вниз – если b<0.

 

Пример 59.

Построить графики функций: а) у=sin x+2; б) у=х2-5.

Решение:

а) в соответствии с правилом график функции у=sin x+2 получается из у=sin x параллельным переносом вдоль оси Оу вверх на две единицы;

б) график функции у=х2-5 получается из у=х2 параллельным переносом вдоль оси Оу вниз на пять единиц.

 

Параллельный перенос вдоль оси абсцисс на вектор (а; 0) задается формулами:

.

График функции y=f(x-a) получается из графика функции y=f(x) переносом вдоль оси абсцисс на а единиц вправо, если a>0, влево – если а<0.

 

Пример 60.

Построить графики функций: а) у=cos(x-π/4); б) у=.

Решение:

а) в соответствии с правилом график функции у=cos(x-π/4) получается из у=cos(x) параллельным переносом вдоль оси Ох вправо на π/4;

б) график функции у= получается из у= параллельным переносом вдоль оси Ох влево на одну единицу.

 

При осевой симметрии относительно оси Ох точка (x; y) переходит в точку (x; -y).

При осевой симметрии относительно оси Оу точка (x; y) переходит в точку (-x; y).

При центральной симметрии относительно начала координат (x; y) переходит в точку (-x; -y).

 

Задания для самостоятельного решения:

№1. Дана точка А(4; 6). Построить точку В, симметричную точке А относительно оси абсцисс, и найти координаты этой точки. Показать, что если точки A и В симметричны относительно оси абсцисс, то их абсциссы равны, а ординаты отличаются только знаками.

№2. Построить точку A(-3; 5) и точку В, симметричную точке А относительно оси ординат. Показать, что если точки A и В симметричны относительно оси ординат, то их ординаты равны, а абсциссы отличаются только знаками.

№3. Построить в одной системе координат графики функций (записать цепочку движения):

1 вариант

у=sin x,

у=sin (x-π/6),

у=sin x+2.

2 вариант

у=cos x,

у=cos x-2,

у=cos (x+π/3).

3 вариант

у=tg x,

у=tg (x-π/2),

у=tg x+3.

4 вариант

у=х2,

у=х2+4,

у=(х-3)2.

5 вариант

у=х2,

у=(х+1)2,

у=х2-2.

6 вариант

у=х3,

у=х3+1,

у=(х-2)3.

7 вариант

у=|х|,

у=|х-2|,

у=|х|+6.

8 вариант

,

,

.

9 вариант

у=|х|,

у=|х+1|,

у=|х|-2.

 

Контрольные вопросы:

1.     Какими формулами задается параллельный перенос?

2.     Как построить точку симметричную относительно осей координат?

 

 

Практическое занятие № 32

Параллельный перенос, растяжение, сжатие

 

Цель работы:

студент должен:

знать:

-       графики элементарных функций;

-       формулы преобразования графиков;

уметь:

-       выполнять построение графиков функций с помощью параллельного переноса, растяжения, сжатия.

 

Сведения из теории:

Растяжение вдоль оси Оу с коэффициентом k, которое задается формулами:

.

Для построения графика функции y=kf(х) надо «растянуть» график функции y=f(х) в k раз вдоль оси ординат (|k|>1).

Если 0<|k|<1, то растяжение с коэффициентом k называют сжатием.

 

Пример 61.

Построить графики функций: а) у=cos x; б) у=-2х2.

Решение:

а) в соответствии с правилом график функции у=cos x получается из у=cos x сжатием вдоль оси ординат с коэффициентом 3;

б) график функции у=-2х2 получается из у=х2 растяжением вдоль оси ординат с коэффициентом 2.

 

Растяжение вдоль оси Ох с коэффициентом k, которое задается формулами:

.

Для построения графика функции y=f(х/k) надо подвергнуть график функции y=f(х) растяжению с коэффициентом k вдоль оси абсцисс.

 

Пример 62.

Построить графики функций: а) у=sin(x/3); б) у=cos(2х).

Решение:

а) в соответствии с правилом график функции у=sin(x/3) получается из у=sin x растяжением вдоль оси Ох с коэффициентом 3;

б) график функции у=cos(2х) получается из у=cos х сжатием вдоль оси Ох с коэффициентом 2.

 

Задания для самостоятельного решения:

Построить в одной системе координат графики функций (записать цепочку движения):

1 вариант

,

,

.

2 вариант

,

,

.

3 вариант

,

,

.

4 вариант

,

,

.

5 вариант

,

,

.

6 вариант

,

,

.

7 вариант

,

,

.

8 вариант

,

,

.

9 вариант

,

,

.

 

Контрольные вопросы:

1.     Какими формулами задается растяжение (сжатие)?

 

 

Практическая работа № 33

Решение задач на перебор вариантов

 

Цель работы:

студент должен:

знать:

- определение соединений, их видов;

- определение вероятности;

- теоремы сложения, умножения вероятностей;

уметь:

-        по условию задачи различать виды соединений;

-        вычислять разные виды соединений;

-        вычислять вероятность событий.

 

Сведения из теории:

Соединения, их виды

Группы, составленные из каких – либо элементов, называются соединениями.

Различаю три основных вида соединений: размещения, перестановки и сочетания.

Размещениями из n элементов по m в каждом называются такие соединения, которые отличаются друг от друга либо самими элементами, либо порядком их расположения.

Число размещений из n элементов по m обозначается и вычисляется по формуле:

.

Перестановками из n элементов называются такие соединения из всех n элементов, которые отличаются друг от друга порядком расположения элементов.

Перестановки представляют частный случай размещений  из n элементов по n в каждом.

Число всех перестановок из n элементов равно произведению последовательных чисел от 1 до n включительно:

,

n!-читается «n-факториал», причем 0!=1 и 1!=1.

Используя приведенные выше определения имеем формулы:

,

при решении задач часто используется равенство:

.

Сочетаниями из n элементов по m в каждом называются такие соединения, которые отличаются друг от друга хотя бы одним элементом.

Число сочетаний из n элементов по m обозначается и вычисляется по формуле: 

,

которую можно записать также в виде

 

или

.

Кроме того, при решении задач используются следующие формулы, выражающие основные свойства сочетаний:

 

Пример 94.

Найти число размещений из 10 элементов по 4.

Решение:

по формуле :

.

 

Пример 95.

Решить уравнение: .

Решение:

используя формулу для вычисления числа размещений имеем:

.

Разделим обе части на одинаковые выражения, получим:

,

и решим получившееся квадратное уравнение: .

 

Пример 96.

Решите систему: .

Решение:

решим второе уравнение:

.

Т. к. , то –11 не удовлетворяет условию задачи. Подставив х=12 в первое уравнение системы, получим

.

Используя основное свойство сочетаний, имеем:

,

тогда

.

Ответ: х=12, у=5.

 

Пример 97.

Сколькими способами из восьми кандидатов можно выбрать три лица на три должности?

Решение:

условию задачи соответствуют размещения 3 из 8, имеем:

.

 

Случайные события

Изучение каждого явления в порядке наблюдения или производства опыта связано с осуществлением некоторого комплекса условий (испытаний). Всякий результат или исход испытания называется событием.

Если событие при заданных условиях может произойти или не произойти, то оно называется случайным.

В том случае, когда событие должно непременно произойти, его называют достоверным, а в том случае, когда оно заведомо не может произойти, невозможным.

События называются несовместными, если каждый раз возможно появление только одного из них. События называются совместными, если в данных условиях появление одного из этих событий не исключает появление другого при том же испытании.

События называются противоположными, если в условиях испытания они, являясь единственными его исходами, несовместны.

Вероятность события рассматривается как мера объективной возможности появления случайного события.

 

Классическое определение вероятности.

Вероятностью события А называется отношение числа благоприятных исходов m, к числу всех возможных исходов n:

.

Вероятность любого события не может быть меньше нуля и больше единицы, т. е. .

Невозможному событию соответствует вероятность Р(А)=0, а достоверному – вероятность Р(А)=1.

 

Пример 98.

В лотерее из 1000 билетов 200 выигрышных. Вынимают наугад один билет. Какова вероятность, что этот билет выигрышный?

Решение:

количество благоприятных событий, удовлетворяющих условию задачи m=200.

Число всех возможных вариантов n=1000.

По определению вероятности: Р(А)=200/1000=0,2.

 

Пример 99.

Из урны, в которой находятся 5 белых и 3 черных шара, вынимают один шар. Найти вероятность того, что этот шар черный?

Решение:

общее число шаров m=8, из них черных n=3, по определению: Р(А)=3/8=0,375.

 

Пример 100.

Из урны, в которой находятся 12 белых и 8 черных шара, вынимают наудачу два шара. Найти вероятность того, что оба шара окажутся черными?

Решение:

общее число возможных случаев n равно числу сочетаний из 20 (12+8) элементов по два:

;

число благоприятных исходов m равно числу сочетаний из 8 элементов по два:

.

По определению: Р(А)=28/190=0,147.

 

Пример 101.

В партии из 18 деталей находятся 4 бракованных. Наугад выбирают 5 деталей. Какова вероятность того, что из этих 5 деталей две окажутся бракованными?

Решение:

число всех равновозможных независимых исходов n равно числу сочетаний из 18 по 5:

.

Подсчитаем число благоприятных исходов m. Среди 5 взятых наугад деталей должно быть 3 качественных и 2 бракованных. Число способов выборки двух бракованных деталей из 4 имеющихся бракованных равно числу сочетаний из 4 по 2:

.

Число способов выборки трех качественных деталей из 14 имеющихся равно числу сочетаний из 14 по 3:

.

Любая группа качественных деталей может комбинироваться с любой группой бракованных, поэтому общее число комбинаций m равно:

,

по определению: Р(А)=2184/8568=0,255.

 

Задания для самостоятельного решения:

Решить следующие задачи, используя определение сочетаний, их видов:

1 вариант

1)    Сколько двузначных чисел можно составить из цифр 1, 3, 5, 8, 9 так, чтобы в каждом числе не было одинаковых цифр?

2)    Из 6 открыток надо выбрать 3. Сколькими способами это можно сделать?

3)    Решите уравнение: .

2 вариант

1)    Сколькими способами могут разместиться 5 человек вокруг круглого стола?

2)    Сколькими способами можно составить флаг, состоящий из трех горизонтальных полос различных цветов, если имеется материал семи различных цветов?

3)    Решите уравнение: .

3 вариант

1)    Из 10 кандидатов нужно выбрать 3 человека на конференцию. Сколькими различными способами это можно сделать?

2)    Сколько различных пятизначных чисел можно составить из цифр 0, 1, 3, 5, 7 так, чтобы в каждом числе не было одинаковых цифр?

3)    Решите уравнение: .

4 вариант

1)    Бригадир должен отправить на работу бригаду из 3 человек. Сколько таких бригад можно составить из 8 человек?

2)    На собрании должны выступить 5 человек (А, Б, В, Г, Д). Сколькими способами их можно разместить в списке выступающих, если А должен выступать первым?

3)    Решите уравнение: .

5 вариант

1)    Сколькими способами можно расставить на полке 6 книг?

2)    Сколькими способами можно выбрать гласную и согласную буквы из слова «журнал»?

3)    Решите уравнение: .

6 вариант

1)    Сколькими способами можно составить список из 6 человек?

2)    Сколькими способами собрание, состоящее из 18 человек, может из своего состава выбрать председателя собрания и секретаря?

3)    Решите уравнение: .

7 вариант

1)    Среди перестановок из цифр 1, 2, 3, 4, 5 сколько таких, которые не начинаются цифрами 3 или 5?

2)    Из города А в город В ведут 6 дорог, а из города В в город С –3 дороги. Сколькими способами можно попасть из города А в город С?

3)    Решите систему: .

8 вариант

1)    В шахматном турнире принимали участие 15 шахматистов, причем каждый из них сыграл только одну партию с каждым из остальных. Сколько всего партий сыграно в этом турнире?

2)    Имеется 8 пар перчаток различных размеров. Сколькими способами можно выбрать из них одну перчатку на левую руку и одну перчатку на правую руку так, чтобы эти перчатки были разных размеров?

3)    Решите систему: .

9 вариант

1) Группа учащихся изучает семь учебных дисциплин. сколькими способами можно составить расписание занятий на понедельник, если в этот учебный день должно быть четыре различных урока?

2) Сколько матчей будет сыграно в футбольном чемпионате с участием 16 команд, если каждые две команды встречаются между собой один раз?

3) Вычислить: .

 

Контрольные вопросы:

1.   Дайте определение соединения, их виды?

2.   Приведите формулы для вычисления разных видов соединений.

3.   Дайте определение случайного события, их виды. Приведите примеры.

4.   Дайте классическое определение вероятности.

 

Практическая работа № 34

Свойства биноминальных коэффициентов

 

Цель работы:

студент должен:

знать:

- формулу бинома Ньютона;

- свойства биноминальных коэффициентов;

уметь:

-       раскладывать бином по степеням х;

-       возводить в различные степени трехчлены.

 

Сведения из теории:

Формула бинома Ньютона

Бином Ньютона – это формула разложения степени двучлена (бинома) (a+b)n в виде многочлена от a и b.

Запишем разложения бинома Ньютона для нескольких первых значений n:

 

Чтобы найти коэффициент при akbn-k  в разложении бинома (a+b)n в общем случае, представим себе, что мы перемножаем n скобок и приводим подобные члены. Член akbn-k  встретится столько раз, сколько можно указать k скобок (из n возможных), из которых мы возьмем множитель а (а из остальных автоматически возьмем b). Это число равно числу выборок k скобок из n возможных, которое носит название числа сочетаний из n по k и обозначается .

В этих обозначениях формула имеет следующий вид:

.

Иными словами, число сочетаний из n по k равно коэффициенту при члене an-kbk  в разложении n-ой степени двучлена (a+b) поэтому числа сочетаний называют иначе биномиальными коэффициентами.

Эту связь можно использовать для вывода свойств сочетаний алгебраическими методами. Такой подход к выводу свойств комбинаторных объектов носит название метода производящих функций.

 

Свойства биномиальных коэффициентов

Биномиальные коэффициенты обладают большим количеством свойств.

Свойство 1. .

Свойство 2.  – биномиальные коэффициенты, равноотстоящие от концов, равны между собой

Свойство 3.  – сумма биномиальных коэффициентов при фиксированном n равна .

Свойство 4.  – суммы биномиальных коэффициентов, стоящих на четных и на нечетных местах, равны между собой (и равны по половине от общей суммы).

Свойство 5.  – рекуррентное соотношение, связывающее биномиальные коэффициенты для соседних степеней.

 

Пример 102.

Разложить бином (1+х)6 по степеням x.

Решение:

применяем формулу бинома Ньютона:

.

Значения биномиальных коэффициентов находим последовательно по формуле :

Т.о.

 

Задача для самостоятельного решения №1. Разложить бином (1+х)5 по степеням x.

 

Пример 103.

Возвести трехчлен a+b+c в четвертую степень.

Решение:

применяем формулу бинома Ньютона:

Задача для самостоятельного решения №2. Возвести трехчлен a+b+c в третью степень.

 

Контрольные вопросы:

1.   Запишите формулу бинома Ньютона.

2.   Перечислите свойства биноминальных коэффициентов.

 

Практическая работа № 35

Треугольник Паскаля

 

Цель работы:

студент должен:

знать:

- принцип построения треугольника Паскаля;

уметь:

-       возводить двучлен в любую натуральную степень.

 

Сведения из теории:

Треугольник Паскаля – бесконечная таблица биномиальных коэффициентов, имеющая треугольную форму. В этом треугольнике на вершине и по бокам стоят единицы. Каждое число равно сумме двух расположенных над ним чисел. Строки треугольника симметричны относительно вертикальной оси. Назван треугольник в честь Блеза Паскаля.

http://www.bymath.net/studyguide/alg/sec/alg31n.gif

Первая строка в этой таблице содержит биномиальные коэффициенты для n=1; вторая – для n=2; третья – для n=3 и т.д.

 

Пример 104.

Разложить выражение: (a+b)7.

Решение:

мы можем получить результат моментально, используя из таблицы разложение по седьмой строке (т.к. седьмая степень двучлена):

.

 

Задача для самостоятельного решения №1. Построить треугольник Паскаля до двадцатой строки.

Задача для самостоятельного решения №2. Разложить выражение: (a+b)n, где n – номер по журналу (если Ваш номер 1-7, то прибавьте к номеру число 5).

 

Контрольные вопросы:

1.   Сформулируйте принцип построения треугольника Паскаля.

 

 

Практическая работа № 36

Сложение и умножение вероятностей

 

Цель работы:

студент должен:

знать:

- теоремы сложения, умножения вероятностей;

уметь:

-       вычислять вероятность событий.

 

Сведения из теории:

Вероятность несовместных событий

Вероятность появления одного из нескольких попарно несовместных событий равна сумме вероятностей этих событий: Р(А+В)=Р(А)+Р(В).

 

Пример 105.

В ящике в случайном порядке разложены 20 деталей, причем пять из них стандартные. Рабочий берет наудачу три детали. Найти вероятность того, что, по крайней мере, одна из взятых деталей окажется стандартной (событие А).

Решение:

очевидно, что, по крайней мере, одна из взятых деталей окажется стандартной, если произойдет любое из трех несовместных событий: В – одна деталь стандартная, две нестандартные; С – две детали стандартные, одна нестандартная; Д – три детали стандартные.

Т.о., событие А можно представить в виде суммы этих трех событий: А=В+С+Д.

Тогда Р(А)=Р(В)+Р(С)+Р(Д).

Вычислим вероятность каждого события:

Итак,

 

Вероятность совместных событий

Вероятность появления хотя бы одного из двух совместных событий равна сумме вероятностей этих событий без вероятности их совместного появления:

Р(А+В)=Р(А)+Р(В)-Р(АВ)

 

Пример 106.

Найти вероятность того, что наудачу взятое двузначное число окажется кратным либо 3, либо 5, либо тому и другому одновременно?

Решение:

пусть А – число кратно 3, В – число кратно 5. Всего имеется 90 двузначных чисел: 10, 11, …, 98, 99. Из них 30 – кратные 3, 18 – кратные 5 и шесть чисел одновременно кратны и 3 и 5, поэтому:

.

Т.к. А и В совместные события, то по формуле имеем:

.

 

Задания для самостоятельного решения:

Решите задачи, используя теоремы сложения, умножения вероятностей:

1) В первой урне находятся 10 белых и 4 черных шаров, а во второй 5 белых и 9 черных шаров. Из каждой урны вынули по шару. Какова вероятность того, что оба шара окажутся черными?

2) Трое учащихся на экзамене независимо друг от друга решают одну и ту же задачу. Вероятности ее решения этими учащимися равны 0,8, 0,7 и 0,6 соответственно. Найдите вероятность того, что хотя бы один учащийся решит задачу.

3) Экспедиция издательства отправила газеты в три почтовых отделения. Вероятность своевременной доставки газет в первое отделение равна 0,95, во второе – 0,9, в третье – 0,8. Найти вероятность следующих событий:

а) только одно отделение получит газеты вовремя;

б) хотя бы одно отделение получит газеты с опозданием.

4) Вероятность хотя бы одного попадания в цель при четырех выстрелах равна 0,9984. Найти вероятность попадания в цель при одном выстреле.

 

Контрольные вопросы:

1.   Сформулируйте теоремы сложения, умножения вероятностей.

 

Практическая работа № 37

Понятие о независимости событий

 

Цель работы:

студент должен:

знать:

- определение независимых событий;

- теорему умножения вероятностей;

уметь:

-       вычислять вероятность независимых событий.

 

Сведения из теории:

Пусть вероятность события В не зависит от появления события А.

Событие В называют независимым от события А, если появление события А не изменяет вероятности события В, т. е. если условная вероятность события В равна его безусловной вероятности:

РA(В) = Р(В).

Итак, если событие В не зависит от события A, то событие A не зависит от события В; это означает, что свойство независимости событий взаимно.

Для независимых событий теорема умножения имеет вид:

Р (АВ) = Р (АР (В),

т. е. вероятность совместного появления двух независимых событий равна произведению вероятностей этих событий.

Два события называют независимыми, если вероятность их совмещения равна произведению вероятностей этих событий; в противном случае события называют зависимыми.

На практике о независимости событий заключают по смыслу задачи. Например, вероятности поражения цели каждым из двух орудий не зависят от того, поразило ли цель другое орудие, поэтому события «первое орудие поразило цель» и «второе орудие поразило цель» независимы.

 

Несколько событий называют попарно независимыми, если каждые два из них независимы. Например, события А, В, С попарно независимы, если независимы события А и В, А и С, В и С.

 

Пример 107.

Пусть в урне имеется 4 шара, окрашенные: один – в красный цвет (А), один – в синий цвет (В), один – в черный цвет (С) и один – во все эти три цвета (АВС). Чему равна вероятность того, что извлеченный из урны шар имеет красный цвет?

Решение:

т.к. из четырех шаров два имеют красный цвет, то Р(А)=2/4=1/2.

Рассуждая аналогично, найдем Р(В)=1/2, Р(С)=1/2.

Допустим теперь, что взятый шар имеет синий цвет, т. е. событие В уже произошло. Изменится ли вероятность того, что извлеченный шар имеет красный цвет, т. е. изменится ли вероятность события А?

Из двух шаров, имеющих синий цвет, один шар имеет и красный цвет, поэтому вероятность события А по-прежнему равна 1/2. Другими словами, условная вероятность события А, вычисленная в предположении, что наступило событие В, равна его безусловной вероятности. Следовательно, события А и В независимы.

Аналогично придем к выводу, что события A и С, В и С независимы. Итак, события А, В и С попарно независимы.

Независимы ли эти события в совокупности? Оказывается, нет.

Действительно, пусть извлеченный шар имеет два цвета, например синий и черный. Чему равна вероятность того, что этот шар имеет и красный цвет? Лишь один шар окрашен во все три цвета, поэтому взятый шар имеет и красный цвет.

Т.о., допустив, что события В и С произошли, приходим к выводу, что событие А обязательно наступит. Следовательно, это событие достоверное и вероятность его равна единице.

Другими словами, условная вероятность РBC (A)=1 события А не равна его безусловной вероятности Р(А)=1/2. Итак, попарно независимые события А, В, С не являются независимыми в совокупности.

 

Вероятность совместного появления нескольких событий, независимых в совокупности, равна произведению вероятностей этих событий:

Р(А1А2 ... Аn) = Р(А1Р(А2)·...·Р(Аn).

 

Пример 108.

Найти вероятность совместного появления герба при одном бросании двух монет.

Решение:

вероятность появления герба первой монеты (событие А): Р(А)=1/2.

Вероятность появления герба второй монеты (событие В): Р(В)=1/2.

События А и В независимые, поэтому искомая вероятность по теореме умножения равна:

Р(АВ)=Р(АР(В)=1/2·1/2=1/4.

 

Пример 109.

Имеется 3 ящика, содержащих по 10 деталей. В первом ящике 8, во втором 7 и в третьем 9 стандартных деталей. Из каждого ящика наудачу вынимают по одной детали. Найти вероятность того, что все три вынутые детали окажутся стандартными.

Решение:

вероятность того, что из первого ящика вынута стандартная деталь (событие А):

P(A)=8/10=0,8.

Вероятность того, что из второго ящика вынута стандартная деталь (событие В):

Р(В)=7/10=0,7.

Вероятность того, что из третьего ящика вынута стандартная деталь (событие С):

Р(С)=9/10=0,9.

Так как события А, В и С независимые в совокупности, то искомая вероятность (по теореме умножения) равна:

Р(АВС)=Р(АР(ВР(С)=0,8·0,7·0,9=0,504.

 

Задания для самостоятельного решения:

Решите задачи:

1) Вероятность того, что в магазине будет продана пара мужской обуви 44-го размера, равна 0,12; 45-го – 0,04; 46-го и большего – 0,01. Найти вероятность того, что будет продана пара мужской обуви не меньше 44-го размера.

2) При условиях задачи 1 найти вероятность того, что очередной будет продана пара обуви меньше 44-го размера.

3) В ящике находятся 5 резцов: два изношенных и три новых. Производится два последовательных извлечения резцов. Определить условную вероятность появления изношенного резца при втором извлечении при условии, что извлеченный в первый раз резец в ящик не возвращается.

4) В урне находятся 5 белых шаров, 4 черных и 3 синих. Каждое испытание состоит в том, что наудачу извлекают один шар, не возвращая его в урну. Найти вероятность того, что при первом испытании появится белый шар (событие А), при втором - черный (событие В) и при третьем – синий (событие С).

 

Контрольные вопросы:

1.   Дайте определение независимых событий.

2.   Какие события называются попарно независимыми?

 

Практическое занятие №38

Дискретная случайная величина, закон ее распределения

 

Цель работы:

студент должен:

знать:

- определение дискретной случайной величины;

уметь:

-        строить многоугольник (полигон) распределения дискретной случайной величины;

-        составлять закон распределения дискретной случайной величины.

 

Сведения из теории:

Случайное событие может состоять, в частности, в появлении некоторого числа, значение которого не может быть однозначно определено условиями его возникновение. Такие события называют случайными величинами. В этой трактовке мы сохраняем классический подход к понятию случайного события. Однако требование корректности в построении математических теорий заставляет нас вновь обратиться к аксиоматическому подходу, сохранив классические модели в качестве наглядных образцов из сферы практических приложений.

Математически корректно определить случайную величину как числовую функцию, заданную в пространстве элементарных событий.

 

Предположим вначале, что пространство элементарных событий является конечным множеством. Соответствующую ему случайную величину называют дискретной: она может принимать лишь конечное число значений, каждому из которых может быть сопоставлена вероятность его появления в опыте. Поэтому дискретные случайные величины можно задать таблицей вида:

 

X

х1

х2

хп

P

p1

p2

рп

 

Здесь буквой Х обозначена случайная величина, х1, х2, …, хп – перечень всех ее возможных значений, а p1,…, рп – соответствующие им вероятности. Такую таблицу называют законом распределения дискретной случайной величины.

События Х=хi, (i=1, 2, ,3, …, n) являются несовместными и единственно возможными, т. е. они образуют полную систему событий. Поэтому сумма их вероятностей равна единице: р1+р2+р3+…+рn=1.

 

Пример 110.

Разыгрываются две вещи стоимостью по 5 руб. и одна вещь стоимостью 30 руб. Составить закон распределения выигрышей для человека, купившего один билет из 50.

Решение:

искомая случайная величина Х представляет собой выигрыш и может принимать значения: 0, 5, 30 руб. Первому результату благоприятствует 47 случаев, второму результату – 2 случая и третьему – 1 случай. Найдем их вероятности:

Р(х1)=47/50=0,94;

Р(х2)=2/50=0,04;

Р(х3)=1/50=0,02.

Тогда закон распределения случайной величины имеет вид:

 

Хi

0

5

30

pi

0,94

0,04

0,02

 

В качестве проверки найдем р1+р2+р3=0,94+0,04+0,02=1.

 

Задания для самостоятельного решения:

 

1 вариант

1) Построить многоугольник (полигон) распределения дискретной случайной величины X, заданной законом распределения:

X

2

4

5

6

р

0,3

0,1

0,2

0,4

2) Стрелок делает по мишени три выстрела. Вероятность попадания в мишень при каждом выстреле равна 0,3. Построить ряд распределения числа попаданий.

2 вариант

1) Построить многоугольник (полигон) распределения дискретной случайной величины X, заданной законом распределения:

X

10

15

20

р

0,1

0,7

0,2

2) Составить таблицу распределения вероятностей случайного числа очков, выпавшего на верхней грани игрального кубика при одном подбрасывании.

3 вариант

1) Построить многоугольник (полигон) распределения дискретной случайной величины X, заданной законом распределения:

X

10

20

30

40

р

0,3

0,1

0,2

0,4

2) Игральную кость бросают дважды. Случайная величина Х – сумма очков при обоих подбрасываниях. Составить таблицу распределения вероятностей.

4 вариант

1) Построить многоугольник (полигон) распределения дискретной случайной величины X, заданной законом распределения:

X

5

10415

15

20

р

0,1

0,3

0,2

0,4

2) В коробке находятся 7 карандашей, из которых 4 – красные. Наудачу берут три карандаша. Какой закон распределения имеет случайная величина, означающая число извлеченных красных карандашей?

5 вариант

1) Построить многоугольник (полигон) распределения дискретной случайной величины X, заданной законом распределения:

X

2

4

5

6

р

0,1

0,2

0,5

0,2

2) Составить таблицу распределения вероятностей случайного числа очков, выпавшего на верхней грани игрального кубика при одном подбрасывании.

6 вариант

1) Построить многоугольник (полигон) распределения дискретной случайной величины X, заданной законом распределения:

X

1

2

3

4

р

0,2

0,4

0,1

0,3

2) Стрелок делает по мишени два выстрела. Вероятность попадания в мишень при каждом выстреле равна 0,3. Построить ряд распределения числа попаданий.

7 вариант

1) Построить многоугольник (полигон) распределения дискретной случайной величины X, заданной законом распределения:

X

1

4

7

10

р

0,3

0,4

0,2

0,1

2) В коробке находятся 9 карандашей, из которых 4 – синие. Наудачу берут три карандаша. Какой закон распределения имеет случайная величина, означающая число извлеченных синих карандашей?

8 вариант

1) Построить многоугольник (полигон) распределения дискретной случайной величины X, заданной законом распределения:

X

10

30450

5

р

0,3

0,5

0,2

2) Игральную кость бросают трижды. Случайная величина Х – сумма очков при трех подбрасываниях. Составить таблицу распределения вероятностей.

9 вариант

1) Построить многоугольник (полигон) распределения дискретной случайной величины X, заданной законом распределения:

X

2

4

5

6

р

0,3

0,1

0,2

0,4

2) Стрелок делает по мишени четыре выстрела. Вероятность попадания в мишень при каждом выстреле равна 0,2. Построить ряд распределения числа попаданий.

 

Контрольные вопросы:

1.     Дайте определение случайного события.

2.     Что называется случайной величиной?

3.     Поясните закон распределения дискретной случайной величины.

 

Практическое занятие №40

Вершины, ребра, грани многогранника

 

Цель работы:

студент должен:

знать:

- свойство, связывающее число вершин, ребер и граней многогранника;

уметь:

-       устанавливать связь между числом плоских углов П многогранника и числом его ребер Р.

 

Сведения из теории:

Для выпуклых многогранников имеет место свойство, связывающее число его вершин, ребер и граней, доказанное в 1752 году Леонардом Эйлером, и получившее название теоремы Эйлера.

Прежде чем его сформулировать рассмотрим известные нам многогранники и заполним следующую таблицу, в которой В – число вершин, Р – ребер и Г – граней данного многогранника:

 

Название многогранника

В

Р

Г

Треугольная пирамида

4

6

4

Четырехугольная пирамида

5

8

5

Треугольная призма

6

9

5

Четырехугольная призма

8

12

6

n-угольная пирамида

n+1

2n

+1

n-угольная призма

2n

3n

n+2

n-угольная усеченная пирамида

2n

3n

n+2

 

Из этой таблицы непосредственно видно, что для всех выбранных многогранников имеет место равенство В-Р+Г=2. Оказывается, что это равенство справедливо не только для этих многогранников, но и для про­извольного выпуклого многогранника.

Заметим, что многоугольники можно деформировать, увеличивать, уменьшать или даже искривлять их стороны, лишь бы при этом не происходило разрывов сторон. Число вершин, ребер и граней при этом не изменится.

Для полученного разбиения многоугольника на более мелкие многоугольники имеет место равенство:

В-Р+Г'=1,

где В – общее число вершин, Р – общее число ребер и Г' – число многоугольников, входящих в разбиение. Ясно, что Г'=Г–1, где Г – число граней данного многогранника.

 

В любом выпуклом многограннике найдется грань с числом ребер меньшим или равным пяти

Для любого многогранника имеет место неравенство .

 

Задания для самостоятельного решения:

1) Может ли число вершин многогранника равняться числу его гра­ней?

2) Установите связь между числом плоских углов П многогранника и числом его ребер Р.

3) Гранями выпуклого многогранника являются только треугольники. Сколько у него вершин В и граней Г, если он имеет: а) 12 ребер; б) 15 ребер? Приведите примеры таких многогранников.

4) Из каждой вершины выпуклого многогранника выходит три ребра. Сколько он имеет вершин В и граней Г, если у него: а) 12 ребер; б) 15 ребер? Нарисуйте эти многогранники.

5) В каждой вершине выпуклого многогранника сходится по четыре ребра. Сколько он имеет вершин В и граней Г, если число ребер равно 12? Нарисуйте эти многогранники.

 

Контрольные вопросы:

1.        Запишите формулу, связывающую число вершин, ребер и граней многогранника.

 

Практическое занятие №41

Параллелепипед. Куб

 

Цель работы:

студент должен:

знать:

- определение параллелепипеда, куба;

- свойства прямоугольного параллелепипеда;

- формулы объема прямоугольного параллелепипеда, куба;

уметь:

-       строить параллелепипед, куб;

-       вычислять объем прямоугольного параллелепипеда, куба.

 

Сведения из теории:

Параллелепипедом называется призма, основаниями которой служат параллелограммы. Все шесть граней параллелепипеда – параллелограммы. Отрезки, соединяющие вершины параллелепипеда, не принадлежащие одной и той же грани, называются диагоналями параллелепипеда.

Свойства параллелепипеда

1) Середина диагонали параллелепипеда является его центром симметрии.

2) Противолежащие грани параллелепипеда попарно равны и параллельны.

3) Все четыре диагонали параллелепипеда пересекаются в одной точке и делятся ею пополам.

http://www.fizmatik.ru/images/Geometry/17_1.JPG

Рисунок 46. Параллелепипед

 

Параллелепипед, боковые ребра которого перпендикулярны плоскости основания параллелепипеда, называется прямым параллелепипедом (ABCDA1B1C1D1 – прямой параллелепипед).

http://www.fizmatik.ru/images/Geometry/17_2.JPG

Рисунок 47. Прямоугольный параллелепипед

 

Прямой параллелепипед, основанием которого служит прямоугольник, называется прямоугольным параллелепипедом. Все грани прямоугольного параллелепипеда – прямоугольники. Длины трех ребер прямоугольного параллелепипеда, выходящих из одной вершины, называются измерениями прямоугольного параллелепипеда.

 

Свойства прямоугольного параллелепипеда

1) Квадрат диагонали прямоугольного параллелепипеда равен сумме квадратов трех его измерений:

d2=a2+b2+c2.

2) Все диагонали прямоугольного параллелепипеда равны.

3) Для куба формула упрощается: 4d²=12а².

 

Пример 114.

Найти длину стороны куба, если его диагональ равна 5 см.

Решение:

из формулы для диагонали куба выразим его сторону:

.

Тогда,

 

Т. к. параллелепипед есть частный случай призмы, то площадь поверхности и объем параллелепипеда вычисляются по формулам для площади поверхности и объема призмы. Кроме того, объем прямоугольного параллелепипед можно вычислять по формуле:

V=abc,

где a, b, c – три измерения прямоугольного параллелепипеда.

 

Куб

Прямоугольный параллелепипед с равными измерениями называется кубом. Все грани куба – равные квадраты.

Объем куба вычисляется по формуле:

V=a3,

где a измерение куба.

 

Как найти сумму длин всех рёбер параллелепипеда

Для удобства введем обозначения: А и В стороны основания параллелепипеда; С – его боковая грань.

Т. о., в основании параллелепипеда лежит параллелограмм со сторонами А и В. Параллелограмм – это четырехугольник, противоположные стороны которого равны и параллельны. Из этого определения следует, что против стороны А лежит равная ей сторона А. Поскольку противолежащие грани параллелепипеда равны (вытекает из определения), то верхняя его грань тоже имеет 2 стороны равные А. Таким образом, сумма всех четырех этих сторон равна 4А.

То же, можно сказать, и о стороне В. Противоположная ей сторона в основании параллелепипеда равна В. Верхняя (противолежащая) грань параллелепипеда тоже имеет 2 стороны, равные В. Сумма всех четырех этих сторон равна 4В.

Боковые грани параллелепипеда тоже являются параллелограммами (вытекает из свойств параллелепипеда). Ребро С одновременно является стороной двух соседних граней параллелепипеда. Поскольку противоположные грани параллелепипеда попарно равны, то все его боковые ребра равны между собой и равны С. Сумма боковых ребер – 4С.

Таким образом, сумма всех ребер параллелепипеда: 4А+4В+4С или 4(А+В+С).

Частный случай прямого параллелепипеда – куб. Сумма всех его ребер равна 12А.

 

Пример 115.

Найдите ширину и высоту прямоугольного параллелепипеда, если ширина b больше его длины а на 1 см, высота c в 2 раза больше длины а, а диагональ d в 3 раза больше длины а.

Решение:

запишем основную формулу квадрата диагонали прямоугольного параллелепипеда:

d2=a2+b2+c2.

Выразим все измерения через заданную длину а: b=а+1; c=2а; d=3а.

Подставим в формулу:

9а² =а²+(а+1)²+4а².

Решив квадратное уравнение, найдем длины всех ребер:

3а²–2а–1=0.

а=1; b=2; c=2.

 

Пример 116.

Два ребра прямоугольного параллелепипеда, выходящие из одной вершины, равны 3 и 4. Площадь поверхности этого параллелепипеда равна 94. Найдите третье ребро, выходящее из той же вершины.

Решение:

обозначим известные ребра за а и b, а неизвестное за c. Площадь поверхности параллелепипеда выражается как

Выразим с:

Подставляя заданные значения, имеем:

Ответ: 5.

 

Задания для самостоятельного решения:

Решите задачи:

1) Надо покрасить пол в комнате. Расход краски на 1 м2 – 120 г, комната имеет размеры 5 м и 4 м. Сколько потребуется краски?

2) Надо оклеить комнату с одним окном и дверью обоями от пола до потолка. Длина комнаты 5 м, ширина – 4 м, высота – 3 м. Площадь окна 3 м2, площадь двери 2 м2. Обои продаются целыми рулонами, 1 рулон на 10 м2. Сколько потребуется рулонов обоев?

3) Два ребра прямоугольного параллелепипеда, выходящие из одной вершины, равны 1, 2. Площадь поверхности параллелепипеда равна 16. Найдите его диагональ.

4) Площадь грани прямоугольного параллелепипеда равна 12. Ребро, перпендикулярное этой грани, равно 4. Найдите объем параллелепипеда.

 

Контрольные вопросы:

1.        Дайте определение параллелепипеда, куба, выполните соответствующие чертежи.

2.        Перечислите свойства прямоугольного параллелепипеда.

3.        Запишите формулы для вычисления объема параллелепипеда, куба.

 

Практическое занятие №42

Сечения куба, призмы, пирамиды

 

Цель работы:

студент должен:

знать:

- метод «следов»;

- правила построения сечений многогранников;

уметь:

-       строить сечения куба, призмы, пирамиды.

 

Сведения из теории:

Сечения куба плоскостью

Если плоскость пересекает три ребра куба, выходящих из одной вершины, то в сечении получается треугольник (рис. 48 слева). При этом если отсекаемые плоскостью отрезки ребер равны, то в сечении получается равносторонний треугольник, если равны два отрезка из трех, то получается равнобедренный треугольник, наконец, если все три отрезка различны, то в сечении получается разносторонний треугольник.

 

http://geometry2006.narod.ru/Art/Lecture7.files/image001.gif

Рисунок 48. Сечения куба плоскостью

 

В сечении куба плоскостью не могут получаться прямоугольный или тупоугольный треугольники.

Выясним, какие четырехугольники могут получаться в сечении куба плоскостью.

Ясно, что если плоскость параллельна одной из граней куба, то в сечении получается квадрат (рис. 48 посередине). Если плоскость параллельна одному из ребер куба, то в сечении получается прямоугольник (рис. 48 справа). Если плоскость пересекает четыре параллельных ребра куба, то в сечении получается параллелограмм (рис. 49 слева).

http://geometry2006.narod.ru/Art/Lecture7.files/image002.gif

Рисунок 49. Сечения куба плоскостью

 

На рис. 49 посередине показано сечение куба плоскостью в форме пятиугольника ABCDE. Прямые AB и DE, CD и AE параллельны, как линии пересечения двух параллельных плоскостей третьей плоскостью.

На рис. 49 справа показано сечение куба плоскостью в форме шестиугольника ABCDEF. Прямые AB и DE, BC и EF, CD и AF параллельны, как линии пересечения двух параллельных плоскостей третьей плоскостью.

Поскольку у куба имеется только шесть граней, то в сечении куба плоскостью не может получиться многоугольник с числом сторон, большим шести.

 

Построение сечений многогранников

Для построения сечений используют метод «следов», заключающийся в нахождении точки пересечения прямой и плоскости по заданным двум точкам этой прямой и их проекциям на плоскость.

 

Пример 117.

Пусть прямая k проходит через точки A, B и известны параллельные проекции A, B этих точек на плоскость π. Требуется найти точку пересечения прямой AB с плоскостью π.

Решение:

через точки A, B проведем прямую k. Тогда пересечение прямой k с прямой k и будет искомым пересечением прямой k с плоскостью π (см. рис. 50).

Рисунок 50.

 

Пример 118.

Даны точки A, B, C и их параллельные проекции A, B, C на плоскость π. Требуется построить линию пересечения плоскости ABC и плоскости π.

Решение:

используя решение предыдущей задачи, построим точки X и Y пересечения прямых AB и AC с плоскостью π. Прямая XY будет искомой линией пересечения плоскости ABC и плоскости π (см. рис. 51).

Рисунок 51.

 

Пример 119.

Через данную точку C (C) провести прямую, параллельную данной прямой AB (AB), и найти ее точку пересечения с плоскостью π.

Решение:

через точку C проводим прямую, параллельную AB. Через точку C проводим прямую, параллельную AB. Точка X пересечения этих прямых и будет искомой (см. рис. 52).

Рисунок 52.

 

Используя метод, рассмотренный в примере 119, решим задачи на построение сечений куба, пирамиды и призмы

 

Пример 120.

Построить сечение куба плоскостью проходящей через три точки A, B, C, принадлежащие попарно скрещивающимся ребрам этого куба (см. рис. 53).

http://geometry2006.narod.ru/Art/Lecture7.files/image004.gif

Рисунок 53.

 

Решение:

найдем пересечение прямой AB, лежащей в плоскости сечения, с плоскостью основания куба. Для этого построим параллельные проекцииA’, B точек A, B на основание куба в направлении бокового ребра куба (см. рис. 54).

http://geometry2006.narod.ru/Art/Lecture7.files/image004.gif

Рисунок 54.

 

Пересечение прямых AB и AB будет искомой точкой P. Она принадлежит плоскости сечения и плоскости основания куба. Следовательно, плоскость сечения пересекает основание куба по прямой CP. Точка пересечения этой прямой с ребром основания куба даст еще одну точку D сечения куба. Соединим точки C и D, B и D отрезками. Через точку A проведем прямую, параллельную BD, и точку ее пересечения с ребром куба обозначим E. Соединим точки E и C отрезком. Через точку A проведем прямую, параллельную CD, и точку ее пересечения с ребром куба обозначим F. Соединим точки A и F, B и F отрезками. Многоугольник AECDBF и будет искомым изображением сечения куба плоскостью.

 

 

Пример 121.

Построить сечение треугольной пирамиды плоскостью, проходящей через три точки A, B, C, принадлежащие ее ребрам (см. рис. 55).

http://geometry2006.narod.ru/Art/Lecture7.files/image004.gif

Рисунок 55.

 

Решение:

проведем прямую AB и ее точку пересечения с боковым ребром пирамиды обозначим через E. Проведем прямую EC и ее точку пересечения с ребром основания пирамиды обозначим через D. Соединим отрезками точки B и C, A и D. Четырехугольник ABCD будет искомым сечением пирамиды.

 

Задания для самостоятельного решения:

Решите задачи:

1) Какой фигурой является сечение куба A...D1 плоскостью, проходящей через вершины B1, D и середину ребра CC1?

2) Какой фигурой является сечение куба A...D1 плоскостью, проходящей через середины ребер AB, BC и DD1?

3) Через середину ребра куба, перпендикулярно скрещивающейся с этим ребром диагонали, проведено сечение. Определите его вид.

4) Какой фигурой является сечение куба плоскостью, которая проходит через две противоположные вершины нижнего основания и середину одного из ребер верхнего основания? Найдите его периметр, если длина ребра куба равна 1.

5) Через вершины A, C, D1 куба AD1 проведено сечение. В каком отношении оно делит диагональ DB1, и какой образует угол с этой диагональю?

6) Каким является сечение тетраэдра ABCD плоскостью, проходящей через середины ребер AB, BC и CD?

7) Какой фигурой является сечение правильного тетраэдра ABCD плоскостью, проходящей через вершину B и точки M, Nсередины соответственно ребер AD, CD?

8) Постройте сечение куба A...D1 плоскостью, проходящей через вершины B1, D и точку H, принадлежащую ребру CC1.

9) Постройте сечение правильной четырехугольной пирамиды плоскостью, проходящей через точки, указанные на рисунке 56.

http://geometry2006.narod.ru/Art/Lecture7.files/image010.gif

Рисунок 56.

 

10) Постройте сечение правильной шестиугольной призмы плоскостью, проходящей через точки, указанные на рисунке 57.

http://geometry2006.narod.ru/Art/Lecture7.files/image010.gif

Рисунок 57.

 

Контрольные вопросы:

1.        Может ли в сечении куба плоскостью получиться: а) трапеция; б) равнобедренная трапеция; в) неравнобедренная трапеция; г) прямоугольная трапеция; д) тупоугольная трапеция?

2.        Какие многоугольники можно получить в сечении четырехугольной пирамиды плоскостью?

3.        Какие могут быть сечения правильного тетраэдра плоскостью?

 

Практическое занятие №43

Представление о правильных многогранниках (тетраэдр, куб, октаэдр, додекаэдр, икосаэдр)

 

Цель работы:

студент должен:

знать:

- определение правильных многогранников;

- виды, элементы, свойства правильных многогранников;

уметь:

-       строить правильные многогранники.

 

Сведения из теории:

Выпуклый многогранник называется правильным, если его гранями являются равные правильные многоугольники, и все многогранные углы равны.

Рассмотрим возможные правильные многогранники и прежде всего те из них, гранями которых являются правильные треугольники. Наиболее простым таким правильным многогранником является треугольная пирамида, гранями которой являются правильные треугольники (рис. слева). В каждой ее вершине сходится по три грани. Имея всего четыре грани, этот многогранник называется также правильным тетраэдром, или просто тетраэдром, что в переводе с греческого языка означает четырехгранник.

http://geometry2006.narod.ru/Art/Lecture6.files/image010.gif

Рисунок 58. Правильные многогранники

 

Многогранник, гранями которого являются правильные треугольники, и в каждой вершине сходится четыре грани, изображен на рисунке посередине. Его поверхность состоит из восьми правильных треугольников, поэтому он называется октаэдром.

Многогранник, в каждой вершине которого сходится пять правильных треугольников, изображен на рисунке справа. Его поверхность состоит из двадцати правильных треугольников, поэтому он называется икосаэдром.

Заметим, что поскольку в вершинах выпуклого многогранника, не может сходиться более пяти правильных треугольников, то других правильных многогранников, гранями которых являются правильные треугольники, не существует.

Аналогично, поскольку в вершинах выпуклого многогранника может сходиться только три квадрата, то, кроме куба (рис. слева), других правильных многогранников, у которых гранями являются квадраты, не существует. Куб имеет шесть граней и поэтому называется также гексаэдром.

http://geometry2006.narod.ru/Art/Lecture6.files/image011.gif

Рисунок 59. Правильные многогранники

 

Многогранник, гранями которого являются правильные пятиугольники, и в каждой вершине сходится три грани, изображен на рисунке справа. Его поверхность состоит из двенадцати правильных пятиугольников, поэтому он называется додекаэдром.

 

Задания для самостоятельного решения:

1) Чему равны плоские углы додекаэдра?

2) Представьте многогранник – бипирамиду, сложенную из двух правильных тетраэдров совмещением их оснований. Будет ли он правильным многогранником?

3) Является ли пространственный крест (фигура, составленная из семи равных кубов – рис. 60) правильным многогранником? Сколько квадратов ограничивает его поверхность? Сколько у него вершин В и ребер Р?

http://geometry2006.narod.ru/Art/Lecture6.files/image022.gif

Рисунок 60.

 

4) Ребро октаэдра равно 1. Определите расстояние между его противоположными вершинами.

5) Сколько красок потребуется для раскраски граней правильных многогранников, так, чтобы соседние грани были окрашены в разные цвета?

 

Контрольные вопросы:

1.        Дайте определение правильного многогранника.

2.        Сколько вершин, ребер и граней имеют: а) тетраэдр; б) октаэдр; в) куб; г) икосаэдр; д) додекаэдр?

 

Практическое занятие №44

Осевые сечения и сечения параллельные основанию

 

Цель работы:

студент должен:

знать:

- свойства проекций;

уметь:

-       строить сечения цилиндра.

 

Сведения из теории:

Сечения цилиндра

Сечения цилиндра плоскостью можно рассматривать как параллельные проекции основания цилиндра на эту плоскость. Поэтому, если плоскость параллельна плоскости основания, то в сечении получается круг, равный основанию.

Если плоскость сечения составляет некоторый угол с плоскостью основания цилиндра и не пересекает основания, то в сечении будет фигура, ограниченная эллипсом.

 

http://geometry2006.narod.ru/Art/Lecture7.files/image015.gif

Рисунок 61. Сечение цилиндра

 

На рис. 61 показано построение точек эллипса, получающегося как сечение боковой поверхности цилиндра плоскостью.

Для этого зададим два сопряженных диаметра AB и CD. Через точку A проведем образующую и выберем на ней какую-нибудь точку A, принадлежащую сечению. Прямая AO пересечет образующую, проходящую через точку B в некоторой точке B, также принадлежащую сечению. Возьмем теперь на отрезке CD произвольную точку и проведем через нее прямую, параллельную AB. Ее точки пересечения с образующими цилиндра будут принадлежать сечению.

Возьмем прямоугольный лист бумаги и нарисуем на нем оси координат Ox и Oy параллельно соответствующим сторонам (рис. 62).

http://geometry2006.narod.ru/Art/Lecture7.files/image018.gif

Рисунок 62.

 

Затем свернем этот лист в боковую поверхность прямого кругового цилиндра, радиус основания которого примем за единицу. Ось Ox свернется в окружность радиуса 1, а ось Oy станет образующей цилиндра (рис. 63).

http://geometry2006.narod.ru/Art/Lecture7.files/image018.gif

Рисунок 63.

 

Через диаметр OD полученной окружности проведем сечение, составляющее с плоскостью окружности угол в 300. В этом случае сечением будет эллипс.

 

Задания для самостоятельного решения:

1) Нарисуйте цилиндр и плоскость, пересекающую его боковую поверхность по эллипсу.

2) Нарисуйте цилиндр и постройте несколько точек эллипса, получающегося в сечении его боковой поверхности плоскостью.

3) В основании цилиндра круг радиуса 5 см. Боковая поверхность цилиндра пересечена плоскостью. Найдите площадь сечения цилиндра этой плоскостью, если она образует с плоскостью основания угол: а) 30°; б) 45°; в) 60°.

4) Возьмем прямоугольный лист бумаги с нарисованными на нем осями координат. Свернем этот лист в боковую поверхность правильной четырехугольной призмы (рис. 64). Сторону основания призмы примем за 1 см. Через точки О и D проведем сечение плоскостью, составляющей с плоскостью основания угол 45°. Развернем лист бумаги. Выясните, какая при этом получится кривая? Что изменится, если сечение проводить под другими углами?

http://geometry2006.narod.ru/Art/Lecture7.files/image045.gif

Рисунок 64.

 

Контрольные вопросы:

1.        В каком случае сечением цилиндра плоскостью является круг?

2.        Что будет сечением цилиндра плоскостью, проходящей через ось цилиндра?

3.        Какую форму принимает поверхность воды в круглом наклоненном стакане?

4.        Может ли в сечении цилиндра плоскостью получиться: а) круг; б) прямоугольник; в) параллелограмм; г) трапеция д) треугольник?

5.        Могут ли в сечениях цилиндра плоскостью получаться фигуры, отличные от круга, прямоугольника и эллипса?

 

Практическое занятие №45

Шар и сфера, их сечения

 

Цель работы:

студент должен:

знать:

- определение шара, сферы, их элементов;

уметь:

-       строить сечения шара.

 

Сведения из теории:

Шаром принято называть тело, ограниченное сферой, т.е. шар и сфера – это разные геометрические тела.

Сфера – это фигура, состоящая из всех точек пространства, удалённых от данной точки на данном расстоянии.

Рисунок 65. Сфера

 

Поверхность шара называют сферой. Если рассечь сферу плоскостью, получим в сечении окружность. Такие окружности имеют разные радиусы: чем дальше плоскость от центра сферы, тем меньше радиус сечения. Самые большие окружности получаются при сечении сферы плоскостями, проходящими через её центр. Такими большими окружностями на земной поверхности являются экватор и меридианы. А параллели это сечения земной поверхности плоскостями, которые параллельны экваториальной плоскости.

Сферой называется фигура, состоящая из всех точек пространства, равноудалённых от данной точки. Эта точка называется центром сферы и обычно обозначается О.

Расстояние от точек сферы до её центра называется радиусом сферы и обычно обозначается R. Радиусом также называется любой отрезок, соединяющий точку сферы с её центром. Сфера это граница шара. Центр, радиус и диаметр сферы являются также центром, радиусом и диаметром шара.

Шаром называется тело, которое состоит из всех точек пространства, находящихся на расстоянии не более чем на данное расстояние. Другими словами, шар – это объединение сферы и всех ее внутренних точек.

Всякое сечение шара плоскостью есть круг. Центр этого круга есть основание перпендикуляра, опущенного из центра шара на секущую плоскость.

 

Основные геометрические формулы

Площадь сферы:

S=4πr2=πd2.

Объем шара, ограниченного сферой:

 

Рисунок 66. Взаимное расположение сферы и плоскости

 

Касательная плоскость к сфере

Плоскость, имеющая со сферой только одну общую точку, называется касательной плоскостью к сфере, а их общая точка называется точкой касания плоскости и сферы.

Радиус сферы, проведённый в точку касания сферы и плоскости, перпендикулярен к касательной плоскости.

Рисунок 67. Касательная плоскость к сфере

 

Если радиус сферы перпендикулярен к плоскости, проходящей через его конец, лежащий на сфере, то эта плоскость является касательной к сфере.

 

Сечение шара

Всякое сечение шара плоскостью есть круг. Центр этого круга есть основание перпендикуляра, опущенного из центра шара на секущую плоскость.

Рисунок 68. Сечение шара

 

Пример 122.

Два сечения шара радиуса 10 см параллельными плоскостями имеют радиусы, равные 6 см и 8 см. Найти расстояние между секущими плоскостями.

Решение:

находим расстояние каждой из параллельных плоскостей до центра шара из прямоугольных треугольников по теореме Пифагора:

 см

или

 см.

В зависимости от того, лежит ли центр шара между плоскостями или нет, получаем два различных ответа к задаче:

d=14 см.

 

Пример 123.

Через середину радиуса шара проведена перпендикулярная ему плоскость. Как относится площадь полученного сечения к площади большого круга (рис. 69)?

Рисунок 69.

Решение:

отношение площади круга к площади полученного сечения равно:

 

Задания для самостоятельного решения:

Решите задачи:

1) Шар, радиус которого равен 41 дм, пересечен плоскостью на расстоянии 9 дм от центра. Вычислите площадь получившегося сечения.

2) Через середину радиуса шара проведена перпендикулярная к нему плоскость. Как относится площадь полученного сечения к площади большого круга?

3) Радиус шара равен 63см. Точка находится на касательной плоскости на расстоянии 16 см от точки касания. Найти ее кратчайшее расстояние от поверхности шара.

4) Радиус шара R. Через конец радиуса проведена плоскость под углом в 600 к нему. Найти площадь сечения.

5) На поверхности шара даны три точки. Прямолинейные расстояния между ними: 6 см, 8 см и 10 см. Радиус шара равен 13см. Радиус шара равен 13см. Найти расстояние от центра шара до плоскости, проходящей через эти три точки.

 

Контрольные вопросы:

1.        Дайте определение шара, сферы.

2.        Запишите формулы площади сферы, объема шара.

3.        Приведите примеры взаимного расположения сферы и плоскости.

 

Практическое занятие № 46

Способы задания последовательностей, свойства числовых последовательностей

 

Цель работы:

студент должен:

знать:

-       способы задания последовательностей;

-       свойства числовых последовательностей;

уметь:

-       вычислять члены последовательностей по общему члену;

-       задавать формулой общий член последовательности.

 

Сведения из теории:

Числовая последовательностьфункция вида y=f(x), xЄN, где N – множество натуральных чисел (или функция натурального аргумента), обозначается y=f(n) или y1, y2, …, yn, ….

Значения y1, y2, y3,…называют соответственно первым, вторым, третьим, … членами последовательности.

 

Пример 63.

Вычислить первые три значения для функции y=n2.

Решение:

подставляя в y=n2 значения n=1, n=2, n=3 получим первые три значения функции:

y1=12=1;

y2=22=4;

y3=32=9.

 

Способы задания последовательностей

Последовательности можно задавать различными способами, среди которых особенно важны три: аналитический, описательный и рекуррентный.

1. Последовательность задана аналитически, если задана формула ее n-го члена: yn=f(n).

Например, yn=2n–1 – последовательность нечетных чисел: 1, 3, 5, 7, 9, …

2. Описательный способ задания числовой последовательности состоит в том, что объясняется, из каких элементов строится последовательность.

Например, «Все члены последовательности равны 1». Это значит, речь идет о стационарной последовательности 1, 1, 1, …, 1, ….

Или, например, «Последовательность состоит из всех простых чисел в порядке возрастания». Таким образом, задана последовательность 2, 3, 5, 7, 11, …. При таком способе задания последовательности в данном примере трудно ответить, чему равен, скажем, 1000-й элемент последовательности.

3. Рекуррентный способ задания последовательности состоит в том, что указывается правило, позволяющее вычислить n-й член последовательности, если известны ее предыдущие члены.

Например, y1=3; yn=yn–1+4, если n=2, 3, 4,….

Здесь y1=3; y2=3+4=7; y3=7+4=11; ….

Можно видеть, что полученная в этом примере последовательность, может быть задана и аналитически: yn=4n–1.

 

Пример 64.

Вычислить следующие четыре члена последовательности y1=1; y2=1; yn=yn–2+yn–1.

Решение:

из формулы yn=yn–2+yn–1 видно, что каждый следующий член последовательности равен сумме двух предыдущих, поэтому:

y1=1; y2=1; y3=1+1=2; y4=1+2=3; y5=2+3=5; y6=3+5=8.

 

Последовательность, составленную в этом примере, специально изучают в математике, поскольку она обладает рядом интересных свойств и приложений. Ее называют последовательностью Фибоначчи – по имени итальянского математика 13в. Задать формулой последовательность Фибоначчи рекуррентно очень легко, а аналитически – очень трудно. n-е число Фибоначчи выражается через его порядковый номер следующей формулой:

.

 

Свойства числовых последовательностей

Числовая последовательность – частный случай числовой функции, поэтому ряд свойств функций рассматриваются и для последовательностей.

Последовательность {yn} называют возрастающей, если каждый ее член (кроме первого) больше предыдущего:

y1<y2<y3< … <yn<yn+1< ….

Последовательность {yn} называют убывающей, если каждый ее член (кроме первого) меньше предыдущего:

y1 > y2 > y3 > … > yn > yn+1 > ….

Возрастающие и убывающие последовательности объединяют общим термином – монотонные последовательности.

 

Например, y1=1; yn=n2 – возрастающая последовательность, а y1=1;  – убывающая последовательность.

 

Последовательность называется периодической, если существует такое натуральное число T, что начиная с некоторого n, выполняется равенство yn=yn+T. Число T называется длиной периода.

Например, последовательность yn=(-1)n периодична с длиной периода T=2.

 

Задания для самостоятельного решения:

1 вариант

1) Найдите первые пять членов последовательности, и определите ее вид по его заданному общему члену:

2) Найдите n-й член последовательности по ее данным первым членам:

2 вариант

1) Найдите первые пять членов последовательности, и определите ее вид по его заданному общему члену:

2) Найдите n-й член последовательности по ее данным первым членам:

3 вариант

1) Найдите первые пять членов последовательности, и определите ее вид по его заданному общему члену:

2) Найдите n-й член последовательности по ее данным первым членам:

4 вариант

1) Найдите первые пять членов последовательности, и определите ее вид по его заданному общему члену:

2) Найдите n-й член последовательности по ее данным первым членам:

5 вариант

1) Найдите первые пять членов последовательности, и определите ее вид по его заданному общему члену:

2) Найдите n-й член последовательности по ее данным первым членам:

6 вариант

1) Найдите первые пять членов последовательности, и определите ее вид по его заданному общему члену:

2) Найдите n-й член последовательности по ее данным первым членам:

7 вариант

1) Найдите первые пять членов последовательности, и определите ее вид по его заданному общему члену:

2) Найдите n-й член последовательности по ее данным первым членам:

8 вариант

1) Найдите первые пять членов последовательности, и определите ее вид по его заданному общему члену:

2) Найдите n-й член последовательности по ее данным первым членам:

9 вариант

1) Найдите первые пять членов последовательности, и определите ее вид по его заданному общему члену:

2) Найдите n-й член последовательности по ее данным первым членам:

 

Контрольные вопросы:

1.     Что называется числовой последовательностью?

2.     Перечислите способы задания последовательностей.

3.     Перечислите свойства числовых последовательностей.

Практическое занятие № 47

Вычисление пределов функции в точке, на бесконечности

 

Цель работы:

студент должен:

знать:

-       определение предела функции;

-       свойства и правила вычисления пределов функции;

уметь:

-       вычислять пределы функции в точке, на бесконечности.

 

Сведения из теории:

Предел функции

Число А называют пределом функции f(x) в точке а если при ха, f(x) →А.

 

Бесконечно малые и бесконечно большие функции

Функция f(x) называется бесконечно малой при ха, если

Функция f(x) называется бесконечно большой при ха, если

 

Свойства бесконечно малых и бесконечно больших функций

Если функции f(x) и g(x) бесконечно малые при ха, то (f(x)+g(x)) бесконечно малая при ха.

Если функция f(x) бесконечно малая при ха и g(x) – ограниченная, то  – бесконечно малая.

Если существует , а g(x) – бесконечно большая при ха, то ; .

Если при ха, f(x) – бесконечно малая, то  – бесконечно большая.

Если при ха, f(x) – бесконечно большая, то  – бесконечно малая.

 

Теоремы о пределах

Если существуют пределы функций f(x) и g(x), то существует предел суммы (разности) этих функций, который равен сумме (разности) пределов функций f(x) и g(x):

.

Если существуют пределы функций f(x) и g(x), то существует предел произведения этих функций, который равен произведению пределов этих функций:

.

Если существуют пределы функций f(x) и g(x) при ха и предел g(x)≠0, то существует предел частного этих функций, который равен отношению их пределов:

.

Следствие: постоянный множитель можно вынести за знак предела:

.

 

Пример 65.

Вычислить предел .

Решение:

здесь применима теорема о пределе частного.

Разложим на множители квадратный трехчлен, для этого достаточно найти корни х1 и х2 квадратного уравнения ах2++с=а(хх1)·(хх2):

9х2+8х–1=9·(х-)·(х+1).

Под знаком предела сократим одинаковые множители и перейдем к пределу:

 

Пример 66.

Вычислить предел

Решение.

обнаружив неопределенность , раскладываем многочлены в числителе и в знаменателе на множители:

.

Числитель дроби стремится к конечному пределу, равному 3, а знаменатель при х®1 является бесконечно малой, тогда дробь при х®1 является бесконечно большой.

 

Для раскрытия неопределенности  следует числитель и знаменатель разделить на одну и ту же старшую степень переменной.

 

Пример 67.

Вычислить предел .

Решение:

в заданном пределе  числитель и знаменатель не имеют конечных пределов, имеем неопределенность . Поделив одновременно числитель и знаменатель на х3, получим

,

т. к. каждая из дробей  является бесконечно малой и стремится к нулю.

 

Задания для самостоятельного решения:

Вычислите пределы:

1 вариант

2 вариант

3 вариант

4 вариант

5 вариант

6 вариант

7 вариант

8 вариант

9 вариант

1) ;

2) ;

3) ;

4) .

 

Контрольные вопросы:

1.   Что называется пределом функции в точке.

2.   Сколько пределов может иметь функция в точке?

3.   Сформулируйте теоремы о пределах.

 

Практическое занятие № 48

Правила вычисления производных

 

Цель работы:

студент должен:

знать:

-       систему и определение производной;

-       табличные решения производных элементарных функций, в том числе обратных тригонометрических функций;

-       правила дифференцирования функций;

уметь:

-     находить производную функции;

-     находить дифференциал функции;

-     дифференцировать элементарные функции.

 

Сведения из теории:

Табличные значения производных элементарных функций, тригонометрических и обратных тригонометрических функций:

 

Правила вычисления производных:

 

Пример 68.

Вычислите производную функции .

Решение:

воспользуемся формулами и правилом 1 вычисления производных:

 

Пример 69.

Вычислите производную функции

Решение:

воспользуемся формулами и правилом 2 вычисления производных:

Приведем дроби к общему знаменателю:

 

Задания для самостоятельного решения:

Вычислите производную функции:

1 вариант

1) ;

2) ;

3) ;

4) .

2 вариант

1) ;

2) ;

3) ;

4).

3 вариант

1) ;

2) ;

3) ;

4).

4 вариант

1) ;

2) ;

3) ;

4).

5 вариант

1) ;

2) ;

3) ;

4).

6 вариант

1) ;

2) ;

3) ;

4) .

7 вариант

1) ;

2) ;

3) ;

4).

8 вариант

1) ;

2) ;

3) ;

4).

9 вариант                                         3) ;

1) ;                   4).

2) ;

 

Контрольные вопросы:

1.   Перечислите значения производных некоторых табличных функций.

2.   Сформулируйте правила вычисления производных.

 

Практическое занятие № 49

Вычисление производных сложной функции

 

Цель работы:

студент должен:

знать:

-       систему и определение производной, второй производной и производных высших порядков;

-       табличные решения производных элементарных функций, в том числе обратных тригонометрических функций;

-       правила вычисления производной сложной функции;

уметь:

-     находить производную сложной функции;

-     находить вторую производную и производную высших порядков.

 

Сведения из теории:

Производная сложной функции

Пусть функция , х(а;b), имеет производную в точке х0(а;b), а функция  имеет производную в точке . Тогда сложная функция  имеет производную в точке х0, которая вычисляется по формуле:

.

 

Пример 70.

Вычислите производную функции .

Решение:

представим заданную функцию как композицию квадратичной функции и степенной

;

 

Производные высших порядков

Вторая производная это производная от первой производной, т.е. , и т.д.

Производные высших порядков обозначаются римскими цифрами.

 

Пример 71.

Найти четвертую производную .

Решение:

вычисляем последовательно производные:

 

Задания для самостоятельного решения:

Вычислите значение «сложной» производной в указанной точке:

1 вариант

1)

2)

3)

4)

5)

2 вариант

1)

2)

3)

4)

5)

3 вариант

1)

2)

3)

4)

5)

4 вариант

1)

2)

3)

4)

5)

5 вариант

1)

2)

3)

4)

5)

6 вариант

1)

2)

3)

4) ;

5)

7 вариант

1)

2)

3)

4)

5)

8 вариант

1)

2)

3)

4)

5)

9 вариант

1)                      2)

3)                       4)

5)

 

Контрольные вопросы:

1.   Сформулируйте правила вычисления производных сложной функции.

2.   Что называется второй производной данной функции?

 

Практическое занятие № 50

Нахождение наименьшего, наибольшего значения функции на отрезке

 

Цель работы:

студент должен:

знать:

-       определение точек максимума (минимума) функции;

-       зависимость поведения функции от знака первой производной;

уметь:

-  применять первую производную для нахождения промежутков монотонности функции;

-  находить наименьшее, наибольшее значение функции на отрезке.

 

Сведения из теории:

Для нахождения наименьшего и наибольшего значений функции, непрерывной в некотором промежутке, необходимо:

1) найти критические точки, принадлежащие заданному промежутку, и вычислить значения функции в этих точках;

2) найти значения функции на концах промежутка;

3) сравнить полученные значения; тогда наименьшее и наибольшее из них являются соответственно наименьшим и наибольшим значениями функции в рассматриваемом промежутке.

 

Пример 72.

Найдите наибольшее и наименьшее значения функции  на промежутке [-2; 0].

Решение:

вычислим критические точки функции, принадлежащие заданному промежутку, с помощью первой производной:

Т.к. -3[-2; 0], х=-1 – критическая точка.

, .

Вычислим значения функции на концах промежутка:

, .

у(0)=0.

Сравним полученные значения: наименьшее значение функции равно  и достигается ею во внутренней точке промежутка, а наибольшее значение равно 0 и достигается на правом конце промежутка.

 

Задания для самостоятельного решения:

Найдите наименьшее и наибольшее значения функций в заданных промежутках:

1)  на промежутке [0; 6];

2)  на промежутке [-2; 2];

3)  на промежутке [1; 3];

4)  на промежутке [-1; 6];

5)  на промежутке [-4; 4];

6)  на промежутке [0; 3];

7)  на промежутке [-4; -1];

8)  на промежутке [-3; 1];

9)  на промежутке [-5; 0].

 

Контрольные вопросы:

1.   Сформулируйте правила вычисления наименьшего и наибольшего значения функции на промежутке.

 

Практическое занятие № 50

Построение графиков функций

 

Цель работы:

студент должен:

знать:

-       общую схему построения графиков функций;

уметь:

-       исследовать функцию с помощью первой, второй производной;

-       строить графики функций.

 

Сведения из теории:

Общая схема построения графиков функций:

1)       найти область определения функции;

2)       найти точки пересечения графика функции с осями координат;

3)       найти промежутки монотонности функции и экстремумы функции;

4)       найти промежутки выпуклости и точки перегиба;

5)       построить график функции, используя полученные результаты исследования.

 

Пример 73.

Исследовать функцию  и построить ее график.

Решение:

1) Данная функция является многочленом (можно раскрыть скобки, получим многочлен третьей степени), поэтому она определена, непрерывна и дифференцируема при любых х. Поэтому область определения функции – вся числовая прямая.

2) Вычислим точки пересечения графика функции с осями координат: график функции у=(х+1)·(х–2)2 пересекает ось Ох при у=0, т. е.

(х+1)·(х–2)2=0;

х+1=0 или (х–2)2=0;

х=-1 или х=2.

График функции у=(х+1)·(х–2)2 пересекает ось Оу при х=0, т. е.

у=(0+1)·(0–2)2=1·4=4.

Т.о. мы получили три точки: (–1; 0), (2; 0), (0; 4).

3) Найдем промежутки монотонности функции и ее экстремумы с помощью первой производной:

у’=((х+1)·(х–2)2)=3х·(х–2).

Из уравнения у¢=0 найдем критические точки:

3х·(х–2)=0;

х1=0, х2=2.

Результаты решения занесем в таблицу:

х

(–∞, 0)

0

(0; 2)

2

(2; +∞)

у¢

+

0

0

+

у

4

0

возрастает

max

убывает

min

возрастает

 

Функция возрастает на интервалах (–∞, 0) и (2, +∞), убывает на интервале (0; 2), имеет максимум при х=0 и минимум при х=2: уmax=у(0)=4; уmin=у(2)=0.

4) Найдем промежутки выпуклости и точки перегиба с помощью второй производной:

у¢¢=(3х·(х–2))’=6·(х-1).

Кривая выпукла там, где у¢¢ < 0, т. е.

6·(х–1) < 0,

х < 1.

Кривая вогнута там, где у¢¢ > 0, т. е. х > 1.

На интервале (–∞, 1) кривая выпукла; на интервале (1, +∞) – вогнута.

Точку перегиба найдем из уравнения у¢¢=0. Т. о., х=1 – абсцисса точки перегиба, т.к. эта точка разделяет интервалы выпуклости и вогнутости кривой. Ордината точки перегиба: у(1)=2.

Результаты решения занесем в таблицу:

х

(–∞, 1)

1

(1; +∞)

у¢¢

-

0

+

у

2

выпукла

перегиб

вогнута

5) По полученным точкам строим график:

Рисунок 23. График функции у=(х+1)·(х–2)2

 

Задания для самостоятельного решения:

Исследуйте следующие функции и постройте их графики:

1 вариант

.

2 вариант

.

3 вариант

.

4 вариант

.

5 вариант

.

6 вариант

.

7 вариант

.

8 вариант

.

9 вариант

.

 

Контрольные вопросы:

1.     Что называется областью определения и областью значений функции?

2.       Приведите примеры применения первой производной к исследованию функции.

3.      Приведите примеры применения второй производной к исследованию функции.

4.      Расскажите общую схему исследования и построения графика функции.

 

Практическое занятие № 51

Вычисление неопределенных интегралов методом замены переменной

 

Цель работы:

студент должен:

знать:

-       таблицу значений неопределенных интегралов;

-       суть метода замены переменной в неопределенном интеграле;

уметь:

-       вычислять неопределенные интегралы методом замены переменной.

 

Сведения из теории:

Табличные значения неопределенных интегралов

 

Интегрирование методом замены переменной

Сущность интегрирования методом замены переменной (способом подстановки) заключается в преобразовании интеграла  в интеграл , который легко вычисляется по таблице значений неопределенных интегралов.

Для нахождения интеграла  заменяем переменную x новой переменной t. Дифференцируя равенство, получаем выражение dх.

После того как интеграл относительно новой переменной t будет найден, с помощью обратной подстановки он приводится к переменной х.

 

Пример 74.

Вычислите интеграл методом замены переменной: .

Решение:

с помощью замены части подынтегрального выражения приведем заданный интеграл к табличному виду:

.

 

Пример 75.

Вычислите интеграл методом замены переменной: .

Решение:

с помощью замены части подынтегрального выражения приведем заданный интеграл к табличному виду:

 

.

 

Задания для самостоятельного решения:

Вычислите следующие интегралы методом замены переменной:

1 вариант

1) ;

2) ;

3) ;

4) .

2 вариант

1) ;

2) ;

3) ;

4) .

3 вариант

1) ;

2) ;

3) ;

4) .

4 вариант

1) ;

2) ;

3) ;

4) .

5 вариант

1) ;

2) ;

3) ;

4) .

6 вариант

1) ;

2) ;

3) ;

4) .

7 вариант

1) ;

2) ;

3) ;

4) .

8 вариант

1) ;

2) ;

3) ;

4) .

9 вариант

1) ;

2) ;

3) ;

4) .

 

Контрольные вопросы:

1.   Какая функция называется первообразной для функции f(x), при x (a; b)?

2.   Что называется неопределенным интегралом?

3.   Перечислите основные формулы интегрирования.

4.   Сформулируйте суть метода непосредственного интегрирования.

5.   Сформулируйте суть метода замены переменной.

 

Практическое занятие № 52

Вычисление неопределенных интегралов методом интегрирования по частям

 

Цель работы:

студент должен:

знать:

-       таблицу значений неопределенных интегралов;

-       суть метода интегрирования по частям;

уметь:

-       вычислять неопределенные интегралы методом интегрирования по частям.

 

Сведения из теории:

Интегрирование по частям

Вычисляя дифференциал произведения, имеем:

,

откуда

.

Если дифференциалы двух функций равны, то их неопределенные интегралы совпадают. Поэтому

и, следовательно,

.

С помощью этой формулы вычисление интеграла  сводится к вычислению интеграла , если последний окажется проще исходного.

 

Пример 76.

Вычислите интеграл методом интегрирования по частям: .

Решение:

преобразуя части подынтегрального выражения, приведем заданный интеграл к табличному виду:

 

.

 

Пример 77.

Вычислите интеграл методом интегрирования по частям: .

Решение:

преобразуя части подынтегрального выражения, приведем заданный интеграл к табличному виду:

.

 

Задания для самостоятельного решения:

Вычислите следующие интегралы методом интегрирования по частям:

1 вариант

1) ;

2) ;

3) .

2 вариант

1) ;

2) ;

3) .

3 вариант

1) ;

2) ;

3) .

4 вариант

1) ;

2) ;

3) .

5 вариант

1) ;

2) ;

3) .

6 вариант

1) ;

2) ;

3) .

7 вариант

1) ;

2) ;

3) .

8 вариант

1) ;

2) ;

3) .

9 вариант

1) ;

2) ;

3) .

 

Контрольные вопросы:

1.   Сформулируйте суть метода интегрирования по частям.

 

Практическое занятие №53

Вычисление объемов тел и поверхностей вращения

 

Цель работы:

студент должен:

знать:

- формулы объемов тел и поверхностей вращения;

уметь:

-       вычислять объемы тел и поверхностей вращения.

 

Сведения из теории:

Объем прямоугольного параллелепипеда

V=abc,

где a, b, c – стороны параллелепипеда.

 

Объем куба

V=a3,

где a – длина грани куба.

 

Объем призмы

призма

Рисунок 70. Призма

 

Объем призмы равен произведению площади основания призмы, на высоту:

V=Soh,

где So – площадь основания призмы, h – высота призмы.

 

Объем параллелепипеда

параллелепипед

Рисунок 71. Параллелепипед

 

Объем параллелепипеда равен произведению площади основания на высоту:

V=So·h,

где So – площадь основания, h – длина высоты.

 

Объем прямоугольного параллелепипеда

прямоугольного параллелепипед

Рисунок 72. Прямоугольный параллелепипед

 

Объем прямоугольного параллелепипеда равен произведению его длины, ширины и высоты:

V=a·b·h,

где a – длина, b – ширина, h – высота.

 

Объем пирамиды

пирамида

Рисунок 73. Пирамида

 

Объем пирамиды равен трети от произведения площади ее основания на высоту:

,

где So – площадь основания пирамиды, h – длина высоты пирамиды.

 

Объем правильного тетраэдра

правильный тетраэдр

Рисунок 74. Тетраэдр

 

где a – длина ребра правильного тетраэдра.

 

Объем цилиндра

цилиндр

Рисунок 75. Цилиндр

 

Объем цилиндра равен произведению площади его основания на высоту:

V=πR2h

или

V=Soh,

где So – площадь основания цилиндра, R – радиус цилиндра, h – высота цилиндра, π=3,141592.

 

Объем конуса

конус

Рисунок 76. Конус

 

Объем конуса равен трети от произведения площади его основания на высоту:

или

,

где So – площадь основания конуса, R – радиус основания конуса, h – высота конуса, π=3,141592.

 

Объем шара

,

где R – радиус шара, π=3,141592.

 

Пример 124.

Если каждое ребро куба увеличить на 2 см, то его объем увеличится на 98 см3. Определите ребро куба.

Решение:

обозначим ребро куба за х и составим уравнение:

(х+2)3=х3+98,
х3+6х2+12х+8=х3+98,
6х2+12х-90=0,
х2+2х-15=0,
х1=-5, х2=3.

х1=-5 – не удовлетворяет условию задачи.

Ответ: 3.

 

Пример 125.

Прямоугольный лист жести, имеющий 1,6 м длины и 0,8 м ширины, можно согнуть в трубку двояким образом: в первом случае длина трубки будет 1,6 м, во втором 0,8 м. Найти отношение объемов трубок.

Решение:

трубки образуют цилиндры, объем, которого вычисляется по формуле:

VR2h.

У первого цилиндра высота будет 1,6 м, тогда радиус 0,4 м. Во втором цилиндре высота будет 0,8 м, тогда радиус 0,8 м. Вычислим отношение объемов двух цилиндров:

Ответ: 1:2.

 

Задания для самостоятельного решения:

Решите задачи:

1) Измерения прямоугольного параллелепипеда: 15 м, 50 м и 36 м. Найти ребро равновеликого ему куба.

2) Измерения прямоугольного бруса: 3 см, 4 см и 5 см. Если увеличить каждое его ребро на х см, то поверхность увеличится на 54 см2. Как увеличится его объем?

3) Осевое сечение цилиндра – квадрат, диагональ которого равна 4. Найти объем цилиндра.

4) Основанием пирамиды служит прямоугольник со сторонами 9 м и 12 м, каждое из боковых ребер равно 12,5 м. Найти объем пирамиды.

5) Основанием пирамиды служит равнобедренный треугольник, у которого равные стороны по 6 см, а основание 8 см. Боковые ребра равны между собой и равны 9 см. Найти объем пирамиды.

6) В прямом параллелепипеде стороны основания равны 8 см и 15 см и образуют угол в 600. Меньшая диагональ параллелепипеда составляем с плоскостью основания угол 300. Найти объем параллелепипеда.

7) Высота и образующая конуса относятся как 4:5, а объем конуса равен 96π см3. Найти полную поверхность конуса.

 

 

Практическое занятие №54

Подобие тел. Отношения объемов подобных тел

 

Цель работы:

студент должен:

знать:

- формулы для вычисления объемов подобных тел;

уметь:

-       вычислять объемы подобных тел.

 

Сведения из теории:

Объемы равных тел равны

Если тело разбито на несколько тел, не имеющих общих внутренних точек, то его объем равен сумме объемов этих тел.

Отношение объемов подобных тел равно кубу коэффициента подобия.

Объем призмы равен

произведению площади ее основания на высоту: V=SoH

или

произведению площади ее перпендикулярного сечения на боковое ребро: V=Sol.

Image

Объем пирамиды равен одной трети произведения площади ее основания на высоту:

Image 

Объемы призм (пирамид), имеющих равновеликие основания, относятся как их высоты.

Объемы призм (пирамид), имеющих равные высоты, относятся как площади их оснований.

Объемы тетраэдров, имеющих общий трехгранный угол, относятся как произведения ребер, содержащих этот угол.

Image

Объем тетраэдра может быть найден по формуле:

, где a и b – длины скрещивающихся ребер, с – расстояние между ними, φ – угол между ними.

Image 

Объем усеченной пирамиды 

Image 

Объем многогранника можно получить, разбив его на не имеющие общих внутренних точек тетраэдры (триангуляция) и суммировав их объемы.

Если в многогранник можно вписать шар, то объем многогранника равен:

,

где R – радиус вписанного шара, Sполн. – площадь полной поверхности многогранника.

 

Пример 126.

Чашка диаметром 8 см и высотой 10 см вмещает 0.5 литра воды. Каких размеров должна быть подобная чашка, вмещающая 4 литра воды?

Решение:

поскольку чашки – подобные цилиндры, то отношение их объёмов равно отношению кубов соответствующих отрезков (в нашем случае – высот и диаметров чашек). Следовательно, высота h новой чашки находится из отношения:

(h/10)3 =4/0,5, т.е. h3=8·103, откуда h=20 см;

Аналогично, для диаметра d получим:

(d/8)3 =4/0,5 , т.е. d3=8·83, откуда d=16 см.

 

Пример 127.

Во сколько, примерно, раз великан ростом в 2 м тяжелее карлика ростом в 1 м?

Решение:

т.к. как фигуры человеческого тела приблизительно подобны, то при вдвое большем росте человек имеет объем не вдвое, а в 8 раз больший. Значит, наш великан весит больше карлика в 8 раз. Самый высокий великан, о котором сохранились сведения, был один житель Эльзаса ростом в 275 см – на целый метр выше человека среднего роста. Самый маленький карлик имел высоту меньше 40 см, т.е. был ниже эльзасца круглым счетом в 7 раз. Поэтому если бы на одну чашку весов поставить великана – эльзасца, то на другую надо бы для равновесия поместить  7·7·7=343  карлика, т.е. целую толпу.

 

Пример 128.

Продаются два арбуза разных размеров. Один на четвертую долю шире другого, а стоит он в полтора раза дороже. Какой из них выгоднее купить?

Решение:

объем большого арбуза превышает объем меньшего в  1,25·1,25·1,25=125/64  раза, т.е. почти вдвое. Выгоднее значит купить крупный арбуз, он дороже только в полтора раза, а съедобного вещества в нем больше раза в два.

 

Пример 129.

Имеются две медные кастрюли одинаковой формы и со стенками одной толщины. Первая в 8 раз вместительнее второй. Во сколько раз она тяжелее?

Решение:

обе кастрюли – тела, геометрически подобные. Если большая кастрюля в 8 раз вместительнее, то все ее линейные размеры в два раза больше: она вдвое выше и вдвое шире. Поэтому ее поверхность больше в  2·2 = 4 раза (поверхности подобных тел относятся, как квадраты линейных размеров). При одинаковой толщине стенок вес кастрюли зависит от величины ее поверхности.

Ответ: большая кастрюля вчетверо тяжелее меньшей кастрюли.

 

Задания для самостоятельного решения:

Решите следующие задачи

1) Строительный кирпич весит 4 кг. Сколько весит игрушечный кирпичик из того же материала, все размеры которого в 4 раза меньше?

2) Продаются две дыни одного сорта. Одна окружностью 60, другая – 50 см. Первая в полтора раза дороже второй. Какую дыню выгоднее купить?

3) Мякоть вишни окружает косточку слоем такой же толщины, как и сама косточка. Будем считать, что и вишня и косточка имеют форму шариков. Можете ли вы сообразить в уме, во сколько раз объем сочной части вишни больше объема косточки?

4) Башня Эйфеля в Париже, 300 м высоты, сделана целиком из железа, которого пошло на нее около 8000000 кг. Я желаю заказать точную железную модель знаменитой башни, висящую всего 1 кг. Какой она будет высоты?

5) Что тяжелее: стакан сахарного песку или такой же стакан колотого сахара?

6) Почему лучина загорается скорее, чем толстое полено, от которого она отколота?

 

Контрольные вопросы:

1.        Чему равны объемы равных тел?

2.        Чему равно отношение объемов подобных тел?

3.        Чему равно отношение объемов призм, пирамид?

 

Практическое занятие №55

Подобие тел. Отношения площадей поверхностей подобных тел

 

Цель работы:

студент должен:

знать:

- формулы для вычисления площадей поверхностей цилиндра, конуса, сферы, подобных фигур;

уметь:

-       уметь вычислять площади поверхностей цилиндра, конуса, сферы, подобных фигур.

 

Сведения из теории:

Площади тел вращения

 

Площадь боковой поверхности цилиндра

S=2πRH

Площадь полной поверхности цилиндра

S=2Soсн.+Sбок.=2πR(R+H)

Image 

Площадь боковой поверхности конуса

S=πRl

Площадь полной поверхности конуса

S=Soсн.+Sбок.=πR(R+l)

Image 

Площадь боковой поверхности усеченного  конуса

Sбок.=π(R+r)l

Площадь полной поверхности усеченного  конуса

Sполн.=π(R2+r2+(R+r)l)

Image 

Площадь поверхности сферы

S=4πR2

Площадь сферической поверхности  сферического сегмента

Sбок.=2πRH

Площадь полной поверхности сферического сегмента

Sполн.=Sбок.+πr2=2πRH.+πr2

Image 

Поверхность вращения отрезка АВ, не  имеющего с осью l общих внутренних точек, равна произведению проекции этого отрезка на ось и длины окружности, радиусом которой служит отрезок серединного перпендикуляра отрезка с концами на оси и на отрезке

Image 

Отношение поверхностей подобных тел

равно квадрату коэффициента подобия

 

Пример 130.

В "Путешествии Гулливера" рассказывается о лилипутах, рост которых в 12 раз меньше нормального. Если на костюм человека нормального роста идет 4 м2 материала, то, сколько материала идет на костюм лилипута?

Решение:

коэффициент подобия =122, т. е. на костюм лилипута идет в 144 раза меньше. Т.о., 40 000 см2 : 144=280 см2.

 

Пример 131.

Один человек на 1/4 ниже другого. Каково отношение поверхностей их тел, считая, что оба тела геометрически подобны?

Решение:

поверхность человека меньшего роста составляет  поверхности более высокого.

 

Пример 132.

Площадь боковой поверхности цилиндра равна 21π, а диаметр основания равен 7. Найдите высоту цилиндра.

Решение:

высота цилиндра равна:

Ответ: 3.

 

Пример 133.

Около конуса описана сфера (сфера содержит окружность основания конуса и его вершину – рис. 77). Центр сферы находится в центре основания конуса. Радиус сферы равен . Найдите образующую конуса.

Решение:

Рисунок 77.

 

высота конуса перпендикулярна основанию и равна радиусу сферы. Тогда по теореме Пифагора получаем:

Радиус сферы равен  поэтому образующая равна:

Ответ: 56.

 

Задания для самостоятельного решения:

Решите задачи:

1) Сторона первого квадрата больше стороны второго квадрата в 2 раза (в 5 раз). Во сколько раз площадь первого квадрата больше площади второго квадрата?

2) Сторона первого квадрата составляет 1/3 (0,1) стороны второго квадрата. Какую часть площадь первого квадрата составляет от площади второго квадрата?

3) Коэффициент подобия в подобных многоугольниках равен 4 (1/5; 0,4; 2,5). Чему равно отношение их площадей?

4) Отношение площадей подобных многоугольников равно 36 (100; 0,09). Чему равно отношение сходственных сторон этих многоугольников?

5) Площадь боковой поверхности цилиндра равна 18π, а диаметр основания равен 9. Найдите высоту цилиндра.

 

Контрольные вопросы:

1.   Запишите формулы площадей поверхности цилиндра, конуса, шара.

2.   Чему равно отношение поверхностей подобных тел?

 

Практическое занятие № 56

Решение уравнений: разложение на множители, введение новых переменных, подстановка

 

Цель работы:

студент должен:

знать:

-       способы решения уравнений;

уметь:

-        решать уравнения различными способами.

 

Сведения из теории:

Метод разложения на множители

Суть данного метода в том, чтобы путем равносильных преобразований представить левую часть исходного уравнения, содержащую неизвестную величину в какой-либо степени, в виде произведения двух выражений, содержащих неизвестную величину в меньшей степени. При этом справа от знака равенства должен оказаться ноль. Проще всего уяснить эту идею на конкретном примере.

 

Пример 83.

Решите уравнение методом разложения на множители: .

Решение:

осуществим разложение на множители (представим исходное выражение в виде произведения). Для этого вынесем переменную  за скобки:

.

Произведение равно нулю тогда и только тогда, когда хотя бы один из множителей равен нулю.

Следовательно,

 или .

Из последнего уравнения получаем:

 или .

Ответ:  и .

 

Задача для самостоятельного решения №1. Решите уравнение методом разложения на множители: .

 

Метод замены переменной

Суть данного метода в том, чтобы удачным образом заменить сложное выражение, содержащее неизвестную величину, новой переменной, в результате чего уравнение принимает более простой вид. Далее полученное уравнение решается относительно новой переменной, после чего происходит возврат к исходной переменной. Все эти идеи проще осознать на конкретном примере.

 

Пример 84.

Решите уравнение методом замены переменной: .

Решение:

такие уравнения называются биквадратными. Перепишем его в виде:

.

Введем новую переменную .Тогда исходное уравнение примет следующий простой вид:

.

Решая полученное квадратичное уравнение, получаем, что:

 или .

Возвращаемся теперь к старой переменной (обратная замена):

 или .

Решений у первого уравнения нет, поскольку не существует такого действительного числа, квадрат которого был бы отрицателен. Второе уравнение имеет два корня .

Ответ: .

 

Задача для самостоятельного решения №2. Решите уравнение методом замены переменной: .

 

Пример 85.

Решите уравнение методом замены переменной: .

Решение:

обращаем внимание на то, что  не является корнем данного уравнения. Следовательно, без потери или приобретения лишних корней можно разделить числитель и знаменатель обеих дробей на. Тогда уравнение принимает вид:

.

Введем новую переменную: . Тогда уравнение примет вид:

.

Выполнив элементарные преобразования: приведем дроби к общему знаменателю, приведем подобные слагаемые, получим:

.

Дробь равна нулю, если нулю равен ее числитель, а знаменатель при этом не равен нулю. То есть уравнение равносильно следующей системе:

.

Решив первое уравнение системы, имеем: t=16 или t=9.

Переходя к обратной подстановке, получаем:

1. , что при  равносильно уравнению , решая которое, получаем  или .

2.  что при  равносильно уравнению , у которого решений нет, поскольку его дискриминант отрицателен.

Ответ: , .

 

Задача для самостоятельного решения №3. Решите уравнение методом разложения на множители: .

 

Контрольные вопросы:

1.   В чем суть решения уравнения методом разложения на множители?

2.   В чем суть решения уравнения методом замены переменной?

 

Практическое занятие № 57

Решение уравнений графическим методом. Нестандартные способы решения уравнений

 

Цель работы:

студент должен:

знать:

-       этапы решения уравнений графическим методом;

уметь:

-       строить графики элементарных функций;

-       решать уравнения различными способами.

 

Сведения из теории:

Метод оценки области значений

Суть данного метода в сравнении областей значений выражений, входящих в уравнение. Часто такой анализ позволяет легко решать сложные уравнения, содержащие различные выражения (рациональные, тригонометрические, логарифмические, показательные и др.). Разберем это на конкретном примере.

 

Пример 86.

Решите уравнение, используя метода оценки области значений: .

Решение:

рассмотрим функцию . Известно, что , поэтому . Итак, функция может принимать значения только из промежутка [0; 1].

Рассмотрим теперь функцию . Графиком данной функции является парабола, ветви которой направлены вверх, а вершина расположена в точке (0; 1).

Т.е. область значений данной функции (те значения, которые может принимать переменная у) представляет собой промежуток [1; +∞).

Т.о. выражения, стоящие справа и слева от знака равенства в исходном уравнении, могут оказаться равными, только если их значения окажутся равными 1, причем при одном и том же значении х. Непосредственной подстановкой убеждаемся, что это условие выполняется при .

Действительно,  и . При всех остальных значениях х функция больше 1. Значит   единственный корень уравнения.

Ответ: 0.

 

Задача для самостоятельного решения №1. Решите уравнение с использованием метода оценки области значений: .

 

Пример 87.

Решите уравнение: .

Решение:

определим область допустимых значений (те значения, которые может принимать переменная х в данном уравнении). Исходим из того, что подкоренное выражение не может быть отрицательным:

.

Решая систему методом интервалов, получаем:

Изображение решений каждого из неравенств системы на числовой прямой

Рисунок 24. Изображение решений системы неравенств на числовой прямой

 

Т.о. область допустимых значений содержит одно единственное значение х=-2. Является ли это значение корнем уравнения, проще всего проверить прямой подстановкой:

,

.

Т.е. х=-2 не является корнем уравнения.

Ответ: корней нет.

 

Задача для самостоятельного решения №2. Решите уравнение: .

 

Пример 88.

Решите уравнение: .

Решение:

помножим уравнение на .

Вообще говоря, это преобразование не является равносильным, даже в области допустимых значений. Ведь могут найтись такие значения х при которых это выражение обратится в нуль. При таком преобразовании могут появиться посторонние корни, поэтому полученные ответы нужно будет проверить непосредственной подстановкой. Но главное, что в результате такого преобразования не произойдет потери корней.

Итак,

,

.

Выражение во вторых скобках не может быть равно нулю. Действительно, оба корня, по крайней мере, неотрицательны, поэтому если к их сумме прибавить 1, получится положительное выражение. То есть остается, что

 или .

Непосредственной подстановкой убеждаемся, что это корень данного уравнения:

, 0=0.

Ответ: 2.

 

Задача для самостоятельного решения №3. Решите уравнение: .

 

Контрольные вопросы:

1.        Поясните суть метода оценки области значений при решении уравнений.

2.     Какие нестандартные способы решения уравнений вы знаете?

 

 

Практическая работа № 58

Решение неравенств методом интервалов

 

Цель работы:

студент должен:

знать:

- правила решения простых, дробно-рациональных неравенств с одной переменной;

уметь:

-        решать неравенства методом интервалов.

 

Сведения из теории:

Пусть заданное неравенство имеет вид: . Для решения этого неравенства используется так называемый метод интервалов, который состоит в следующем.

1. На числовую ось наносят точки х1,   , хn разбивающие ее на промежутки, в которых выражение  определено и сохраняет знак («плюс» или «минус»). Такими точками могут быть корни уравнений  и . Соответствующие этим корням точки отмечают на числовой оси: закрашенными кружками – точки, удовлетворяющие заданному неравенству, а светлыми кружками – не удовлетворяющие ему.

2. Определяют и отмечают на числовой оси знак выражения  для значенийх, принадлежащих каждому из полученных промежутков. Достаточно определить знак функции  в любом таком промежутке, а в остальных промежутках знаки «плюс» и «минус» будут чередоваться.

Изменение знаков удобно иллюстрировать с помощью волнообразной кривой (кривой знаков), проведенной через отмеченные точки и лежащей выше или ниже числовой оси в соответствии со знаком дроби  в рассматриваемом промежутке. Промежутки, которые содержат точки, удовлетворяющие данному неравенству, иногда покрывают штрихами. Заштрихованная область в совокупности с полученными точками будет являться ответом к неравенству:

Прямая знаков метод интервалов

Рисунок 25. Кривая знаков

Пример 89.

Решите неравенство: .

Решение:

упрощаем неравенство путем равносильных преобразований: при умножении или делении обеих частей неравенства на отрицательное число, знак неравенства меняется на противоположный:

.

Приведем дроби к общему знаменателю:

,

,

.

Выражения, стоящие в числителе и знаменателе, можно разложить на множители, тогда неравенство примет вид:

.

Далее находим корни уравнений  и .

Из первого получаем х1=4, х2=1. Из второго получаем х3=2, х4=3.

Наносим на числовую прямую получившиеся точки, причем точки х1, х2 обозначаем закрашенными кружочками (для них неравенство выполняется), а точки х3, х4 светлыми (при этих значениях, выражение, стоящее слева от знака неравенства, не имеет смысла).

Определяем теперь знаки выражения  на полученных промежутках (подставляем любое значение х из каждого полученного промежутка в данное выражение), изображаем кривую знаков, заштриховываем те промежутки, на которых исходное неравенство выполняется:

Кривая знаков решение неравенства методом интервалов

Рисунок 26. Кривая знаков выражения

 

Итак, исходному неравенству удовлетворяют следующие значения: хЄ(-∞; 1]U(2; 3)U[4; +∞).

 

Задача для самостоятельного решения №1. Решите неравенство: .

 

Пример 90.

Решите неравенство: .

Решение:

подкоренное выражение, как известно, не может принимать отрицательных значений, также не допускается нахождение в знаменателе дроби нуля. Следовательно, область допустимых значений данного неравенства определяется неравенством  и тем условием, что .

Решаем уравнения  и .

Из первого уравнения получаем, что х1=9.

Из второго уравнения получаем, что х2=2.

Наносим область допустимых значений неравенства и полученные точки на числовую прямую, причем эти точки будут светлыми, поскольку ни одно из значений не удовлетворяет неравенству. Сразу определяем знаки выражения  в каждом из полученных промежутков и рисуем кривую знаков:

Кривая знаков в методе интервалов решения неравенств

Рисунок 27. Кривая знаков выражения

 

Верхней стрелкой на рисунке обозначена область допустимых значений неравенства. Ответом к неравенству будет являться промежуток, соответствующий на рисунке заштрихованной области.

Ответ: хЄ[0; 2)U(9; +∞).

 

Задача для самостоятельного решения №2. Решите неравенство: .

 

Пример 91.

Решите неравенство: .

Решение:

подкоренное выражение не может принимать отрицательных значений, а в знаменателе дроби не должно быть нуля. Следовательно, область допустимых значений неравенства определяется следующей системой:

.

Решаем уравнение .

Получаем, что х1=0 и . Наносим полученные точки на числовую прямую, не забывая о том, какие из них следует закрасить, а какие осветлить. Изображаем также на ней область допустимых значений и изображаем кривую знаков:

Кривая знаков в методе интервалов решения неравенств

Рисунок 28. Кривая знаков выражения

 

Пунктирные лини на рисунке ограничивают область допустимых значений неравенства. Заштрихованная область соответствует решению неравенства.

Ответ: .

 

Задача для самостоятельного решения №3. Решите неравенство: .

 

Контрольные вопросы:

1.     Дайте определение неравенства с одной переменной.

2.     В чем суть метода интервалов?

 

 

 

 

 

 

 

 

Литература

 

1)  Богомолов В.С. Основы высшей математики. – М., Издательский центр «Академия», 2007.

2)  Богомолов Н.В. Практические занятия по математике: Учебное пособие для средних специальных учебных заведений. – М.: Высшая школа, 2004.

3)  Колмогоров А.Н., Абрамов А.Н., Дудницын Ю.П. и др.; Под ред. А.Н. Колмогорова. Алгебра и начала анализа: Учеб. для 10-11 кл. общеобразоват. учреждений. – 10-е изд. – М.: Просвещение, 2010. – 384 с.: ил.

4)  Лисичкин В.Т., Соловейчик И.Л. Математика: Учеб. пособие для техникумов. – М.: Высш.шк., 1991. – 480 с.: ил.

5)  Луканкин Г.Л. Высшая математика для экономистов: курс лекций: учебное пособие для вузов / Г.Л. Луканкин, А.Г. Луканкин. – 2-е изд., стереотип. – М.: Издательство «Экзамен», 2009. – 285 с.

6)  Математика для техникумов. Алгебра и начала анализа. Под ред. Г.Н.Яковлева – М.: Наука, 1987 – Часть 1.

7)  Математика для техникумов. Алгебра и начала анализа. Под ред. Г.Н.Яковлева – М.: Наука, 1987 – Часть 2.

8)  Математика для техникумов. Геометрия.  Под ред. Г.Н.Яковлева – М.: Наука, 1989.

9)  Математика: Учеб. для студ. образоват. учреждений сред. проф. образования/Игорь Дмитриевич Пехлецкий. – 2-е изд., стереотип. – М., Издательский центр «Академия», 2003. – 304 с.

10)  Михеев В.С. Краткий справочник по математике. – Красногорск, 1996.

11)  Пехлецкий И.Д. Математика – М., Издательский центр «Академия», 2001.

12)  Подольский В.А. и др. Сборник задач по математике: Учебное пособие для средних специальных учебных заведений. – М., Издательский центр «Академия», 2004.

13)  Сергиенко Л.Ю., Самойленко П.И. Планирование учебного процесса по математике: Учеб. – метод. пособие для преподавателей сред. спец. учеб. заведений. – М.: высш. шк., 1987. – 424 с.: ил.

14)  Григорьев С.Г., Иволгина С.В. Математика: учебник для СПО, 2012, http://www.sprinter.ru/books/matematika-uchebnik-7-e-izd-ster-grigorev-2061370.html

15)  http://festival.1september.ru/articles/538170/

Просмотрено: 0%
Просмотрено: 0%
Скачать материал
Скачать материал "МЕТОДИЧЕСКИЕ УКАЗАНИЯ по выполнению практических работ по математике"

Методические разработки к Вашему уроку:

Получите новую специальность за 2 месяца

Управляющий рестораном

Получите профессию

Технолог-калькулятор общественного питания

за 6 месяцев

Пройти курс

Рабочие листы
к вашим урокам

Скачать

Скачать материал

Найдите материал к любому уроку, указав свой предмет (категорию), класс, учебник и тему:

6 669 329 материалов в базе

Скачать материал

Другие материалы

Конспект урока по математике 3 класс УМК "Школа России" "Таблица умножения на 5"
  • Учебник: «Математика (в 2 частях)», Моро М.И., Бантова М.А., Бельтюкова Г.В. и др.
  • Тема: Табличное умножение и деление (продолжение)
  • 02.12.2022
  • 130
  • 0
«Математика (в 2 частях)», Моро М.И., Бантова М.А., Бельтюкова Г.В. и др.

Вам будут интересны эти курсы:

Оставьте свой комментарий

Авторизуйтесь, чтобы задавать вопросы.

  • Скачать материал
    • 03.12.2022 506
    • DOCX 6.5 мбайт
    • Оцените материал:
  • Настоящий материал опубликован пользователем Гайнуллина Наркас Юлаевна. Инфоурок является информационным посредником и предоставляет пользователям возможность размещать на сайте методические материалы. Всю ответственность за опубликованные материалы, содержащиеся в них сведения, а также за соблюдение авторских прав несут пользователи, загрузившие материал на сайт

    Если Вы считаете, что материал нарушает авторские права либо по каким-то другим причинам должен быть удален с сайта, Вы можете оставить жалобу на материал.

    Удалить материал
  • Автор материала

    Гайнуллина Наркас Юлаевна
    Гайнуллина Наркас Юлаевна
    • На сайте: 10 лет
    • Подписчики: 0
    • Всего просмотров: 3509
    • Всего материалов: 2

Ваша скидка на курсы

40%
Скидка для нового слушателя. Войдите на сайт, чтобы применить скидку к любому курсу
Курсы со скидкой

Курс профессиональной переподготовки

Копирайтер

Копирайтер

500/1000 ч.

Подать заявку О курсе

Курс повышения квалификации

Развивающие математические задания для детей и взрослых

36 ч. — 180 ч.

от 1700 руб. от 850 руб.
Подать заявку О курсе
  • Сейчас обучается 66 человек из 26 регионов
  • Этот курс уже прошли 81 человек

Курс повышения квалификации

Практические аспекты применения современных технологий при обучении школьников математике в рамках ФГОС ООО

36 ч. — 144 ч.

от 1700 руб. от 850 руб.
Подать заявку О курсе
  • Сейчас обучается 55 человек из 32 регионов
  • Этот курс уже прошли 414 человек

Курс профессиональной переподготовки

Математика: теория и методика преподавания в образовательной организации

Учитель математики

300/600 ч.

от 7900 руб. от 3650 руб.
Подать заявку О курсе
  • Сейчас обучается 1280 человек из 84 регионов
  • Этот курс уже прошли 3 816 человек

Мини-курс

Вероятность и статистика в рамках обновленного ФГОС

3 ч.

780 руб. 390 руб.
Подать заявку О курсе

Мини-курс

Здоровьесбережение и физическое развитие школьников

3 ч.

780 руб. 390 руб.
Подать заявку О курсе

Мини-курс

Психологическая экспертиза в юридической сфере: теоретические аспекты

2 ч.

780 руб. 390 руб.
Подать заявку О курсе
  • Этот курс уже прошли 11 человек